• Shuffle
    Toggle On
    Toggle Off
  • Alphabetize
    Toggle On
    Toggle Off
  • Front First
    Toggle On
    Toggle Off
  • Both Sides
    Toggle On
    Toggle Off
  • Read
    Toggle On
    Toggle Off
Reading...
Front

Card Range To Study

through

image

Play button

image

Play button

image

Progress

1/231

Click to flip

Use LEFT and RIGHT arrow keys to navigate between flashcards;

Use UP and DOWN arrow keys to flip the card;

H to show hint;

A reads text to speech;

231 Cards in this Set

  • Front
  • Back
A car strikes a pedestrian on the lateral side of her leg. Following the accident, she has "foot drop". Her foot hangs loosely in plantar flexion when she raises it off the ground. She can still invert her foot, but cannot evert it. She can flex but not extend her toes. Which nerve is most likely to have been crushed in her accident?

common fibular
tibial
superficial fibular
medial plantar
sciatic
The correct answer is: common fibular
All of these symptoms are consistent with damage to the common fibular nerve, whose branches, the deep and superficial fibular nerves, innervate the anterior and lateral compartments. The anterior compartment is important for dorsiflexion. When the tibialis anterior of this compartment is denervated, you will see foot drop. This compartment also contains extensor hallucis longus and extensor digitorum longus. These muscles and their brevis counterparts on the dorsum of the foot (which are also innervated by the deep fibular nerve) are responsible for extending the toes. Since this function is also lost, it is clear that the injury has affected the deep fibular nerve. Finally, the loss of eversion suggests that the lateral, everter compartment has been damaged--it is innervated by the superficial fibular nerve.

If the injury was only to the superficial fibular nerve, you would not expect foot drop or difficulty extending the toes. Instead, the main symptom would be a loss of cutaneous sensation over the distal third of the leg and the dorsum of the foot. An injury to the tibial nerve would lead to problems with plantarflexion due to denervation of the posterior compartment. The medial plantar nerve innervates structures in the foot and would not produce these symptoms. Finally, the sciatic nerve gives rise to the tibial and common fibular nerves. An injury to the sciatic nerve would lead to deficits in all the compartments of the leg.
A player is kicked on the front of his leg during a soccer game, and a large bruise develops. A hematoma deep to the crural fascia can create extreme pressure within the anterior compartment of the leg, compressing structures within it. The most likely finding resulting from this anterior compartment syndrome is:
numbness on the dorsum of the foot
inability to evert the foot
inability to invert the foot
foot drop
inability to plantarflex the foot
The correct answer is: foot drop
Anterior compartment syndrome leads to foot drop due to the compression of the deep fibular nerve. Since the deep fibular nerve innervates tibialis anterior, an important dorsiflexer, injuring this nerve will impair dorsiflexion and cause the foot to drop. Numbness on the dorsum of the foot would suggest an injury to the superficial fibular nerve. An inability to evert the foot might also be due to an injury to the superficial fibular nerve, since that nerve supplies the lateral, everter compartment. Damage to the deep fibular nerve might also impair inversion since that is an action of tibialis anterior, but this would be a more subtle finding. Also, remember that tibialis posterior is also an inverter, so that muscle might be able to compensate for the injury to the anterior compartment. Finally, an inability to plantarflex the foot would stem from damage to the tibial nerve and the posterior compartment.
A patient has been diagnosed with bone cancer in the fibula that necessitates its removal. Which of the following muscles would be least affected following removal of the fibula?
biceps femoris
extensor digitorum longus
flexor digitorum longus
flexor hallucis longus
peroneus tertius
The correct answer is: flexor digitorum longus
Flexor digitorum longus is the most medial muscle in the deep posterior compartment of the leg. This means that it takes origin from the middle half of the posterior surface of the tibia and is not attached to the fibula in any way. Biceps femoris inserts on the head of the fibula and the lateral condyle of the tibia. Extensor digitorum longus is a muscle on the lateral side of the anterior compartment of the leg. This means that it takes origin from the fibula, and would be affected by its removal. Flexor hallucis longus is the most lateral muscle in the posterior compartment, so it originates from the fibula, too. Finally, fibularis (peroneus) longus and brevis, the two muscles of the lateral compartment, both take origin from the fibula.
Your patient was struck by a car's bumper as she crossed the street, and her fibular neck is broken. After the bone has healed, she has "foot drop", i. e. she cannot dorsiflex her foot, and so it flops onto the ground during walking. Denervation (paralysis) of which of the following muscles would be associated with foot drop?
fibularis longus
tibialis posterior
fibularis brevis
tibialis anterior
popliteus
The correct answer is: tibialis anterior
Tibialis anterior is the major dorsiflexer of the foot--if it is damaged, you will observe foot drop. It is found in the anterior compartment and is innervated by the deep fibular nerve. This patient probably damaged her common fibular nerve in the accident. This nerve wraps around the neck of the fibula before giving off its two branches: the deep fibular nerve and the superficial fibular nerve. You might hypothesize that this patient would also have a loss of cutaneous sensation on the distal third of the anterior leg and the dorsum of the foot, since those are the areas that receive cutaneous innervation from the superficial fibular nerve.

None of the other muscles listed are dorsiflexers. Fibularis longus and brevis evert and plantarflex the foot; tibialis posterior plantarflexes and inverts the foot; popliteus flexes and rotates the leg medially so that the knee can unlock.
A pedestrian is struck by a car, and his fibular neck is fractured. There is no indication of foot drop, but he cannot evert his foot and the top of his foot is numb. This apparent nerve lesion would affect which of the following muscles?
Tibialis posterior
Tibialis anterior
Fibularis tertius
Fibularis longus
Adductor hallucis
he correct answer is: fibularis longus
A fracture of the fibular neck commonly causes an injury to the common fibular nerve, which has two branches: the deep fibular nerve and the superficial fibular nerve. However, this case isn't as simple. If the common fibular nerve was damaged, the deep fibular nerve would also be impaired. This would mean that the anterior compartment of the leg would be denervated, and the patient would suffer from foot drop. But that's not happening here, so you know that the common fibular nerve must be intact. The superficial fibular nerve, however, innervates the lateral compartment of the leg which allows for eversion. It also provides cutaneous sensation to the dorsum of the foot. An injury to this nerve fits with the patient's symptoms. The superficial fibular nerve innervates fibularis longus and brevis, so D is your answer.

Tibialis anterior and fibularis tertius are both muscles in the anterior compartment of the leg--they are innervated by the deep fibular nerve. You should know that this compartment of the leg is intact because there is no foot drop. Tibialis posterior is innervated by the tibial nerve--this nerve was not involved in the accident. Finally, adductor hallucis is a foot muscle innervated by the deep branch of the lateral plantar nerve, which is a branch of the tibial nerve.
While at the beach in Florida after final exams, a medical student steps on a stingray, which responds by stinging her in the ankle. The stinger pierces the skin, subcutaneous tissue, and flexor retinaculum of the ankle. Which other structure passing under the retinaculum may be injured?
Tibial nerve
Tibialis anterior
Quadratus plantae
Anterior tibial artery
Plantar arterial arch
The correct answer is: Tibial nerve
The flexor retinaculum is immediately posterior to the medial malleolus. The structures which pass under the flexor retinaculum are coming from the posterior compartment of the leg to enter the foot. These tendons, vessels, and nerve are all organized behind the flexor retinaculum in a very characteristic way. From anterior to posterior the structures are: tendon of Tibialis posterior, tendon of flexor Digitorum longus, posterior tibial Artery (and vein), tibial Nerve, and tendon of flexor Hallucis longus. Tom, Dick, ANd Harry! Out of all the answer choices, the tibial nerve is the only one which lies behind the flexor retinaculum, so that's your answer.

Tibialis anterior is in the anterior compartment of the leg--its tendon just crosses under the extensor retinaculum to enter the dorsum of the foot. The anterior tibial artery is also in the anterior compartment of the leg. It crosses the ankle under the extensor retinaculum and enters the dorsum of the foot as dorsalis pedis. Quadratus plantae is a deep muscle of the plantar surface of the foot--it insures that the tendons from flexor digitorum longus flex the toes properly. The plantar arterial arch is a structure that supplies blood to the deep foot. None of these other structures are associated with the flexor retinaculum.
A long distance runner complained of swelling and pain of his shin. At physical examination, skin testing showed normal cutaneous sensation of the leg. However, muscular strength tests showed marked weakness of dorsiflexion and impaired inversion of the foot. Which nerve serves the muscles involved?
common fibular
deep fibular
sciatic
superficial fibular
tibial
The correct answer is: deep fibular nerve
The deep fibular nerve provides motor innervaton to the anterior compartment of the leg. This compartment contains tibialis anterior, a muscle that allows for dorsiflexion and inversion of the foot. If a patient is unable to dorsiflex the ankle, he or she will have foot drop. When the deep fibular nerve is damaged, cutaneous sensation to the leg and foot is normal, except for the loss of sensation on the web of skin between the first and second toe. This is the only place were the deep fibular nerve supplies cutaneous innervation. This patient probably has anterior compartment syndrome, which occurs when the muscles in the anterior compartment of the leg swell and press on the bone or fascial lining. Eventually, this swelling can compress the deep fibular nerve, leading to foot drop, or the anterior tibial artery, stopping blood flow to the dorsum of the foot.

Injuries to the common fibular nerve often occur after fibular fracture because the common fibular nerve wraps around the neck of the fibula. If this nerve was injured, you would see the same symptom of foot drop, but it would be accompanied by a loss of cutaneous sensation to the anterolateral aspect of the leg and dorsum of the foot. The sciatic nerve branches to form the common fibular and tibial nerve. An injury to this nerve would damage all of the compartments in the leg. The superficial fibular nerve innervates the lateral compartment of the leg, which everts the foot. It also provides cutaneous sensation to the distal third of the anterior leg and the dorsum of the foot, so an injury here would lead to a significant loss of cutaneous sensation. Finally, the tibial nerve innervates the posterior compartment of the leg. An injury to this nerve would impair plantarflexion of the foot.
A patient with a fracture to the left upper tibia was treated with a plaster cast. A few days later he started to develop progressive numbness over the dorsum of the foot and weakness in dorsiflexion. The cast was quickly changed and the signs were attributed to nerve compression. The compressed nerve was most likely the:
Tibial
Obturator
Sciatic
Femoral
Common fibular
The correct answer is: Common fibular
The common fibular nerve must have been compressed in the cast. You can figure this out by thinking about the two branches of the common fibular nerve. The first branch is the deep fibular nerve which innervates the anterior compartment of the leg and is responsible for dorsiflexion of the foot. The second branch, the superficial fibular nerve, innervates the lateral (everter) compartment and provides cutaneous innervation to the dorsum of the foot. The combination of symptoms (impaired dorsiflexion and a loss of cutaneous sensation on the dorsum of the foot) suggests that both nerves are injured. So the common fibular nerve, which both nerves branch from, must be the structure that was damaged.

The tibial nerve innervates the posterior compartment of the leg--an injury to this nerve would result in an inability to plantarflex the foot. The obturator nerve innervates the medial compartment of the thigh--damage to this nerve would impair adduction of the hip. The sciatic nerve branches to the tibial and common fibular nerve--an injury to this nerve would result in a great motor and sensory deficit in the leg and thigh. Finally, the femoral nerve innervates the anterior compartment of the thigh--damage to this nerve would denervate the quadriceps and impair extension at the knee.
The most usual site for feeling the pulsations of the dorsalis pedis artery in the foot is:
Just behind the medial malleolus
Just lateral to the tendon of extensor hallucis longus
Behind the tendon of fibularis tertius muscle
In the second dorsal metatarsal space
Just behind the lateral malleolus.
The correct answer is: Just lateral to the tendon of extensor hallucius longus muscle
The dorsalis pedis artery is the continuation of the anterior tibial artery which continues on to the dorsum of the foot. The name change from anterior tibial to dorsalis pedis occurs at the level of the ankle. As the artery crosses into the foot, it lies just lateral to the tendon of extensor hallucis longus, so that's where you would feel a pulse. See Netter Plate 513 for a picture.

The pulse of the posterior tibial artery, which comes from the posterior compartment of the leg, might be felt behind the medial malleolus. The pulse of the fibular artery might be felt behind the lateral malleolus, but that pulse would be very weak. There are no special pulses associated with the tendon of fibularis tertius or the second dorsal metatarsal space.
The lateral plantar nerve is a branch of which nerve?
Deep fibular
Femoral
Saphenous
Sural
Tibial
The correct answer is: Tibial nerve
The lateral and medial plantar nerves are both branches of the tibial nerve. These branches continue to the plantar surface of the foot, innervating the muscles on the plantar surface of the foot and providing cutaneous innervation to the skin of the sole. The deep fibular nerve innervates the anterior compartment of the leg, the muscles on the dorsum of the foot, and provides sensory innervation to the web of skin between the first and second toe. The femoral nerve innervates the anterior (quadriceps) compartment of the thigh, which allows for extension at the knee. The saphenous nerve is a branch of the femoral nerve that travels with the great saphenous vein; it provides cutaneous innervation to the skin of the medial side of the leg and medial side of the foot. The sural nerve is a cutaneous nerve that provides sensory innervation to the skin of the posterior surface of the lower leg and the skin of the lateral side of the foot.
A medical student goes for a swim while on vacation in Florida after final exams. A barracuda bites his foot, severing an artery on the dorsum of his foot just below his ankle. Which artery has been severed?
Anterior tibial
Deep fibular
Deep plantar
Dorsalis pedis
Dorsal metatarsal
The correct answer is: Dorsalis pedis
The dorsalis pedis is a continuation of the anterior tibial artery--the name change here occurs at the level of the ankle. Since the artery on the dorsum of the foot was cut just below the ankle, dorsalis pedis must have been the vessel that was injured. The deep plantar artery is a branch of dorsalis pedis--it supplies the deep foot, and reaches this area by passing through a space between the first and second metatarsals on the dorsum of the foot. The dorsal metatarsal arteries are small arteries that come off of the arcuate artery and travel to the individual digits. They supply the dorsum of digits, excluding the distal phalangeal segment. There is no such thing as a deep fibular artery, but it sounds good.
While walking barefoot on the beach in Florida following final exams, a medical student steps on a sharp shell which punctures the sole of her foot. She notices that she can no longer spread her toes apart (without using her hands, that is). Which nerve must have been injured?
deep fibular
lateral plantar
medial plantar
sural
tibial
The correct answer is: lateral plantar
This medical student is unable to abduct her toes--this must mean that her dorsal interosseus muscles are denervated. These dorsal interosseus muscles are innervated by the deep branch of the lateral plantar nerve, so this must be the nerve that was damaged. The medial plantar nerve innervates muscles on the plantar surface of the foot, including abductor hallucis, flexor hallucis brevis, and flexor digitorum brevis. None of these muscles are responsible for abducting the toes.

As far as the other nerves go... you should know that the lateral and medial plantar nerves are the branches of the tibial nerve that continue to the plantar surface of the foot. However, the tibial nerve proper innervates the posterior compartment of the leg, allowing for plantarflexion of the foot. The deep fibular nerve is the nerve that innervates the anterior compartment of the leg--it also continues to the dorsum of the foot and innervates extensor hallucis brevis and extensor digitorum brevis. Finally, note that the sural nerve provides cutaneous innervation to the skin of the posterior surface of the lower leg and the skin of the lateral side of the foot--it is not a motor nerve.
In an accident involving farm machinery, a farmer recieves a cut on the dorsum of his ankle. As you inspect the wound and test for functional and sensory deficits, you find that no tendons have been cut, but the dorsalis pedis artery and the accompanying nerve have been cut. You would expect to find:
club foot
foot drop
inability to extend the big toe
numbness between the first and second toes
weakness in inversion of the foot
The correct answer is: numbness between the first and second toes
The nerve running with the dorsalis pedis artery is the deep fibular nerve. This nerve innervates the anterior compartment of the leg and the extensors on the dorsum of the foot. At the point where it was cut, the nerve had already given off all its branches to the anterior compartment of the leg. So, the farmer should still be able to dorsiflex his foot (preventing foot drop), and all of the muscles that invert his foot should be intact. Although extensor hallucis brevis (on the dorsum of the foot) has been denervated, extensor hallucis longus is in the anterior compartment of the leg, and that muscle should still be able to extend the great toe. However, the deep fibular nerve would have been cut before it could supply fibers to the area of cutaneous innervation between the first and second toe. So, there would be numbness in that area of skin.

Club foot is a congenital anomaly which refers to a foot that has been twisted out of position. In cases of club foot, the foot is inverted, the ankle is plantarflexed, and the forefoot is adducted.
A construction worker steps on a board with a nail in it. The nail pierces the sole of his boot and enters his foot 2 cm anterior to his calcaneal tuberosity, at the middle of the width of his foot. Before reaching bone, the nail would pass through, in order:
Skin, calcaneometatarsal ligament, flexor digitorum brevis, quadratus plantae, long plantar ligament
Skin, plantar aponeurosis, flexor digitorum brevis, quadratus plantae, long plantar ligament
Skin, plantar aponeurosis, flexor digitorum brevis, tendons of the flexor digitorum longus, long plantar ligament
Skin, plantar aponeurosis, quadratus plantae, flexor digitorum brevis, long plantar ligament
Skin,transverse metatarsal ligament, flexor digitorum brevis, quadratus plantae, long plantar ligament
The correct answer is: Skin, plantar aponeurosis, flexor digitorum brevis, quadratus plantae, long plantar ligament
2 cm anterior to the calcaneal tuberosity, the nail would pass through skin, the plantar aponeurosis, flexor digitorum brevis (a superficial muscle), quadratus plantae (a deeper muscle), and the long plantar ligament. See the pictures in Netter plates 496-500 to get a better idea of this. The calcaneometatarsal ligament is the lateral band of the plantar aponeurosis--it is on the lateral part of the foot, and would not be found at the "middle of the width of his foot." Tendons of flexor digitorum longus are found far anterior in the foot--not just 2 cm anterior to the calcaneal tuberosity. The transverse metatarsal ligaments are also found far anterior in the foot, at the distal ends of the metatarsals.
Pain may result from friction of the tendon of which muscle that passes deep to the anterior fibers of the long plantar ligament.
tibialis posterior
flexor hallucis longus
fibularis longus
tibialis anterior
flexor hallucis brevis
The correct answer is: fibularis longus
Remember: fibularis longus runs from the lateral side of the foot across the entire sole of the foot, traveling deep to the long plantar ligament. The friction between these two closely related structures may lead to discomfort. The tendon of tibialis posterior crosses behind the medial malleolus and enters the plantar surface of the foot on the medial side, inserting on navicular, the medial cuneiform, and metatarsals 2-4. The tendon of flexor digitorum longus travels posterior to the medial malleolus, and inserts on the bases of the distal phalanges of digits. The tendon of flexor hallucius longus also crosses behind the medial malleolus, travels under sustentaculum tali, and inserts on the base of the distal phalanx of the first toe. Finally, tibialis anterior does not really cross onto the plantar surface. It travels across the dorsum of the foot, and its tendon inserts on the medial surface of the medial cuneiform and the first metatarsal.
Compression of the lateral plantar nerve as it passes between the flexor digitorum brevis and quadratus plantae could result in weakness of any of the following actions EXCEPT:
abduction of the great toe
adduction of the middle toe
abduction of the little toe
adduction of the great toe
abduction of the middle toe
The correct answer is: abduction of the great toe
The medial plantar nerve innervates abductor hallucis, so this muscle would be saved if the lateral plantar nerve was disrupted. The muscles responsible for all the rest of the actions listed are innervated by the lateral plantar nerve, or its branch, the deep lateral plantar nerve: Adduction of the middle toe: plantar interosseus muscles (deep branch); Abduction of the little toe: abductor digiti minimi (lateral plantar); Adduction of the great toe: adductor hallucis (deep branch); Abduction of the middle toe: third dorsal interosseus muscle (deep branch).
A construction worker lacerates the dorsum of his foot just below his ankle. Profuse bleeding that occurs would result from cutting which vessel?
small saphenous vein
dorsalis pedis artery
medial plantar artery
anterior tibial artery
fibular artery
The correct answer is: dorsalis pedis
The dorsalis pedis is an extension of the anterior tibial artery that supplies the dorsal aspect of the foot. (The name change occurs at the level of the ankle, so that's why this isn't the anterior tibial artery.) Dorsalis pedis travels on the dorsum of the foot, slightly toward the medial side, and it would be the artery most likely damaged by this injury. The lesser saphenous vein forms from the dorsal venous arch of the foot. It travels on the lateral side of the foot, and, at the ankle, passes posterior to the lateral malleolus. So, the lesser saphenous vein would not be traveling on the dorsum of the foot by the time it reached the ankle. The medial plantar artery is not found on the dorsum of the foot; it would not be damaged by this injury. Finally, the fibular artery is a branch of the posterior tibial artery. It supplies the lateral compartment of the leg and does not send blood to the foot.
A soldier complains of foot pain following a 50 mile hike. Upon examination, the physician diagnoses tendonitis of the fibularis longus tendon. Because the tenderness is located deeply on the sole of the foot, it appears that the irritation occurred where the tendon lies against bone, covered by a structure called the:
long plantar ligament
plantar aponeurosis
short plantar ligament
spring ligament
tendon of tibialis posterior
The correct answer is: long plantar ligament
Remember: the fibularis longus tendon enters the foot on the lateral side, then arches across the foot to insert on the medial cuneiform and the base of the first metatarsal. The long plantar ligament lies directly over this tendon. The long plantar ligament is a deep structure that connects calcaneus with cuboid and the bases of the lateral three metatarsals. If the fibularis longus tendon was inflamed, the long plantar ligament might also be affected.

The plantar aponeurosis is a very superficial structure that covers and protects the sole of the entire foot--it is not closely associated with deep structures like the long plantar ligament. The short plantar ligament (also called the calcaneocuboid ligament) connects calcaneus and cuboid. The spring ligament connects sustantaculum tali with the inferior surface of navicular. Finally, the tendon of tibialis posterior enters the foot after traveling posterior to the medial malleolus and inserts on navicular, the medial cuneiform, and metatarsals 2-4.
A deep laceration, 2 cm in length, immediately posterior to the medial malleolus, may injure any of the following EXCEPT:
fibular artery
tibial nerve
tendon of tibialis posterior m.
tendon of flexor digitorum longus m.
tendon of flexor hallucis longus m.
The correct answer is: Fibular artery
The flexor retinaculum is immediately posterior to the medial malleolus. So, any of the tendons or structures coming from the posterior compartment and crossing under flexor retinaculum to the plantar surface of the foot might be injured. The fibular artery is a branch of the posterior tibial artery that delivers blood to the lateral compartment of the leg. It does not cross into the foot, and it is nowhere near the medial malleolus. So that's your answer.

All of the other listed structures are coming from the posterior, flexor compartment--these structures are all organized behind the flexor retinaculum in a very characteristic way. From anterior to posterior the structures are: tendon of Tibialis posterior, tendon of flexor Digitorum longus, posterior tibial Artery (and vein), tibial Nerve, and tendon of flexor Hallucis longus. So, T, D, A, N, H equals Tom, Dick, ANd Harry. This is an important relationship to remember!
A patient has stepped on a board with a long nail sticking up from it, and the nail penetrated the patient's foot between the bases of the first and second metatarsals. What artery is most likely injured at this location?
arcuate
deep plantar
dorsalis pedis
lateral plantar
medial plantar
The correct answer is: deep plantar artery
The deep plantar artery is a branch of dorsalis pedis, the major artery supplying blood to the dorsum of the foot. The deep plantar artery dives from the dorsum of the foot to the sole of the foot between the two heads of the dorsal interosseous muscle between the first and second toes. It then unites with the lateral plantar artery to form the plantar arterial arch. The arcuate artery is on the dorsum of the foot. It would be unlikely to injure this artery from the plantar surface. The medial plantar artery is also on the medial side of the foot and it does distribute to the area where the injury occured. However, the medial plantar artery supplies blood to the superficial plantar surface of the foot, and the scenario in the question points to an injury that would disturb deep structures in the foot. So, deep plantar artery is the best answer.
A worker falls from a height and lands on his feet. Radiographs reveal a fracture of the sustentaculum tali. The muscle passing immediately beneath it that would be adversely affected is the:
fibularis longus
flexor digitorum longus
flexor hallucis longus
tibialis anterior
tibialis posterior
The correct answer is: flexor hallucis longus
The tendon of flexor hallucius longus passes under sustentaculum tali, creating a groove in the bone. The sustentaculum tali is a shelf-like, medial projection of the calcaneus, which supports the talus. So, if the sustenaculum tali was fractured, the tendon of flexor hallucis longus would be displaced from its usual position and the muscle would be affected. The fibularis longus tendon enters the foot on the lateral side. It grooves the cuboid bone and travels deep to the long plantar ligament to insert on the medial cuneiform bone. The tendon of flexor digitorum longus crosses onto the plantar surface anterior to sustentaculum tali and eventually divides into 4 tendons that insert into the bases of the distal phalanges of digits 2-5. The tendon from tibialis anterior crosses the dorsal side of the foot and inserts on the medial surface of the first cuneiform and the first metatarsal. Finally, the tibialis posterior tendon crosses under the foot on the medial side, anterior to both flexor hallucis longus and flexor digitorum longus. It inserts on navicular, the medial cuneiform, and metatarsals 2-4. So, you should not see any of these other tendons associated with sustenaculum tali.
During recovery from a gunshot wound of the right pelvis, the patient notices a lurch in his gait. When he lifts his left foot off of the ground, his pelvis dips down on the left side. The nerve that appears to have been injured is the:
nerve to piriformis
nerve to obturator internus
sciatic nerve
femoral nerve
superior gluteal nerve
The correct answer is: superior guteal nerve
The superior gluteal nerve supplies gluteus minimus and medius--two muscles that are important abductors of the hip--as well as tensor fasciae latae. These muscles stabilize the pelvis when walking. The gluteus medius and minimus work in such a way that when you are standing on your right leg only, the muscles on the right side are supporting the left side of the pelvis. When you are standing on your left leg only, the muscles of the left side are supporting the right side of the pelvis. If a patient exhibits a characteristic hip drop on the uninjured side (in this case, the left side) while standing on the injured side (right), this is called Trendelenburg's sign. It occurs when the superior gluteal nerve--the nerve supply to the abductors of the thigh--is disrupted due to injury or disease.

Nerve to piriformis and nerve to obturator internus allow those muscles to laterally rotate the thigh. However, if one of these muscles was denervated, the other might be able to compensate and prevent a significant loss of function. The sciatic nerve has no direct muscular branches; however, its two branches, common fibular and tibial nerves, innervate the posterior compartment of the thigh and all the muscles of the leg. A disruption to the sciatic nerve would lead to a significant motor deficit. The femoral nerve innervates the quadriceps. If this nerve were damaged, a patient would present with an inability to extend the knee.
The team doctor tells a football player that he has "a pulled hamstring" muscle. This results from a tearing of the origin of a hamstring muscle from the:
sacrum
posterior gluteal line
ischial tuberosity
obturator membrane
iliac tubercle
The correct answer is: ischial tuberosity
The hamstring muscles are: biceps femoris, semimembranosus, and semitendinosus. They originate from the ischial tuberosity and insert on the tibia and fibula (biceps). They comprise the posterior compartment of the thigh and are innervated by the tibial nerve, with the exception of the short head of biceps femoris which is innervated by the common fibular nerve. These muscles allow for extension at the hip and flexion at the knee. The anterior sacrum is the origin of the piriformis muscle, while the posterior sacrum and ilium posterior to the superior gluteal line serve as the origin of gluteus maximus. Obturator internus and externus take origin from the obturator membrane and the margins of the obturator foramen.
If the head of the femur is dislocated postero-medially, compression of which nerve is likely to result?
Femoral
Lumbosacral trunk
Obturator
Sciatic
Superior gluteal
The correct answer is: sciatic nerve
The sciatic nerve is closely related to the posterior hip joint, which makes this nerve very vulnerable in cases where the femur is dislocated postero-medially. If the sciatic nerve was completely paralyzed, the compartments innervated by its two branches: the common fibular and tibial nerves, would lose function. This would mean that the hamstrings and all the muscles of below the knee would lose their innervation. (Luckily, complete paralysis of the sciatic nerve is very rare.) The hip joint is very stable, so it is difficult to dislocate the femur. Most dislocations occur when the hip is flexed and the thigh is adducted. In flexion, the joint capsule is lax, and the femoral head tends to dislocate posteriorly when forces drive the femur posteriorly. This means that the sciatic nerve will be very vulnerable when the femur is dislocated!

The other nerves listed in the question are not closely related to the hip joint. The femoral nerve innervates the quadriceps and is on the anterior of the thigh. The lumbosacral trunk is located in the pelvis. The obturator nerve innervates the adductor compartment, and is on the anteromedial side of the thigh. Although the superior gluteal nerve innervates muscles near the hip socket (gluteus medius, minimus, and tensor fasciae latae), it would not be damaged by a dislocated hip.
What muscle passes through the lesser sciatic foramen?
Gluteus minimus
Obturator internus
Piriformis
Quadratus femoris
Superior gemellus
The correct answer is: obturator internus
Obturator internus leaves the pelvis by passing through the lesser sciatic foramen. It eventually inserts on the greater trochanter of the femur and helps to laterally rotate and abduct the thigh. Piriformis leaves the pelvis through the greater sciatic foramen and also inserts of the greater trochanter of the femur. It helps with the same movements as obturator internus--lateral rotation and abduction of the thigh.

The other muscles listed act at the hip, but they are not related to the greater or lesser sciatic foramen. Gluteus minimus originates on the ilium and inserts on the greater trochanter of the femur--it abducts and medially rotates the thigh. Quadratus femoris is a lateral rotator of the thigh which originates on the ischial tuberosity and inserts on the quadrate line. Superior gemellus is another lateral rotator of the thigh which inserts with obturator internus on the obturator tendon.
In order to avoid injury to the sciatic nerve, intramuscular injections should be given in which quadrant of the buttock?
upper medial
upper lateral
lower medial
lower lateral
middle
The correct answer is: upper lateral
An injection in the posterior hip region is best placed in the upper, outer quadrant of the hip, just inferior and posterior to the anterior superior iliac spine. This is an area where there are not many nerves and vessels that could be injured by the injection. There are several nerves in the posterior hip region that may be injured if intramuscular injections are carelessly placed in the other quadrants of the buttock, including the sciatic nerve, posterior femoral cutaneous nerve, inferior gluteal nerve, and superior gluteal nerve.
The femoral artery enters the popliteal fossa (becoming the popliteal artery) by passing through the:
Adductor hiatus
Femoral canal
Femoral sheath
Femoral triangle
Saphenous opening
The correct answer is: Adductor hiatus
The adductor hiatus is a split in the adductor magnus muscle found at the end of the adductor canal. At the adductor hiatus, the femoral vessels pass through to reach the posterior surface of the leg, changing names to become the popliteal vessels. The femoral triangle is an area in the proximal anteromedial thigh, bounded by the inguinal ligament, sartorius, and the medial edge of adductor longus. It contains the femoral nerve and the three structures covered by the femoral sheath: the femoral artery, vein, and canal. The femoral canal is a structure in the femoral sheath that usually holds a deep inguinal lymph node; it may also be the site of a femoral hernia. The saphenous opening is an opening in the fascia lata where the great saphenous vein passes deep to join the femoral vein.
The short head of biceps femoris muscle is innervated by which nerve?
Common fibular
Femoral
Inferior gluteal
Obturator
Tibial
The correct answer is: Common fibular
The short head of biceps femoris is innervated by the common fibular nerve; all the other muscles in the hamstring compartment are innervated by the tibial nerve. Both of these nerves are branches of the sciatic nerve. The inferior gluteal nerve innervates gluteus maximus. The obturator nerve innervates the medial, adductor compartment of the thigh. The femoral nerve innervates the muscles of the anterior thigh.
As a patient with paralyzed gluteus medius and minimus muscles on the left side attempts to stand on the left limb only, the right side of the pelvis typically:
Drops
Elevates
Rotates laterally
Rotates medially
Thrusts forward
The correct answer is: Drops
When gluteus medius and minimus are injured, a patient will show a positive Trendelenburg sign. This means that when this patient attempts to stand on the left leg only, the uninjured right hip will drop. This is because gluteus medius and minimus on the stable left leg support the pelvis so that it will remain level when the right leg is lifted--when these muscles are injured on the supported left side, the right side of the pelvis will drop.

Make sure you understand what Trendelenburg sign is, what it means, and why the pelvis drops on the uninjured side!
When the femur is fractured, the broken distal end often turns posteriorly to enter the popliteal fossa due to muscle traction. Because of its position deepest in the fossa, which structure is most vulnerable to laceration?
Common fibular n.
Lesser saphenous v.
Popliteal a.
Popliteal v.
Tibial n.
The correct answer is: Popliteal artery
The popliteal fossa contains the popliteal artery, popliteal vein, common fibular nerve, and tibial nerve. The popliteal artery is the deepest structure in the fossa--it lies on the posterior side of the knee joint. So, it is the structure that would be most likely to be lacerated by the broken distal end of the femur. The popliteal vein lies superficial to the artery, and the tibial nerve lies superficial to the popliteal vein. These three structures are right in the middle of the fossa. The common fibular nerve descends toward the fibular neck, on the lateral side of the popliteal fossa. The lesser saphenous vein drains the superficial posterolateral lower limb. It eventually terminates in the popliteal vein.
The deep femoral artery is the principle blood source for the muscles in which compartment of the thigh?
Anterior
Lateral (gluteal)
Medial (adductor)
Posterior (hamstring)
The correct answer is: Posterior
The deep femoral artery supplies the posterior compartment of the thigh with three to four perforating arteries. These arteries pierce adductor magnus and supply blood to the hamstrings--biceps femoris, semitendinosus, and semimembranosus. The anterior compartment of the thigh (the quadriceps) receives blood from the femoral artery. The medial compartment of the thigh receives blood from the obturator artery and medial circumflex femoral artery, as well as the deep femoral. The gluteal region receives blood from the superior and inferior gluteal arteries.
After suffering a deep stab wound in the medial upper quadrant of the right buttock, an emergency room patient found walking to be very difficult. The basic problem was that, during stepping, her left hip sagged down as soon as the left foot was lifted off the ground to swing forward. What nerve was damaged?
Femoral
Inferior gluteal
Obturator
Pudendal
Superior gluteal
The correct answer is: superior gluteal
The superior gluteal nerve supplies gluteus minimus and medius--two muscles that are important abductors of the hip--and tensor fasciae latae. These muscles stabilize the pelvis when walking. They work in such a way that when you are standing on your right leg only, the muscles on the right side are holding the left side of the pelvis level. When you are standing on your left leg only, the muscles of the left side are supporting the right side of the pelvis. So, if a patient exhibits this characteristic hip drop on the uninjured side while standing on the injured side, this is called Trendelenburg sign. It demonstrates that the superior gluteal nerve was injured and the gluteus minimus and medius can no longer support the hip.
A fracture of the ischial tuberosity might be expected to most directly affect the muscles that produce which lower limb movement?
Abduction at the hip
Adduction at the hip
Extension at the knee
Flexion at the hip
Flexion at the knee
The correct answer is: flexion at the knee
The ischial tuberosity is the origin for the hamstrings muscles which are the muscles that allow for extension at the hip and flexion at the knee. If the ischial tuberosity was fractured, the hamstrings would be separated from their origin and would not function properly. The most important hip abductors are gluteus medius and minimus. These muscles are most commonly damaged by an injury to the superior gluteal nerve. The hip adductors are adductor longus, brevis and magnus, which insert on the linea aspera of the femur and are innervated by the obturator nerve. The muscles responsible for extending the knee are the quadriceps. They take origin from the surface of the femur and the anterior inferior iliac spine (rectus femoris). The quadriceps are innervated by the femoral nerve. Finally, the hip flexors are rectus femoris (from the quadriceps), pectineus, iliopsoas, and sartorius. These muscles have diverse origins, and are mostly innervated by the femoral nerve.
When, in approximately 12% of people, the common fibular nerve passes through the piriformis muscle, the nerve may be compressed. This would affect part of which muscle?
Adductor magnus
Biceps femoris
Gluteus maximus
Semimembranosus
Semitendinosus
The correct answer is: Biceps femoris
To answer this question, you need to determine the innervations of all the muscles listed. Biceps femoris, semimembranosus, and semitendinosus are all part of the hamstrings compartment, which is innervated by the tibial nerve. But, the short head of biceps femoris is the one part of the hamstring compartment that is innervated by the common fibular nerve instead of the tibial nerve. So biceps femoris is the answer you're looking for!

Adductor magnus is innervated by the obturator nerve--it's in the medial compartment of the leg. Gluteus maximus is innervated by the inferior gluteal nerve.
An elderly patient complains of difficulty in walking up stairs. Tests by her doctor reveal weakness in extension at her hip, but no change in hip flexion, or flexion or extension of the knee. Based upon these results, what muscle is most likely not functioning properly.
Adductor magnus
Gluteus maximus
Gluteus medius
Iliopsoas
Semitendinosus
The correct answer is: Gluteus maximus
Gluteus maximus is the most important muscle for powerfully extending the thigh. This is the muscle that is used for forceful extension at the hip joint, which is what you need to go up the stairs or to jump powerfully. That's why gluteus maximus is the answer. Semitendinosus is the other muscle mentioned which extends the thigh--it's one of the hamstrings muscles in the posterior compartment of the thigh. However, semitendinosus flexes the leg, and there is no weakness with this motion. Also, you should remember that gluteus maximus is the key muscle for very forceful extension--not semitendinosus.

Adductor magnus adducts and medially rotates the thigh, while gluteus medius abducts and medially rotates the thigh. Iliopsoas is a hip flexor. There is no weakness in any of these movements, so you should know that these muscles are not injured.
Weakness in climbing stairs or jumping would indicate a lesion of which nerve?
Tibial
Superior gluteal
Inferior gluteal
Obturator
Femoral
The correct answer is: Inferior gluteal nerve
The first thing that you want to do with this question is determine which muscle was impaired. Since the question specifies that there is a weakness when climbing steps or jumping, you should know that there is a problem with powerfully extending the hip. And that's what gluteus maximus does. The nerve to gluteus maximus, the inferior gluteal nerve, must be the nerve that was injured. The tibial nerve innervates the hamstring compartment. Although the hamstrings are involved in extending the hip, they are not the most important muscles for these powerful motions. Gluteus maximus and the inferior gluteal nerve enable the type of powerful extension used to climb steps or jump

The superior gluteal nerve innervates gluteus minimus and medius. These muscles are important for stabilizing the pelvis on the supported side of the hip when the opposite leg is lifted. If the superior gluteal nerve is damaged on the supported side of the hip, the unsupported side of the hip will drop. This is called Trendelenburg gait. The obturator nerve innervates the medial, adductor compartment of the hip. The femoral nerve innervates the anterior compartment of the hip, including the quadriceps. This nerve allows for extension at the knee.
During surgical repair of a popliteal artery aneurism, ligation of the femoral artery at mid-thigh would not interrupt supply to the hamstring muscles because the
Genicular anastomosis ensures blood supply to the posterior thigh
Cruciate anastomosis ensures blood supply to the posterior thigh
Perforating branches of the deep femoral artery supply the posterior thigh
Obturator artery supplies the posterior thigh
Anterior and posterior femoral circumflex arteries anastomose with the inferior gluteal artery
The correct answer is: Perforating branches of the deep femoral artery supply the posterior thigh
The deep femoral artery supplies the posterior compartment of the thigh with three to four perforating arteries. These arteries pierce adductor magnus and supply blood to the hamstrings--biceps femoris, semitendinosus, and semimembranosus. Genicular anastomosis supply the knee joint, not the posterior thigh. The cruciate anastomosis supply blood to the area around the head of the femur. The medial and lateral circumflex arteries and the inferior gluteal artery also supply the hip joint. The obturator artery supplies the medial hip and thigh.
The medial thigh muscles rotate the femur medially, counterbalanced by muscles of the _____________ thigh, including the ___________ muscle, which rotates the femur laterally.
Lateral; tensor fasciae latae
Anterior; rectus femoris
Lateral; piriformis
Posterior; biceps femoris
Posterior; quadriceps femoris
The correct answer is: Lateral; piriformis
A good way to answer this question is to look for a muscle that is a lateral rotator of the thigh and then make sure that the compartment that it is listed with is correct. Piriformis laterally rotates the thigh; it is a member of the lateral compartment which includes other muscles that laterally rotate the thigh, like obturator internus and the superior and inferior gemellus muscles. So, this is the best answer.

Tensor fasciae latae is in the lateral compartment, but it's a medial rotator of the thigh, so this is not a correct answer. Rectus femoris is one of the quadriceps muscles, found in the anterior compartment of the thigh. This compartment extends the knee and flexes the hip, but these muscles do not rotate the thigh. Biceps femoris is a muscle in the posterior compartment--it's a hamstring muscle that extends the hip and flexes the knee.
Following a major operation, a patient was placed on a course of antibiotics which were delivered via intramuscular injection to the buttocks. After one injection in the right buttock, the patient complained of more pain than usual in the region of the injection. Later, as he was walking in the hall, the nurse noticed that he had a limp that had not been present before--his left hip dropped every time he lifted his left foot off the floor. Which nerve had been injured by the injection?
Femoral
Inferior gluteal
Obturator
Sciatic
Superior gluteal
The correct answer is: Superior gluteal
The superior gluteal nerve supplies gluteus minimus and medius--two muscles that are important abductors of the hip. These muscles stabilize the pelvis when walking. They work in such a way that when you are standing on your right limb only, the muscles on the right side are holding the left side of the pelvis level. When you are standing on your left leg only, the muscles of the left side are supporting the right side of the pelvis. This patient has injured his right superior gluteal nerve. He is exhibiting a characteristic hip drop on the uninjured side while standing on the injured side--a positive Trendelenburg sign. The superior gluteal nerve was evidently damaged during the injection. This is why it's important to place injections in the upper, outer quadrant of the buttock--that's the quadrant of the buttock that has fewer nerves that might be damaged by a needle. Nerves that could be injured from careless injections include the sciatic nerve, posterior femoral cutaneous nerve, inferior gluteal nerve, and superior gluteal nerve.

An injury to the femoral nerve would weaken the quadriceps--the patient would not be able to extend his leg. An injury to the obturator nerve would denervate the adductors and impair adduction of the hip. Damage to the sciatic nerve would paralyze the hamstrings and all the muscles in the leg and foot. Finally, injuring the inferior gluteal nerve would paralyze gluteus maximus and impair extension of the hip.
The hamstrings muscles are supplied by branches of which artery?
Deep femoral
Fibular
Obturator
Pudendal
Superficial femoral
The correct answer is: Deep femoral
The deep femoral artery supplies the posterior compartment of the thigh with three to four perforating arteries. These arteries pierce adductor magnus and supply blood to the hamstrings--biceps femoris, semitendinosus, and semimembranosus. The fibular artery is a branch of the posterior tibial artery which supplies the muscles and fascia of the lateral leg and ankle. The obturator artery comes from the anterior division of the internal iliac artery; it delivers blood to the medial thigh and hip. The internal pudendal artery is the major source of blood to the perineum. The superficial femoral artery is the name used clinically for the femoral artery after the deep femoral is given off. Therefore, the short segment of femoral artery proximal to the deep/superficial division is sometimes called the common femoral.
Of the branches of the internal iliac artery, the one exiting from the greater sciatic foramen superior to the piriformis muscle is the:
Iliolumbar artery
Internal pudendal artery
Lateral sacral artery
Superior gluteal artery
The correct answer is: Superior gluteal artery
Piriformis is the key to the posterior thigh, and there are many important nerves and arteries that exit the greater sciatic foramen and enter the posterior thigh either above or below piriformis. The superior gluteal artery and nerve are 2 structures that travel through the greater sciatic foramen superior to piriformis. There are many structures that travel through the greater sciatic foramen inferior to piriformis and lie in the posterior thigh. From medial to lateral, these structures are: pudendal nerve, nerve to obturator internus, nerve to quadratus femoris, inferior gluteal artery, posterior femoral cutaneous nerve, inferior gluteal nerve, and sciatic nerve. See Netter plates 502 and 503 for a picture. You should know what nerves and vessels are superior and inferior to piriformis!

None of the other arteries mentioned travel superior to piriformis to reach the posterior thigh. The iliolumbar artery is a branch of the posterior division of the internal iliac. It travels on the posterior body wall to supply blood to the iliacus, psoas major, and quadratus lumborum. The internal pudendal artery is a branch from the anterior division of the internal iliac artery. It leaves the pelvis with the inferior gluteal artery, traveling inferior to piriformis. It enters the perineum through the lesser sciatic foramen to supply muscles, skin, and the erectile bodies there. Finally, the lateral sacral arteries are branches of the posterior division of the internal iliac artery--they are on the posterior body wall and they supply the sacrum, sacral nerve rootlets, meninges, and the nearby muscles.
Because of its muscle attachments, a fracture to the ischial tuberosity would affect which movement of the lower limb?
Abduction of the thigh
Dorsiflexion of the foot
Extension of the leg
Flexion of the leg
Flexion of the thigh
The correct answer is: Flexion of the leg
The ischial tuberosity is the origin of the hamstrings muscles, so fracturing this bone would disrupt this origin. The hamstrings are important for flexing the leg and extending the thigh; this means that these motions would be impaired following the fracture.

Gluteus minimus and medius are important abductors of the hip. These muscles both take origin from the ilium. So, a fracture to the ilium might impair abduction of the hip, although these muscles are more commonly impaired by damage to the superior gluteal nerve. The tibialis anterior, in the anterior compartment of the leg, is responsible for dorsiflexion of the foot. It originates on the lateral condyle of the tibia, so breaking off the lateral condyle of the tibia might impair dorsiflexion.

The quadriceps, in the anterior compartment of the thigh, are responsible for extending the leg. Rectus femoris takes origin from the anterior inferior iliac spine, while the vastus muscles originate from the body of the femur. Finally, the muscles responsible for flexing the hip are pectineus, iliopsoas, sartorius, and rectus femoris. Pectineus originates from the pecten pubis, iliopsoas originates on the lumbar vertebrae and the iliac fossa, and sartorius originates on the anterior superior iliac spine.
When the distal femur is fractured, the sharp broken ends of the bone often damage the structure that lies closest to the femur in the popliteal fossa, i.e., the:
Descending genicular artery
Greater saphenous vein
Popliteal artery
Popliteal vein
Sciatic nerve
The correct answer is: Popliteal artery
The popliteal fossa contains the popliteal artery, popliteal vein, common fibular nerve, and tibial nerve. The popliteal artery is the deepest structure in the fossa--it lies on the posterior side of the knee joint. So, it is the structure that would be most likely to be lacerated by the broken distal end of the femur. The descending genicular artery is one of the arteries that supplies the knee through the genicular anastomoses. It is found on the medial side of the knee, not in the popliteal fossa. The greater saphenous vein is a superficial vein that drains the lower limb. It is found traveling posterior to the medial epicondyle, but it's not in the popliteal fossa. The sciatic nerve is found in the superior part of the thigh--it quickly splits into the common fibular and tibial nerves which are found in the popliteal fossa.
n a hunting accident, an arrow pierces the mid-calf of a hunter. A major artery is lacerated in the posterior leg, and you notice that the sole of his foot is cold and pale. The dorsum of the foot is warm and normally colored. The artery that seems to be injured is the:
fibular artery
posterior tibial artery
femoral artery
popliteal artery
medial plantar artery
The correct answer is: Posterior tibial artery
The popliteal artery divides into two arteries: the posterior tibial artery and anterior tibial artery. The posterior tibial artery supplies the posterior compartment of the leg with blood. It also passes into the sole of the foot, where it branches to form the medial and lateral plantar arteries. So, the injury to the posterior surface of the leg and the lack of perfusion to the foot all point to an injury to the posterior tibial artery.

The fibular artery is a branch of the posterior tibial artery that gets blood to the fibular compartment of the leg. You can tell that this was not the main artery damaged because this artery is not important for perfusing the foot. The femoral artery is an artery of the anterior thigh--it would not be damaged by a mid-calf injury. The popliteal artery is the artery that branches to form the posterior and anterior tibial arteries. It is clear that this artery is intact because the anterior of the leg and dorsum of the foot--areas supplied by the anterior tibial artery and its continuation, dorsalis pedis--are normal. Finally, the medial plantar artery is found on the plantar side of the foot--it would not even be close to an arrow in the mid-calf.
A patient complains of localized pain in a swollen lower calf and cannot strongly plantar flex his foot. What tendon may have ruptured?
calcaneal
fibularis tertius
flexor digitorum longus
flexor hallucis longus
tibialis anterior
The correct answer is: calcaneal
Gastrocnemius and soleus insert on the calcaneus via the calcaneal tendon, a tendon of the lower calf which is the thickest and strongest tendon of the body. These muscles are important plantarflexors of the foot, so it is likely that the tendon connected to these muscles has been damaged.

The other tendons/muscles listed and their actions are as follows: Fibularis teritus: everts the foot; flexor digitorum longus: flexes toes 2-5; flexor hallucis longus: flexes toe 1; tibialis anterior: inverts and dorsiflexes the foot. Since none of these actions is disrupted, the other tendons are probably fine.
A tennis player feels a "pop" in her calf as she is playing. Her calf becomes tender and there is some slight amount of swelling. Upon examination, her physician informs her that she has ruptured the slender tendon of a small muscle that attaches to the calcaneus. She has apparently ruptured the:
gastrocnemius
plantaris
popliteus
soleus
tibialis posterior
The correct answer is: plantaris
Plantaris is a very small muscle in the posterior compartment of the leg. It has a very long and thin tendon that attaches directly to the calcaneus--it does not insert on the calcaneal tendon along with gastrocnemius and soleus. Plantaris acts to flex the leg and plantarflex the foot, but is is not the major muscle responsible for either of these motions. Like the other muscles of the posterior compartment, it is innervated by the tibial nerve.

Gastrocnemius and soleus are large muscles of the posterior compartment that attach to calcaneus via the thick calcaneal tendon. If that tendon was ruptured, the patient would have impaired plantarflexion and significant pain. Popliteus is a muscle on the posterior knee which allows the knee to rotate medially and unlock to initiate flexion of the knee. Tibialis posterior is also a muscle in the posterior compartment--it plantarflexes and inverts the foot.
A football player tears his calcaneal tendon. You would expect to find weakness in:
dorsifiexion of the foot
eversion of the foot
extension of the knee
inversion of the foot
plantarflexion of the foot
The correct answer is: Plantarflexion of the foot
The calcaneal tendon is the thickest and strongest tendon of the body. It is the tendon that gastrocnemius and soleus--the major plantarflexors of the posterior compartment--use to insert on the dorsum of calcaneus. So, if this tendon was ruptured, gastrocnemius and soleus would not be inserting on the calcaneus, and the football player would be unable to plantarflex his leg.

Dorsiflexion would be impaired if there was damage to the anterior compartment of the leg. Specifically, an injury to tibialis anterior or the deep fibular nerve would lead to a weakness in dorsiflexion. Eversion would be impared if there was an injury to the fibularis muscles--fibularis tertius, fibularis longus, and fibularis brevis--which are all responsible for everting the foot. Inversion would be impaired if there was damage to tibialis anterior and tibialis posterior. These are the two muscles which invert the foot. And remember--extending the knee is done by the quadriceps muscles in the anterior thigh. They are innervated by the femoral nerve, so an injury to this nerve or to the quadriceps muscles themselves might impair extension at the knee.
While on vacation in Florida following her final exams, a scuba diving medical student is accidentally speared by her diving partner. The end of the spear passes medial to lateral, posterior to the medial malleolus. It severs an artery there, which is the:
anterior tibial
dorsalis pedis
fibular
medial plantar
posterior tibial
The correct answer is: Posterior tibial
The posterior tibial artery passes from the posterior compartment of the leg, behind the medial malleolus, before entering the foot. In the foot, it splits to form the medial and lateral plantar arteries. Remember--this is one of the structures that is lined up behind the flexor retinaculum in a very characteristic way. To review, the order here is, from anterior to posterior: tendon of Tibialis posterior, tendon of flexor Digitorum longus, posterior tibial Artery (and vein), tibial Nerve, and tendon of flexor Hallucis longus. So, T, D, A, N, H equals Tom, Dick, ANd Harry.

The anterior tibial artery comes from the anterior compartment of the leg--it changes name to the dorsalis pedis artery at the level of the ankle. If the spear had stabbed the student on the dorsum of the foot or leg, either above or below the ankle, one of these two arteries might have been injured. The fibular artery is a branch of posterior tibial artery that supplies the lateral compartment of the leg--an injury to the lateral leg might damage this artery. The medial plantar artery is a branch of the posterior tibial artery in the foot--if the spear had stabbed her medial foot on the plantar surface, this is the artery that might have been injured.
A patient with painful swelling in the distal calf cannot plantar flex at the ankle with any power. Which tendon was likely ruptured?
Calcaneal
Extensor digitorum longus
Extensor hallucis longus
Plantaris
Tibialis anterior
The correct answer is: Calcaneal
The calcaneal tendon is the thickest and strongest tendon of the body. It is the tendon that gastrocnemius and soleus--the major plantarflexors of the posterior compartment--use to insert on the dorsum of calcaneus. Tearing this tendon is painful, and it means that a patient won't be able to plantarflex the foot. So, the symptoms of this patient fit with a rupture to the calcaneal tendon.

Extensor digitorum longus is important for extending the digits, while extensor hallucis longus allows the great toe to extend. Tibialis anterior is important for dorsiflexion. These three muscles are all in the anterior compartment of the leg, so injuries to these tendons would not fit with pain in the distal calf. Plantaris is a very small muscle in the posterior compartment which flexes the leg and plantar flexes the foot--an injury to plantaris would not cause such severe symptoms.
A 63-year-old female patient says that she has pain in her groin and upper thigh. Upon examination, you palpate a lump located below the inguinal ligament lateral to its attachment to the pubic tubercle. You suspect that this may be a hernia passing through the:
femoral canal
adductor hiatus
obturator canal
deep inguinal ring
superficial inguinal ring
The correct answer is: femoral canal
To understand the anatomy of a femoral hernia, you need to understand the relationships in the femoral triangle. The femoral triangle is an area bounded superiorly by the inguinal ligament, laterally by the sartorius, and medially by the medial edge of adductor longus. There are two main things in the femoral triangle: the femoral nerve and the femoral sheath, which encloses the femoral artery, vein, and canal. The femoral nerve is at the lateral edge of the triangle--it is not enclosed by the femoral sheath. Medial to the femoral nerve, you can see the femoral sheath, enclosing the artery, vein, and canal, in that order from lateral to medial. The femoral canal is the most medial structure in the triangle. It opens up to the abdominal cavity through the femoral ring, so abdominal contents might herniate through that ring and into the femoral canal. Because the hernia is inferior to the inguinal ligament, it has entered the femoral canal.

The other common hernia site listed among the answers are the superficial and deep inguinal rings. However, these hernias would be palpated near the superficial inguinal ring, which is superolateral to the pubic tubercle. The adductor hiatus is a space in the adductor magnus that allows the femoral vessels to pass through and reach the posterior surface of the leg, where they become the popliteal vessels. Finally, the obturator canal is the space in the obturator foramen through which the obturator nerve and vessels travel.
The femoral canal contains the:
Deep inguinal lymph node(s)
Femoral artery
Femoral nerve
Femoral vein
Ilioinguinal nerve
The correct answer is: Deep inguinal lymph node(s)
The femoral canal usually contains a deep inguinal lymph node, called the gland or node of Cloquet. This canal opens to the abdominal cavity through the femoral ring, so it might be the site of a femoral hernia. The femoral artery and vein are enclosed in the femoral sheath with the femoral canal, but they are not in the femoral canal. The femoral nerve is also in the femoral triangle, but it is not enclosed by the femoral sheath. It is the most lateral structure in the femoral triangle. The ilioinguinal nerve is not really part of the anatomy of this region. It innervates muscles of the lower abdominal wall and is associated with the superficial inguinal ring.

Make sure to remember the difference between the femoral canal and femoral sheath!
Which movement would fail in case of paralysis of the quadriceps femoris muscle?
Adduction at the hip
Extension at the hip
Extension at the knee
Flexion at the knee
Medial rotation at the knee
The correct answer is: extension at the knee.
Quadriceps femoris extends the leg (rectus femoris and the vastus muscles) and helps flex the hip (rectus femoris). It is innervated by the femoral nerve, which is made from the contributions of L2, 3, and 4 in the lumbar plexus. If a patient could not adduct at the hip, the adductor muscles might be damaged. These are supplied by the obturator nerve, which also comes from L2, 3, and 4 in the lumbar plexus. If a patient could not extend at the hip or flex the leg, the hamstring muscles might be damaged. These muscles are supplied by the tibial nerve, a branch of the sciatic nerve. If a patient could not rotate the knee medially, popliteus might be injured. This muscle, which is innervated by the tibial nerve, allows the knee to twist and unlock from a fully extended position, as in erect stance.
Which structure does NOT enter or leave the inguinal region by passing deep to the inguinal ligament?
Femoral artery
Femoral nerve
Femoral vein
Psoas major muscle
Round ligament of the uterus
The correct answer is: round ligament of the uterus
The round ligament of the uterus travels through the inguinal canal, superior to the inguinal ligament. It eventually leaves the canal through the superficial ring, enters the perineum, and terminates in the labia majora. So, it is never passing deep to the inguinal ligament. The femoral nerve, artery, and vein all travel deep to the inguinal ligament to enter the femoral triangle. The psoas major also travels deep to the inguinal ligament. It then joins with the iliacus to form the iliopsoas, which inserts on the lesser trochanter of the femur via the iliopsoas tendon.
An elderly woman was found to have a hernial sac in the right femoral triangle and a marked weakness in adduction at the right hip joint. Which nerve was likely compressed by this herniation?
Femoral
Inferior gluteal
Obturator
Pudendal
Sciatic
The correct answer is: obturator
Use the defect in motion, not the location of the injury, to answer questions like this one! It's true that the femoral nerve is in the femoral triangle, so you might want to jump to that answer. However, an injury to the femoral nerve would not cause problems with adduction. Instead, the patient would have problems extending her knee. The nerve which supplies the adductor compartment is the obturator nerve, so that must be the structure that was injured. Although the obturator nerve is not in the femoral triangle, it enters the thigh by passing through the obturator canal and under pectineus, part of the floor of the femoral triangle. So, it could be injured by an obturator hernia passing through the obturator canal.

The inferior gluteal nerve passes through the greater sciatic foramen inferior to the piriformis muscle--it innervates gluteus maximus. The pudendal nerve supplies muscles and skin in the perineum. The sciatic nerve has no direct muscular branches. It does, however have two important branches that supply many muscles: the tibial and common fibular nerves. The tibial nerve supplies the posterior (hamstrings) compartment of the thigh and the posterior compartment of the leg. Branches of the common fibular nerve supply the lateral and anterior compartments of the leg. There would be a major motor deficit if the sciatic nerve was injured.
When walking, the action of the iliopsoas muscle results in what motion at the hip joint?
Abduction
Adduction
Extension
Flexion
Medial rotation
The correct answer is: flexion
The iliopsoas is a combination of the iliacus muscle and the psoas major which inserts on the lesser trochanter of the femur. It is the most powerful hip flexor. Other hip flexors include sartorius, rectus femoris, and pectineus. These muscles are in the anterior compartment of the thigh, with the exception of pectineus (medial compartment). The abductor muscles are in the lateral compartment of the thigh. They include gluteus medius and minimus, tensor fasciae latae, piriformis, and obturator internus. Adductor longus, brevis, and magnus are muscles in the medial compartment which adduct the hip. The major hip extenders are the hamstrings--semimembranosus, semitendinosus, and biceps femoris. They are in the posterior compartment. Gluteus maximus, in the lateral compartment, is also an important muscle for powerfully extending the hip. Finally, the adductors and gluteus medius, minimus, and tensor fasciae latae are the medial rotators of the thigh.
The pulse of the femoral artery is best felt at which superficial reference point?
Anterior to the ankle joint
Femoral triangle
Mid-thigh
Popliteal fossa
Right lateral portion of the hypogastrium
The correct answer is: femoral triangle
The femoral artery passes through the femoral triangle. It lies between the femoral nerve on its lateral side and the femoral vein on its medial side. This is where you would take the femoral pulse. Anterior to the ankle joint, you can feel for the pulse of the dorsalis pedis artery. In the popliteal fossa, you can feel for the pulse of the popliteal artery. (Although, this can be somewhat difficult since the popliteal artery is deep to the popliteal nerve and vein.)
If the femoral artery is occluded at the beginning of the adductor canal, which artery could help provide viability to the leg through collateral circulation?
Descending branch of the lateral circumflex femoral
Descending genicular
First perforating branch of the deep femoral
Medial circumflex femoral
Obturator
The correct answer is: descending branch of the lateral circumflex femoral
The descending branch of the lateral circumflex femoral artery anastamoses with both the descending genicular branch of the femoral artery as well as the lateral superior genicular branch of the popliteal artery. These connections provide collateral circulation to the knee and leg. The descending genicular artery branches from the femoral artery just superior to the adductor hiatus. If the femoral artery was occluded, this artery would not receive blood flow. The perforating branches of the deep femoral artery provide blood to the posterior thigh. The medial circumflex femoral artery is a branch of the femoral that supplies blood to the medial thigh and hip. The obturator artery is a branch of the anterior division of the internal iliac which supplies the medial thigh and hip. None of these arteries would be involved in any anastomoses with the leg.
At which site could one expect to enter the femoral vein with a simple percutaneous (through the skin) introduction of an instrument?
Above the middle of the inguinal ligament
Lateral to the femoral arterial pulse
Lateral to the pubic tubercle
Medial to the femoral arterial pulse
Medial to the pubic tubercle
The correct answer is: medial to the femoral arterial pulse
From lateral to medial, these are the structures in the femoral triangle: femoral nerve, artery, vein, and canal. The artery, vein, and canal are in the femoral sheath, but the nerve is not! So, the vein would be found medial to the arterial pulse. Although the femoral triangle is lateral to the pubic tubercle, that answer is too general. If you were entering the femoral vein, you would find the femoral pulse, use that as a landmark, then insert the instrument medial to that pulse.
What anterior thigh muscle must be retracted to expose the adductor canal and its contents?
Adductor magnus
Gracilis
Rectus femoris
Sartorius
Vastus intermedius
The correct answer is: sartorius
Sartorius is the roof of the adductor canal. If it was retracted, the contents of the adductor canal would be exposed. The structures contained in the adductor canal are: the femoral artery, femoral vein, saphenous nerve, and nerve to vastus medialis. Adductor longus and magnus make the posterior boundary of the canal, and vastus medialis is the lateral boundary of the canal.
A serious complication of fractures of the femoral neck is avascular necrosis of the femoral head. This usually results from rupture of which artery?
Acetabular branch of obturator
Deep circumflex iliac
Descending branch of lateral circumflex femoral
Medial circumflex femoral
Second perforating branch of lateral circumflex
The correct answer is: medial circumflex femoral
It is important to remember that the medial circumflex femoral artery supplies blood to the femoral neck. During fractures of the femoral neck, this artery may be ruptured, and the femoral neck will be deprived of blood. The acetabular branch of the obturator artery provides blood to the acetabulum of the hip bone, which articulates with the head of the femur to form the hip joint. The deep circumflex iliac artery is a branch of the external iliac artery that runs on the deep aspect of the anterior abdominal wall, supplying that region. The decending branch of the lateral circumflex femoral supplies the lateral thigh and hip. Finally, the perforating branches of the deep femoral artery all go to supply the posterior thigh.
A ruptured aneurysm in the most proximal portion of the deep femoral artery would result in a hematoma located initially in the:
Adductor canal.
Femoral canal.
Femoral triangle.
Inguinal canal.
Popliteal fossa.
The correct answer is: femoral triangle
The deep femoral artery branches from the femoral artery in the femoral triangle. So, if the aneurysm ruptured in the most proximal portion of the deep femoral artery, the hematoma would be in the femoral triangle. The femoral artery travels in the adductor canal, but this is after the deep femoral artery has already been given off. Remember--the femoral artery is in the femoral sheath, but not in the femoral canal! The femoral canal is medial to the femoral artery, and it contains a deep inguinal lymph node.
Following a penetrating injury to the left femoral triangle, a patient related that walking was virtually impossible because at every step the left knee collapsed into flexion. This history suggests paralysis of which muscle?
Adductor magnus.
Biceps femoris.
Gluteus maximus.
Quadriceps femoris.
Sartorius.
The correct answer is: Quadriceps femoris
If a knee is continually collapsing into flexion, it means that there is something wrong with the knee extenders--the quadriceps muscles. This makes sense given the clinical history--the injury to the left femoral triangle probably damaged the femoral nerve which innervates the quads. Sartorius is also a flexor of the hip and knee which is innervated by the femoral nerve. However, it is a much weaker muscle and damage to this muscle would not be as debilitating as damage to the quadriceps. Adductor magnus is innervated by the obturator nerve, except for the part inserting on adductor tubercle, which is innervated by tibial nerve. An injury to this muscle or nerve would result in impaired adduction of the thigh. Biceps femoris is a hamstring muscle of the posterior compartment which extends the hip and flexes the knee. Its long head is innervated by the tibial nerve and its short head is innervated by the common fibular nerve. Gluteus maximus is the muscle for powerful extension of the hip--it is innervated by the inferior gluteal nerve.
A patient with a tuberculous abscess (localized collection of pus) on the iliopsoas muscle in the femoral triangle presented impaired flexion of the thigh and extension of the leg. Which of the following nerves was likely involved?
femoral
inferior gluteal
obturator
sciatic
superior gluteal
The correct answer is: femoral
The quadriceps muscles, innervated by the femoral nerve, are the most important leg extensors. Rectus femoris, a component of the quads, is also a hip flexor. So, the patient's deficits, as well as the clinical history of an abscess in the femoral triangle, are consistent with an injury to the femoral nerve. The inferior gluteal nerve innervates gluteus maximus. If this nerve or muscle was injured, the patient would have trouble extending his leg powerfully. The obturator nerve innervates the adductor compartment; a defect with this nerve would result in impaired adduction. The sciatic nerve does not innervate any muscles itself. However, it has two branches, the common fibular and tibial nerves, which innervate the posterior (hamstring) compartment of the thigh and all of the muscles of the leg and foot. Clearly, an injury to the sciatic nerve would lead to a huge motor impairment. Finally, the superior gluteal nerve innervates gluteus medius and minimus and tensor fasciae latae. If this nerve is injured, a patient will exhibit Trendelenburg's sign, which means that when the patient stands on the injured leg only, the pelvis will drop on the unsupported side. This indicates that the gluteus medius and minimus on the supported side are not functional.
An obturator hernia that compresses the obturator nerve in the obturator canal may affect the function of all of the following muscles EXCEPT:
Adductor brevis
Adductor longus
Gracilis
Obturator externus
Pectineus
Pectineus is a muscle of the medial compartment of the thigh. Most of the muscles in this compartment are adductors and medial rotators, innervated by the obturator nerve. However, pectineus is the exception of the medial compartment--it's innervated by the femoral nerve, and it's a hip flexor. So, damage to the obturator nerve would not affect pectineus!

Adductor brevis, adductor longus, and gracilis are muscles of the medial compartment which adduct and medially rotate the thigh--they are all innervated by the obturator nerve. Obturator externus is also innervated by the obturator nerve--it is a lateral rotator found in the medial compartment of the thigh.
Childhood immunizations are sometimes given via intramuscular injections into the quadriceps muscles of the anterior thigh. At the mid-thigh level, a needle passing into the space deep to the sartorius muscle might pierce the femoral vessels as they lie in the:
Adductor canal
Adductor hiatus
Adductor triangle
Femoral canal
Femoral ring
The correct answer is: Adductor canal
Sartorius is the roof of the adductor canal. The femoral artery, femoral vein, saphenous nerve, and nerve to vastus medialis are all found in the adductor canal, deep to sartorius. So, if a needle pierced sartorius to enter the femoral vessels, the needle would be entering the adductor canal. The adductor hiatus is a space in the distal thigh, between the two insertions of adductor magnus. This is the place where the femoral vessels leave the adductor canal, travel to the posterior thigh, and become the popliteal vessels. There is no such thing as an adductor triangle.

The femoral canal is one of the structures in the femoral sheath--it usually contains a deep inguinal lymph node, called the gland of Cloquet. This canal opens to the abdominal cavity through the femoral ring, so a femoral hernia could pass into the femoral canal through the femoral ring. Remember: the femoral vein and artery are contained in the femoral sheath, but NOT in the femoral canal!
A female patient complains of pain in her groin region that increases with coughing. An MRI reveals a loop of ilium passing inferiorly, posterior to the superior pubic ramus. What type of hernia is this?
obturator
femoral
direct inguinal
indirect inguinal
lumbar
The correct answer is: Obturator
An obturator hernia is a protrusion of a loop of bowel through the obturator canal. This is exactly what is being described here, with a loop of ilium passing inferiorly, posterior to the superior pubic ramus. A femoral hernia is a protrusion of abdominal viscera through the femoral ring into the femoral canal. It often appears as a tender mass in the femoral triangle, inferolateral to the pubic tubercle.

A direct inguinal hernia leaves the abdominal cavity medial to the inferior epigastric artery and protrudes through the posterior wall of the inguinal canal. It will be felt superior to the pubic tubercle, and it usually does not enter the scrotum. An indirect inguinal hernia leaves the abdominal cavity lateral to the inferior epigastric vessels and enters the deep inguinal ring. It will usually traverse the entire length of the inguinal canal and enter the scrotum. It should also be palpated superior to the pubic tubercle.

Lumbar hernias are hernias through the lumbar triangle, which is bordered medially by the latissimus dorsi, laterally by the external abdominal oblique, and inferiorly by the iliac crest.
During a surgical procedure in which the lower abdominal wall has been opened, the retractor blades damaged a nerve lying between the iliacus and psoas major muscles. The patient has weakened hip flexion and an inability to extend the knee, as well as analgesia on the anterior thigh and medial leg. What nerve has been compressed?
obturator
femoral
sciatic
tibial
common fibular
The correct answer is: Femoral
The femoral nerve lies between the iliacus and psoas major muscles. So, if a surgeon pulls these muscles apart with a retractor, the femoral nerve can be easily injured. This patient's symptoms match with an injury to the femoral nerve. Because the quadriceps are denervated, she cannot extend her knee. The femoral nerve also innervates powerful hip flexors like pectineus, rectus femoris, part of iliopsoas, and sartorius. With these muscles denervated, she can't flex her hip well. Finally, consider cutaneous innervation--the anterior cutaneous branches of the femoral nerve supply cutaneous sensation to the anterior thigh, and the saphenous nerve (which is a branch of the femoral nerve) supplies cutaneous sensation to the medial leg. This is exactly where she is experiencing analgesia. So, everything here points to a femoral nerve injury.

The obturator nerve innervates the medial compartment of the thigh--an injury to this nerve would cause weakness in adduction and medial rotation. The sciatic nerve is found in the posterior thigh--it does not innervate any muscles directly, but its branches are the common fibular and tibial nerve. If the sciatic was damaged and these two branches were lost, there would be severe motor and sensory deficits throughout the posterior thigh and the leg and foot. The tibial nerve innervates the hamstrings, the posterior compartment of the leg, and the plantar foot. Damaging the tibial nerve would impair flexion at the knee and plantarflexion of the foot. The common fibular nerve has branches that innervate the lateral and anterior compartment of the leg and provide sensory innervation to the lower anterior leg and dorsum of the foot. An injury to this nerve would impair dorsiflexion of the foot, causing foot drop, and cause numbness on the dorsal foot.
Which of the following is NOT located within the adductor canal?
Saphenous nerve
Femoral artery
Nerve to vastus medialis
Femoral vein
Deep femoral artery
The correct answer is: Deep femoral artery
The adductor canal contains: the femoral artery, femoral vein, saphenous nerve, and nerve to vastus medialis. Adductor longus and magnus make the posterior boundary of the canal, and vastus medialis is the lateral boundary of the canal. The roof of the canal is formed by sartorius.

The deep femoral artery branches from the femoral artery in the femoral triangle; it then travels deep in the thigh, posterior to adductor longus. So, it is deep to the canal. The deep femoral artery supplies blood to the posterior compartment of the thigh through perforating branches.
Inability to extend the knee and loss of cutaneous sensation over the anterior surface of the thigh would indicate a lesion or compression of the:
Superior gluteal nerve
Lateral femoral cutaneous nerve
Sciatic nerve
Femoral nerve
Obturator nerve
The correct answer is: Femoral nerve
The femoral nerve innervates the quadriceps, which are the muscles that extend the knee. It also provides cutaneous innervation to the anterior thigh and medial leg. This means that the symptoms here fit with an injury to the femoral nerve. The superior gluteal nerve supplies gluteus minimus and medius--two muscles that are important abductors of the hip. These muscles stabilize the pelvis when walking.

The lateral femoral cutaneous nerve is a sensory nerve only--if this nerve was injured, the patient would have a loss of sensation over the lateral thigh but no motor weakness. The sciatic nerve is found in the posterior thigh--it does not innervate any muscles directly, but its branches are the common fibular and tibial nerve. If the sciatic was damaged and these two branches were lost, there would be severe motor and sensory deficits throughout the posterior thigh and the leg and foot. The obturator nerve innervates the medial compartment of the thigh--an injury to this nerve would cause weakness in adduction and medial rotation.
Which statement is true?
The femoral artery lies medial to the femoral vein
The femoral vein lies medial to the femoral artery
The external iliac veins join to form the inferior vena cava
The inferior vena cava can not be imaged radiographically
A and C
The correct answer is: The femoral vein lies medial to the femoral artery
From lateral to medial, the structures in the femoral triangle are organized as follows: femoral nerve, femoral artery, femoral vein, and the femoral canal (with lymph node). So, the vein is medial to the artery. Remember--only the artery, vein, and canal are in the femoral sheath--the femoral nerve is not contained in the femoral sheath!

The external iliac veins do not join to form the inferior vena cava--the common iliac veins are the vessels that form the inferior vena cava. The IVC can be imaged radiographically--remember, you see the inferior vena cava in the axial CT scans of the thorax and abdomen.
Blood flow around an occlusion of the femoral (superficial femoral) artery at the apex of the femoral triangle could be provided by:
Anterior tibial recurrent
Descending branch of the lateral circumflex femoral
Descending genicular
Medial circumflex femoral
Perforating branches of the deep femoral
The correct answer is: Descending branch of the lateral circumflex femoral
The descending branch of the lateral circumflex femoral artery anastamoses with both the descending genicular branch of the femoral artery as well as the lateral superior genicular branch of the popliteal artery. These connections provide collateral circulation to the knee and leg. This could help blood flow around an occlusion to the proximal femoral artery. The anterior tibial recurrent artery is an artery that supplies the knee--it is a branch of the anterior tibial artery and it is not an artery that could help blood flow around an obstruction in the femoral artery. The descending genicular artery branches from the femoral artery just superior to the adductor hiatus. If the femoral artery was occluded, this artery would not recieve blood flow.

The medial circumflex femoral artery is a branch of the deep femoral that supplies blood to the medial thigh and hip. The perforating branches of the deep femoral artery provide blood to the posterior thigh. None of these arteries would allow blood to flow around an occlusion in the femoral artery--in fact, these arteries would not receive blood if the femoral artery was blocked.
After suffering a penetrating injury in the left femoral triangle, a patient was unable to walk normally because her left knee collapsed into flexion upon weight bearing. The nerve to which muscle was damaged?
Adductor magnus
Biceps femoris
Gluteus maximus
Gluteus medius
Quadriceps femoris
The correct answer is: Quadriceps femoris
This patient cannot extend her knee, so her quadriceps muscle must be injured. Remember, the quadriceps is the major muscle responsible for extending the knee. Also, note that the quadriceps is innervated by the femoral nerve, which is the nerve that would be easily damaged by a penetrating injury to the femoral triangle.

Adductor magnus adducts the thigh; it is innervated by the obturator nerve. Although the obturator nerve might be damaged by an injury to the femoral triangle, the symptoms do not fit with a deficit of this nerve. Biceps femoris is innervated by the tibial nerve and common fibular nerve; gluteus maximus is innervated by the inferior gluteal nerve; gluteus medius is innervated by the superior gluteal nerve. All of these nerves are found in the posterior thigh, and they would not be related to an injury in the femoral triangle.
"Scissor gait" is a condition in which one limb crosses in front of the other during stepping as a result of powerful hip adduction caused by continuous, unwanted nerve activity. What is the nerve involved in this condition?
Femoral
Inferior gluteal
Obturator
Tibial
The correct answer is: Obturator
With "scissor gait" there is overactive adduction of the thigh. So, you need to identify which nerve innervates the adductor compartment. And that nerve is the obturator nerve. The obturator nerve innervates the medial compartment of the thigh, including adductor longus, magnus, and brevis. If the obturator nerve was firing too much, the leg would be constantly adducting, causing the scissor gait.

The femoral nerve innervates the quadriceps muscles, which extend the leg at the knee. The inferior gluteal nerve innervates gluteus maximus, which is important for powerful extension of the thigh. The tibial nerve innervates the hamstrings, which flex the knee and extend the thigh. The tibial nerve also innervates the muscles of the posterior compartment of the leg, which plantarflex the foot.
In the middle third of the thigh, the superficial and deep femoral arteries are separated by:
Adductor longus
Adductor magnus
Gracilis
Pectineus
Vastus medialis
The correct answer is: Adductor longus
The femoral artery is superficial to adductor longus, while the deep femoral artery is deep to adductor longus. Since adductor longus is the posterior border of the adductor canal, this explains why the femoral artery is considered part of the adductor canal, while the deep femoral artery is not in the adductor canal. The deep femoral artery runs deep to the adductor canal, but immediately superficial to adductor brevis and magnus.
Fracture of the femoral neck may lead to avascular necrosis of the femoral head due to the interruption of which artery?
First perforating branch of the deep femoral
Inferior epigastric
Internal pudendal
Lateral circumflex femoral
Medial circumflex femoral
The correct answer is: Medial circumflex femoral
It is important to remember that the medial circumflex femoral artery supplies blood to the femoral neck. During fractures of the femoral neck, this artery may be ruptured, and the femoral neck will be deprived of blood. The perforating branches of the deep femoral artery supply the posterior compartment of the thigh, including the hamstrings. The inferior epigastric artery is a branch of the external iliac artery that supplies the lower abdominal wall. The internal pudendal artery is the major source of blood to the perineum. The lateral circumflex femoral artery supplies the lateral thigh and hip. Although it contributes to the circulation around the hip, the primary supply to the head of the femur usually comes from the medial femoral circumflex.
Following surgical opening of the adductor canal, a patient experienced a loss of cutaneous sensation of the medial side of the leg. Which nerve was cut?
Ilioinguinal
Femoral
Obturator
Medial sural cutaneous
Saphenous
The correct answer is: Saphenous
The saphenous nerve is a branch of the femoral nerve. This nerve, and the nerve to vastus medialis, are the only nerves found in the adductor canal. So, it makes sense that one of these structures was injured. The saphenous nerve is a sensory nerve only; it supplies the skin on the medial side of the leg. The patient's symptoms, as well as the type of procedure done, both point toward an injury to the saphenous nerve.

The femoral nerve does not travel in the adductor canal -- it ends by branching superior to the adductor canal, in the femoral triangle. If this nerve was injured, the quadriceps would be impaired, and the patient would have difficulty extending at the knee. The ilioinguinal nerve is a branch of the lumbar plexus which innervates muscles of the lower abdominal wall. The medial sural cutaneous nerve is a branch of the tibial nerve, responsible for providing cutaneous sensation to the upper posterior calf. The obturator nerve innervates the medial, adductor compartment of the thigh. Neither the symptoms nor the location of injury suggest that any of these other nerves were damaged.
A needle biopsy of the sural nerve resulted in the formation of a hematoma. Which of the following veins closely adjacent to the nerve was accidently injured?
accessory saphenous
femoral
greater saphenous
lesser saphenous
superficial external pudendal
The correct answer is: lesser saphenous
The sural nerve runs directly next to the lesser saphenous vein, so that is the source of the hematoma. The accessory saphenous vein is a small branch of the greater saphenous vein that lies on the medial side of the thigh. The femoral vein is a large, deep vein that drains the upper thigh. The greater saphenous vein is a superficial vein that lies on the medial side of the leg. It travels anterior to the medial malleolus and posterior to the medial condyle of the femur to lay on the anteromedial side of the thigh. The superficial external pudendal vein drains the external genitalia.
Competency of venous valves in a patient with severe crural varicose veins was tested as follows: The superficial veins were emptied by elevating the limb and then they were compressed with a band just below the saphenous opening in the proximal thigh. When the patient quickly stood up, filling of the superficial veins took more than 30 seconds. The delayed filling of the superficial veins demonstrates that the valves of the:
deep (central) veins are competent
deep (central) veins are incompetent
perforating veins are competent
perforating veins are incompetent
superificial veins are competent
The correct answer is: perforating veins are competent
The job of the perforating veins is to shunt blood from the superficial veins to the deeper veins. If these valves are incompetent, there will be no way to stop blood from flowing into the cutaneous veins, and they will immediately fill with blood. If the valves are competent, you should see a delayed filling time, indicating that blood can't just simply flow backwards through defective perforating veins. Since the patient has a delayed filling time, that is an indication that her valves are competent.

Although the obvious abnormality with varicose veins involves the superficial veins, the underlying problem tends to involve the valves of the perforating veins.
The lesser saphenous vein was mobilized for grafting. Following the surgery, the patient complained of a loss of cutaneous sensation at the distal posterolateral side of the leg, lateral side of the foot and small toe. What nerve was damaged during the operation?
common fibular (peroneal)
saphenous
superficial fibular (peroneal)
sural
tibial
The correct answer is: sural
The sural nerve runs with the lesser saphenous vein, posterior to the lateral malleolus and up the back of the leg. It provides cutaneous innervation to the skin of the posterior surface of the lower leg and the skin of the lateral side of the foot. So, both the case history and the symptoms point to a sural nerve injury. The common fibular nerve gives rise to the lateral sural cutaneous nerve, superficial fibular nerve, and deep fibular nerve. If this nerve was injured, the patient would have a sensory defect on the distal third of the leg and the dorsum of the foot, stemming from the loss of the superficial fibular nerve. Damage to the common fibular nerve would also result in significant motor loss--no innervation to the anterior or lateral compartments of the leg would cause foot drop. The saphenous nerve travels with the great saphenous vein and provides cutaneous innervation to the medial leg and foot. Finally, the tibial nerve is most important as a motor nerve--it innervates the posterior compartments of the leg and the plantar foot. It also provides the sensory innervation to the plantar surface of the foot.
Varicosities in the subcutaneous veins of the medial thigh were observed at physical examination. The vein involved was most likely the:
Femoral.
Greater saphenous.
Lesser saphenous.
Popliteal.
Pudendal.
The correct answer is: greater saphenous
Remember: subcutaneous venous varicocities occur in the superficial veins due to valve failure in the perforating veins! Varicose veins can be caused by activities that allow blood to pool in the legs, like excessive standing. Since the greater saphenous vein is a superficial vein on the anteromedial side of the leg and thigh, it's our winner! The femoral vein is a deep vein that drains much of the thigh. It is not a vein that would become varicose. The lesser saphenous vein is a superficial vein that runs up the middle of the posterior leg. It could be the cause of varicose veins on the posterior lower leg. The popliteal vein is a deep vein behind the knee which receives the lesser saphenous vein and eventually becomes the femoral vein. It is deep in the leg, so it's not the type of vein that would become varicose. Finally, the pudendal veins drain the external genitalia--they are not really relevant here.
The distal part of the lesser saphenous vein was mobilized for grafting. Following the operation, the patient complained of numbness (loss of cutaneous sensation) on the distal lateral side of the leg and the lateral side of the foot. What nerve was damaged during the operation?
Deep fibular (peroneal)
Posterior femoral cutaneous
Saphenous
Superficial fibular (peroneal)
Sural
The correct answer is: sural
The sural nerve runs with the lesser saphenous vein, posterior to the lateral malleolus and up the back of the leg. It provides cutaneous innervation to the skin of the posterior surface of the lower leg and the skin of the lateral side of the foot. So, both the case history and the symptoms point to a sural nerve injury. The deep fibular nerve is not a major nerve for cutaneous innervation--it innervates the muscles of the anterior compartment of the leg as well as the web of skin between the 1st and 2nd toe. If this nerve was injured, a patient would not have a distinct loss of cutaneous sensation (except in that small web of skin). Instead, the prominent symptom would be "foot drop." The posterior femoral cutaneous nerve provides sensory innervation to the skin of the lower buttock and posterior thigh. The saphenous nerve travels with the great saphenous vein and provides cutaneous innervation to the medial leg and foot. Finally, the superficial fibular nerve provides sensory innervation to the anterior surface of the distal third of the leg and the dorsal surface of the foot.
Subcutaneous venous varicosities were observed in the anteromedial thigh. The vein involved is likely to be the:
Femoral
Greater saphenous
Lesser saphenous
Popliteal
Superficial external pudendal
The correct answer is: greater saphenous
Remember: Subcutaneous venous varicocities happen when valves of the perforating veins fail, causing blood to pool in the superficial veins. Because these superficial veins become distended, their valves fail too, and they become varicose. So, this question is asking you to identify the superficial veins on the anteromedial thigh.

The greater saphenous vein is on the medial side of the leg. It begins as the dorsal venous arch of the foot, and then heads up the leg, anterior to the medial malleolus. At the knee, it goes behind the medial condyle of the femur, and then turns slightly anterior and lateral as it moves up the thigh. It travels throught the saphenous opening and drains into the femoral vein. Since this is a superficial vein on the anteromedial side of the leg, that's the answer!

The femoral vein is a deep vein that drains much of the thigh. It is not a vein that would become varicose. The lesser saphenous vein is a superficial vein that runs up the middle of the back of the leg. It could be the cause of varicose veins on the posterior lower leg. The popliteal vein is a deep vein behind the knee that receives the lesser saphenous vein and eventually becomes the femoral vein. Finally, the superficial external pudendal vein is a superficial branch of the femoral vein that drains the external genitalia.
You are in the clinic when a patient presents with varicose saphenous veins in her lower limb. The attending physician asks where the venous valves are located that are defective and cause this condition. Having just studied a clinical case in that regard, you respond. "The valves in the:
deep veins."
perforating veins."
superficial veins."
venae commitantes."
The correct answer is: perforating veins
Perforating veins are anastamosing channels that shunt blood from the superficial veins to deep veins. These perforating veins have valves to ensure that blood only flows in the superficial to deep direction. If these valves fail, blood will back up in the superficial veins, causing valves to fail in the superficial veins, too. Then, the veins become varicose. Venae commitantes are special veins, usually paired, that accompany deep arteries in the limbs. They surround arteries in an irregular branching network that creates a vascular sheet around the arteries.

The veins in the lower limb that are most likely to become varicose are the greater and lesser saphenous veins and their tributaries.
A 50-year-old female patient has large varicose veins located primarily on the posterior aspect of her calf. These veins are most likely direct tributaries to the:
great saphenous vein
sural vein
small saphenous vein
femoral vein
dorsal venous arch
The correct answer is: small saphenous vein
When the valves of the perforating veins fail, subcutaneous varicose veins develop. Perforating veins direct venous blood from superficial veins to deeper veins; they have valves that are designed to prevent blood from flowing backwards into the superficial veins. If these valves fail, all of the blood from the superficial veins and some blood from the deep veins is forced into the superficial veins. Because these superficial veins become distended, their valves fail too, and they become varicose. So, this question is asking you to identify the superficial veins on the posterior aspect of the calf.

The lesser or small saphenous vein is a superficial vein that begins as the dorsal venous arch and runs behind the lateral malleolus, up the middle of the back of the leg, and terminates as the popliteal vein in the popliteal fossa. It would be the cause of varicose veins on the posterior lower leg, so it's the answer. The greater saphenous vein is on the medial side of the leg. It also begins as the dorsal venous arch of the foot, and then heads up the leg, anterior to the medial malleolus. At the knee, it goes behind the medial condyle of the femur, and then turns slightly anterior and lateral as it moves up the thigh. It travels through the saphenous opening and drains into the femoral vein.

The femoral vein is a deep vein that drains much of the thigh. It is not a vein that would become varicose. Finally, the dorsal venous arch is located on the dorsum of the foot, and it is where the lesser and greater saphenous veins begin. This is not a common site for varicose veins.
Your patient complains of pain on her calf. Upon inspection, you find tortuous, dilated vessels lying subcutaneously on the posterior aspect of her leg. These vessels are most likely:
Tributaries to the greater saphenous vein
Tributaries to the lesser saphenous vein
Perforating veins
Through the iliotibial tract
Through the popliteal fascia
The correct answer is: Tributaries to the lesser saphenous vein
When the valves of the perforating veins fail, subcutaneous varicose veins develop. Perforating veins direct venous blood from superficial veins to deeper veins; they have valves that are designed to prevent blood from flowing backwards into the superficial veins. If these valves fail, all of the blood from the superficial veins and some blood from the deep veins is forced into the superficial veins. Because these superficial veins become distended, their valves fail too, and they become the varicose veins. Even though the perforating veins are the primary culprits here, this question is not asking about them--it is asking you to identify the superficial veins on the posterior aspect of the calf that have become varicose.

The lesser saphenous vein is a superficial vein that runs down the middle of the back of the leg. It, and its tributaries, would be the cause of varicose veins on the posterior lower leg, so this is the answer. The greater saphenous vein is a superficial vein on the anteromedial side of the leg and thigh--it can also develop varicosities, but it's not in the right area for this question.

The posterior tibial vein and its tributaries are deep veins--these veins don't become varicose. Finally, remember that these varicosities develop in superficial veins, never in superficial arteries!
A cashier develops painful and tortuous varicose veins in her lower limb. Her doctor explains that prolonged standing at her job led to failure of the valves in the:
deep femoral vein
great saphenous vein
lesser saphenous vein
perforating veins
popliteal vein
The correct answer is: Perforating veins
Perforating veins are anastamosing channels that shunt blood from the superficial veins to deep veins. These perforating veins have valves to ensure that blood only flows in the superficial to deep direction. If these valves fail, blood will back up in the superficial veins, causing valves to fail in the superficial veins, too. Then, the superficial veins become varicose. So, the primary cause of the varicose veins is the failure of the valves in the perforating veins.

The greater and lesser saphenous veins are superficial veins which may become varicose once the perforating veins fail, but these are not the veins at the root of the problem. Deep veins, like the deep femoral vein or the popliteal vein, do not become varicose or cause other veins to become varicose--these veins receive blood from the superficial veins.
A saphenous cutdown is a surgical procedure that involves cutting through the skin to locate the greater saphenous vein in order to insert a catheter or cannula. You can find the vein as it passes ___________________ with the saphenous nerve.
Anterior to the medial epicondyle
Anterior to the medial malleolus
Through the saphenous opening
Tributaries to the posterior tibial vein
Subcutaneous branches of the posterior tibial artery
The correct answer is: Anterior to the medial malleolus
At the ankle, the great saphenous vein travels anterior to the medial malleolus, with the saphenous nerve. So, you want to make an incision anterior to the medial malleolus, because this is where you will insert the catheter or cannula. At the knee, the great saphenous vein travels posterior to the medial epicondyle of the femur, and it will eventually travel up the thigh and drain into the femoral vein at the saphenous opening. However, saphenous cutdowns are done anterior to the medial malleolus, not at the saphenous opening or behind the medial epicondyle!
A construction worker, who wears a heavy work belt all day on the job, notices a tingling sensation on the anterolateral surface of his thigh. This is most likely a condition called:
compartment syndrome
cryptorchidism
hydrocoele
meralgia parasthetica
thrombophlebitis
The correct answer is: Meralgia paresthetica
Meralgia paresthetica is a sense of tingling and itching on the lateral side of the thigh, in the area of distribution of the lateral femoral cutaneous nerve. It is a palsy of the lateral cutaneous nerve of the thigh, and it is often caused by wearing something that presses against this nerve, like tight jeans or a work belt. Compartment syndrome relates to a group of symptoms which indicate a nerve, blood vessel or tendon is being constricted due to swelling within a closed anatomic space. Anterior comparment syndrome is commonly seen in the leg, and carpal tunnel syndrome is a compartment syndrome in the wrist. Symptoms of compartment syndrome include numbness, tingling, pain or loss of movement in an extremity. Cryptorchidism is the term for undescended testes. Hydrocoele is the term for the presence of excess fluid in a persistent processus vaginalis. This congenital anomaly may be associated with an indirect inguinal hernia. Thrombophlebitis is the name for inflammation that may develop around a vein once there is a thrombus in that vein.
A patient with a diabetic ulcer in the anterior midline of the ankle region experienced loss of cutaneous sensation on the dorsal surface of the foot. Which nerve was most likely damaged?
Femoral
Lateral sural cutaneous
Saphenous
Superficial fibular
Sural
The correct answer is: superficial fibular
The superficial fibular nerve provides cutaneous innervation to the lower anterior third of the leg and the dorsum of the foot. It reaches the dorsum of the foot by crossing over the anterior midline of the ankle region. Both the area of injury and the subsequent symptoms should point to damage of the superficial fibular nerve. The femoral nerve is mostly important as a motor nerve--it innervates the quads, sartorius, and pectineus. Its anterior femoral cutaneous branches provide sensory innervation to the medial and anterior thigh. Another branch of the femoral nerve, the saphenous nerve, provides sensory innervation to the medial side of the leg and the foot. The lateral sural cutaneous nerve is a branch of the common fibular nerve--it provides sensory innervation to the skin of the lateral side of the leg. Finally, the sural nerve, which runs down the posterior leg with the lesser saphenous vein, provides sensory innervation to the posterior surface of the lower leg and the lateral side of the foot.
While doing a physical examination on a young boy, you noticed an enlarged superficial inguinal lymph node. The patient indicated that he has recently sustained an infected skin wound in the:
anterior abdominal wall above the umbilicus.
anterior chest wall.
skin of the lower limb.
upper back.
The correct answer is: skin of the lower limb
The skin of the lower limb is drained by superficial lymph vessels that send lymph to the superficial inguinal lymph nodes. These important nodes also receive lymph from the anal canal below the level of the pectinate line, the external genitalia, and the lower abdominal wall. The anterior abdominal wall is drained by superficial lymphatic vessels accompanying the subcutaneous veins. If these vessels are superior to the umbilicus, they direct lymph to the axillary lymph nodes, and if they are inferior to the umbilicus, they send lymph to the superficial inguinal lymph nodes. So, a skin infection above the umbilicus would cause an enlargement of the axillary nodes. The anterior chest wall and skin of the upper back also drain to subdivisions of the axillary nodes.
A 'saphenous vein cut-down' is a procedure used to locate the great saphenous vein at the ankle. To find this vein, the skin would be incised:
anterior to the lateral epicondyle
posterior to the medial epicondyle
anterior to the medial malleolus
posterior to the lateral malleolus
over the base of the fifth metatarsal
The correct answer is: anterior to the medial malleolus
At the ankle, the great saphenous vein travels anterior to the medial malleolus. At the knee, it travels posterior to the medial condyle of the femur. However, saphenous cut-downs are done at the ankle, not the knee, so anterior to the medial malleolus is the correct answer. The other important relationship to remember is that the lesser saphenous vein travels posterior to the lateral malleolus.
Following a car accident in which the patient received a deep laceration on the medial side of his right knee, the patient notices numbness along the medial side of his right leg and foot. He has no motor deficit. The nerve which appears to have been injured is the:
femoral nerve
saphenous nerve
sural nerve
superficial fibular nerve
deep fibular nerve
The correct answer is: saphenous
The saphenous nerve travels with the great saphenous vein, running along the medial side of the leg and thigh. It provides cutaneous innervation to the medial leg and foot and does not provide motor innervation to any muscles. So, both the case history and the symptoms point to a saphenous nerve injury. Branches of the femoral nerve provide cutaneous innervation to the skin of the anterior thigh--these nerves would not have been involved with this accident. The sural nerve runs with the lesser saphenous vein, posterior to the lateral malleolus and up the back of the leg. It provides cutaneous innervation to the skin of the posterior surface of the lower leg and the skin of the lateral side of the foot. The superficial fibular nerve provides sensory innervation to the distal third of the leg and the dorsal surface of the foot. Finally the deep fibular nerve is not a major nerve for cutaneous innervation--it innervates the muscles of the anterior compartment of the leg as well as the web of skin between the 1st and 2nd toe. If this nerve was injured, a patient would not have a distinct loss of cutaneous sensation (except in that small web of skin). Instead, the prominent symptom would be "foot drop."
Following thyroid surgery, it was noted that a patient frequently aspirated fluid into her lungs. Upon examination it was determined that the area of the piriform recess above the vocal fold of the larynx was numb. What nerve may have been injured?
External branch of the superior pharyngeal
Hypoglossal
Internal branch of the superior laryngeal
Lingual
Recurrent laryngeal
The correct answer is: Internal branch of the superior laryngeal
The internal branch of the superior laryngeal is a sensory nerve that pierces the thyrohyoid membrane along with the superior laryngeal artery. It supplies sensory fibers to the mucous membrane of the larynx, superior to the vocal folds. Since this area lost sensation, it appears that the internal branch of the superior laryngeal nerve must have been injured. The external branch of the superior laryngeal nerve is a motor nerve that innervates the cricothyroid muscle--it does not provide any sensory innervation to the larynx. The recurrent laryngeal nerve ascends from the thorax and provides motor innervation to the upper esophagus, lower pharynx, and all the laryngeal muscles except cricothyroideus.

The hypoglossal nerve supplies motor innervation to the muscles of the tongue. The lingual nerve is a sensory nerve for the anterior 2/3 of the tongue. These nerves are not important for innervating the larynx.
The laryngeal muscle most responsible for stretching (elongating) the vocal ligament is the :
Posterior cricoarytenoid
Lateral cricoarytenoid
Thyroarytenoid
Arytenoid
Cricothyroid
The correct answer is: Cricothyroid
Cricothyroid draws the thyroid cartilage forward, tensing the vocal ligaments. This is the one muscle innervated by the external branch of the superior laryngeal nerve. Posterior cricoarytenoid is a very important muscle; it's the only muscle that abducts the vocal folds. This muscle is innervated by the inferior laryngeal nerve, which is a continuation of the recurrent laryngeal nerve. If this muscle is denervated, the vocal folds may be paralyzed in an adducted position, which would prevent air from entering the trachea. Lateral cricoarytenoid, thyroarytenoid, and arytenoid are all muscles that adduct the vocal folds. They are all innervated by the inferior laryngeal nerve.
The muscle most responsible for the abduction of the vocal folds is the :
Arytenoid
Cricothyroid
Lateral cricoarytenoid
Posterior cricoarytenoid
Thyroarytenoid
The correct answer is: Posterior cricoarytenoid
The posterior cricoarytenoid is the only muscle that abducts the vocal folds. This muscle is innervated by the inferior laryngeal nerve, which is a continuation of the recurrent laryngeal nerve. If this muscle is denervated, the vocal folds may be paralyzed in an adducted position, which would prevent air from entering the trachea. Arytenoid, lateral cricoarytenoid, and thyroarytenoid all adduct the vocal folds. Cricothyroid is the only laryngeal muscle innervated by the external branch of the superior laryngeal. It tenses the vocal ligaments by tipping the thyroid cartilage forward relative to the cricoid cartilage.
The nerve that innervates all but one muscle of larynx is the:
Glossopharyngeal nerve, pharyngeal branch
Inferior laryngeal nerve
Superior laryngeal nerve, external branch
Superior laryngeal nerve, internal branch
Thyrohyoid nerve
The correct answer is: Inferior laryngeal nerve
The inferior laryngeal nerve is the continuation of the recurrent laryngeal nerve, superior to the inferior border of the cricoid cartilage. It innervates the posterior cricoarytenoid muscle, which is responsible for abducting the vocal folds. The inferior laryngeal nerve also innervates all of the other intrinsic muscles of the larynx, with the exception of the cricothyroid muscle. The pharyngeal branch of the glossopharyngeal nerve provides sensory innervation to the pharynx, but not to the larynx. The glossopharyngeal nerve only provides motor innervation to stylopharyngeus. The external branch of the superior laryngeal nerve innervates the cricothyroid muscle, which is responsible for stretching the vocal ligaments. The internal branch of the superior laryngeal nerve is responsible for providing sensory innervation to the mucosa of the larynx, superior to the true vocal fold. The thyrohyoid nerve is a branch of ansa cervicalis-- it innervates the thyrohyoid muscle, which elevates the larynx and depresses the hyoid bone (or helps to fix the hyoid in space, so that other muscles, such as tongue muscles, can work).
The vocalis muscle is most responsible for the fine control of phonation because of its attachment into the:
Arytenoid cartilage
Cricoid cartilage
Thyroid cartilage
Vestibular ligament
Vocal ligament
The correct answer is: Vocal ligament
Vocalis represents the innermost fibers of the thyroarytenoid muscle, from the inner surface of the thyroid cartilage. Because vocalis inserts on the vocal ligament, it can relax very specific segments of the vocal ligament to adjust the pitch. Although there are many muscles that insert on the other pieces of cartilage listed, vocalis is the only one that can provide for such fine control of tone because it is inserting right on the vocal ligament.
The constrictor muscles of the pharynx receive their motor nerve supply from the:
Glossopharyngeal nerve
Hypoglossal nerve
Spinal accessory nerve
Sympathetic trunk
Vagus nerve
The correct answer is: Vagus nerve (CN X)
The vagus nerve supplies motor innervation to the muscles of the larynx and pharynx, with the exception of stylopharyngeus (innervated by the glossopharyngeal nerve). It also supplies motor innervation to the palate muscles, with the exception of tensor veli palatini (innervated by the V3 division of the trigeminal nerve). The glossopharyngeal nerve (CN IX) provides the sensory, but not motor, innervation to the pharynx. The hypoglossal nerve (CN XII) provides motor innervation to the muscles of the tongue. The accessory nerve (CN XI) provides motor innervation to the trapezius and the sternocleidomastoid. Finally, the sympathetic trunk supplies sympathetic innervation to the head and neck.
Twenty-four hours following a partial thyroidectomy where the inferior thyroid artery was also ligated (tied off), the patient now spoke with a hoarse voice (whisper), and had difficulty in breathing. Which nerve was injured?

Internal branch of superior laryngeal
Ansa cervicalis
Ansa subclavia
Recurrent laryngeal
External branch of superior laryngeal
The correct answer is: Recurrent laryngeal
The recurrent laryngeal nerve runs with the inferior thyroid artery toward the lower lobes of the thyroid. This means that the recurrent laryngeal nerve would be at risk in any surgery involving the inferior thyroid artery or the inferior poles of the thyroid. The recurrent laryngeal nerve becomes the inferior laryngeal nerve at the inferior border of cricoid cartilage, and this nerve continues on to innervate all the muscles of the larynx with the exception of cricothyroid. So, an injury to the recurrent laryngeal nerve might lead to hoarseness and difficulty breathing (due to a laryngeal spasm).

The internal branch of superior laryngeal runs with the superior laryngeal artery and pierces the thyrohyoid membrane. Ansa cervicalis is a branch of the cervical plexus which hangs in front of the internal jugular vein. It innervates the strap muscles, not the laryngeal muscles. Ansa subclavia is part of the sympathetic trunk which loops around the subclavian artery. Finally, the external branch of the superior laryngeal nerve runs with the superior thyroid artery. This is the artery and nerve that might be damaged when removing the superior lobe of the thyroid.
In performing a thyroidectomy, caution should be exercised when ligating (tying) the inferior thyroid artery, as it lies in a very close relationship to which nerve?
ansa cervicalis
hypoglossal
phrenic
recurrent laryngeal
vagus
The correct answer is: recurrent laryngeal
The recurrent laryngeal nerve crosses the inferior thyroid artery near the lower lobe of the thyroid. This means that the recurrent laryngeal nerve would be at risk in any surgery involving the inferior thyroid artery or the inferior poles of the thyroid. The recurrent laryngeal nerve becomes the inferior laryngeal nerve at the inferior border of cricopharyngeus, and this nerve continues on to innervate all the muscles of the larynx with the exception of cricothyroid. So, you really need to take care to protect the recurrent laryngeal nerve--injuring this structure could lead to hoarseness, permanent loss of voice, or even death due to a laryngeal spasm.

None of the other listed nerves are related to the inferior thyroid artery. Ansa cervicalis is a branch of the cervical plexus which hangs in front of the internal jugular vein. It innervates the strap muscles. The hypoglossal nerve winds behind the internal jugular vein, then sweeps around lateral to the carotid vessels and into the floor of the mouth, where it innervates the tongue muscles. The phrenic nerve lies on top of the anterior scalene muscles--it then travels through the thorax to innervate the diaphragm. Finally, the vagus is found in the carotid sheath--it is associated with the common carotid and the internal carotid arteries.
A 60-year-old man has occasional blackouts and light-headedness. Studies reveal atherosclerotic plaques within the common carotid arteries and the bifurcation of the vessels. A carotid endartectomy is undertaken. Which nerve bundle running vertically within the carotid sheath must the surgeon be careful not to injure?
Accessory
Cervical sympathetic trunk
Glossopharyngeal
Hypoglossal
Vagus
The correct answer is: Vagus
In a carotid endarterectomy, surgeons enter the carotid artery and excise the diseased endothelium and media of the artery so that the lumen is left with a smooth lining. To do this, the surgeons must enter the carotid sheath, which means that any structure within that sheath or near that sheath might be injured. This question specifically asks which structure in the sheath could be injured, so the correct answer is the vagus nerve. The carotid sheath contains 2 nerves: the vagus and the superior ramus of ansa cervicalis. It also contains 3 vessels: internal carotid, common carotid, and internal jugular vein. Any of these structures could be damaged during the procedure.

The accessory nerve is most closely associated with the posterior triangle of the neck. It cuts through this triangle to innervate sternocleidomastoid and trapezius and lies posterior to the carotid vessels The cervical sympathetic trunk lies in front of the prevertebral muscles in the prevertebral fascia. The glossopharyngal nerve comes out of the jugular foramen and divides into 2 branches--pharyngeal and lingual branches. This nerve sweeps near the carotid sheath, but it's not in the carotid sheath. The hypoglossal nerve travels lateral to the carotid vessels before entering the floor of the mouth. It crosses the carotid artery, but it's not in the sheath.
A 55-year-old woman has difficulty swallowing and frequently aspirates fluids while drinking. She is diagnosed as having a skull base tumor occupying the space behind the jugular foramen. Involvement of which structure is responsible for the findings?
Ansa cervicalis
Cervical sympathetic trunk
Accessory nerve
Hypoglossal nerve
Vagus
The correct answer is: Vagus
The vagus nerve, which exits the skull through the jugular foramen, is the motor nerve to the pharynx. So, it allows for swallowing. This patient's symptoms and the location of the tumor clearly point to an injury of the vagus nerve. The accessory nerve also exits the skull through the jugular foramen--it could be injured from this tumor, but the symptoms are not suggestive of an accessory nerve injury. If her accessory nerve was damaged, the patient's sternocleidomastoid and trapezius would be denervated, which means the acromion of her shoulder would sag. The other nerve that comes through the jugular foramen is the glossopharyngeal nerve--this nerve might also be damaged by the tumor.

The other nerves listed do not come from the jugular foramen, so they would not be the first nerves injured by the tumor. Ansa cervicalis is a branch of the cervical plexus that innervates the strap muscles in the anterior neck. The cervical sympathetic trunk is the structure that provides sympathetic innervation to the head and neck. If this trunk was disrupted, the patient would have Horner's syndrome. The symptoms of Horner's syndrome include: ptosis, constricted pupil, lack of sweating on the affected side, flushing on the affected side, and enophthalmos (sunken-in eye). Finally, the hypoglossal nerve exits the skull through the hypoglossal canal, providing motor innervation to the muscles of the tongue. An injury to the hypoglossal nerve would cause the tongue, when protruded, to point to the side of the face with the injured nerve.
While doing a postoperative physical on a patient who has undergone carotid endarterectomy on the right side, it was noted that the tongue deviated toward the right when the patient was asked to point the tongue outward. What nerve crossing the carotid artery must have been injured?
Glossopharyngeal
Hypoglossal
Inferior alveolar
Lingual
Vagus
The correct answer is: Hypoglossal
The hypoglossal nerve crosses the carotid artery before diving into the floor of the mouth, so it might get injured during this procedure. This nerve innervates the muscles of the tongue. When the hypoglossal nerve is injured and a patient sticks out the tongue, the tongue points to the side with the injured nerve. So, this patient, who had the right side endarctectomy, has an injury to the right hypoglossal nerve. The symptoms that you see in this case should point you toward suspecting damage to the hypoglossal nerve.

The glossopharyngeal nerve crosses around the internal carotid artery and the stylopharyngeus muscle--it could theoretically be injured in this procedure, but the symptoms are not consistent with an injury to this structure. The inferior alveolar nerve and lingual nerves are branches of the mandibular division of the trigeminal nerve. They are not associated with the carotid vessels. The vagus nerve is found in the carotid sheath. This nerve can be easily injured by an endarterectomy, although damage to the vagus would cause problems with swallowing, not with tongue movements.
Any manipulation of the superior thyroid artery must be undertaken with care not to damage its small companion nerve, the:
Cervical sympathetic trunk
External branch of the superior laryngeal
Inferior root of the ansa cervicalis
Internal branch of the superior laryngeal
Recurrent laryngeal
The correct answer is: External branch of the superior laryngeal
The external branch of the superior laryngeal nerve runs with the superior thyroid artery--this artery and nerve might be damaged when removing the superior pole of the thyroid. If this nerve was damaged, the cricothyroid muscle would be paralyzed, and a patient would be unable to tense the vocal cords. The internal branch of the superior laryngeal nerve runs with the superior laryngeal artery and provides sensory innervation to the mucous membrane of the larynx, superior to the vocal fold. The recurrent laryngeal nerve crosses the inferior thyroid artery, near the lower pole of the thyroid. This means that the recurrent laryngeal nerve would be at risk in any surgery involving the inferior thyroid artery or the inferior poles of the thyroid. The recurrent laryngeal innervates all the muscles of the larynx with the exception of cricothyroid. So, an injury to the recurrent laryngeal nerve might lead to hoarseness and dificulty breathing.

The cervical sympathetic trunk lies in front of the prevertebral muscles in the prevertebral fascia--it would not be near the structures involved with thyroid surgery. The inferior root of ansa cervicalis is a branch of the cervical plexus. It innervates the strap muscles. Ansa cervicalis hangs in a loop over the carotid vessels; it is not associated with the thyroid gland.
The carotid body is innervated by a branch of the:
Glossopharyngeal nerve
Hypoglossal nerve
Spinal accessory nerve
Sympathetic trunk
Vagus nerve
The correct answer is: Glossopharyngeal nerve (CN IX)
The carotid body is innervated by the carotid branch of the glossopharyngeal nerve. It is a small, reddish-brown ovoid mass that lies on the medial side of the carotid bifurcation, serving as a chemoreceptor that monitors the level of carbon dioxide in the blood. The hypoglossal nerve (CN XII) innervates the muscles of the tongue. The spinal accessory nerve is a motor nerve that innervates the sternocleidomastoid and the trapezius. The sympathetic trunk provides sympathetic innervation to the head and neck. Finally, the vagus nerve supplies motor innervation to the muscles of the pharynx and larynx, with the exception of stylopharyngeus, and the muscles of the palate, with the exception of tensor veli palatini.
Loss of sensation from the temporal region and loss of secretory function of the parotid gland would be caused by interruption of which nerve?
Auriculotemporal
Chorda tympani
Deep temporal, posterior
Facial
Great auricular
The correct answer is: Auriculotemporal nerve
The auriculotemporal nerve is a branch of the mandibular division of the trigeminal nerve (V3). It has two important functions: First, it carries postganglionic parasympathetic fibers to the parotid gland. These fibers come from the otic ganglia, where they synapsed with the presynaptic fibers from the glossopharyngeal nerve (CN IX). Second, the auriculotemporal nerve provides sensory innervation to the skin of anterosuperior ear, part of the external auditory meatus, and the temporomandibular joint. So, the listed symptoms match with an injury to the auriculotemporal nerve.

Chorda tympani is a branch of the facial nerve that provides secretomotor innervation to the submandibular and sublingual glands. It carries preganglionic parasympathetic axons to the submandibular ganglion. In the infratemporal fossa, chorda tympani joins the lingual nerve--it continues with the lingual nerve to the tongue where it supplies taste to the anterior 2/3 of the tongue. The posterior deep temporal nerve is a branch of the mandibular division of the trigeminal nerve which supplies motor innervation to temporalis. The facial nerve (CN VII) innervates all of the muscles of facial expression, and, through the chorda tympani, provides secremotor innervation to the submandibular and sublingual glands as well as taste sensation to the anterior 2/3 of the tongue. Finally, the great auricular nerve comes from the cervical plexus--it provides sensory innervation to the skin of the ear and the skin below the ear.
An elderly man presented with severe pain beneath the left eye, radiating into the lower eyelid, lateral side of the nose and upper lip. What nerve was involved?
Buccal
Infraorbital
Mental
Supratrochlear
Zygomatic
The correct answer is: infraorbital
The infraorbital nerve is a cutaneous nerve from the maxillary division of trigeminal nerve (V2). It innervates the skin of the lateral nose, lower eyelid, upper lip and zygomatic region. This is exactly where this man's pain is, so it seems like his pain must be transmitted on the infraorbital nerve. The buccal branch of the trigeminal nerve is part of the mandibular division (V3)--this nerve provides sensory innervation to the skin of the cheek and the mucosal lining the cheek. It is not a motor nerve--only sensory! (NOTE: The buccal nerve is NOT the same as the buccal branch of the facial nerve. The buccal branch of the facial nerve is a motor nerve only--it innervates several muscles of facial expression. It does not have a sensory component--only motor!) The mental nerve is a branch of the inferior alveolar nerve, which is a branch of the mandibular division of the trigeminal nerve (V3). It provides sensory innervation to the skin of the chin and lower lip. The supratrochlear nerve is a branch of the frontal nerve, from the ophthalmic division of the trigeminal nerve. It gives sensory innervation to the skin of the medial forehead and the medial part of the upper eyelid. The zygomatic nerve is part of the maxillary division of the trigeminal nerve (V2). It provides sensory innervation to the skin of the face lateral and superior to the orbit. For a good picture of these nerves, see Netter Plate 18.
During a face lift operation on a 48-year-old woman, the plastic surgeon inadvertently cut the marginal mandibular branch of the facial nerve. Which of the following muscles would be paralyzed because of the injury?
Buccinator
Depressor anguli oris
Levator anguli oris
Levator labii superioris
Stylohyoid
The correct answer is: depressor anguli oris
The marginal mandibular branch of the facial nerve provides motor innervation to the muscles of facial expression near the lower lip and chin--right where you find depressor anguli oris. So, if the marginal mandibular branch of the facial nerve was injured, depressor anguli oris would be paralyzed. The buccal branches of the facial nerve provide motor innervation to the buccinator muscle and the muscles of the upper lip (levator anguli oris and levator labii superioris). Finally, stylohyoid is innervated by the facial nerve shortly after exiting the stylomastoid foramen--it is not innervated by any of the special branches of the facial nerve that innervate the muscles of facial expression.
As a result of a face lift operation, a 46-year-old woman noticed an asymmetry of the inferior lip and could not fully depress the angle of her mouth on the right side. Which of the following nerves was most likely damaged during the surgery?
zygomatic (VII)
buccal (VII)
mental (V3)
marginal mandibular (VII)
infraorbital (V2)
The correct answer is: marginal mandibular
Depressor anguli oris is the muscle that depresses the angle of the lip--it is innervated by the marginal mandibular branch of the facial nerve. So, if the marginal mandibular branch of the facial nerve was injured, depressor anguli oris would be paralyzed. The zygomatic branches of the facial nerve innervate the muscles of facial expression that are right around the eye, including orbicularis oculi. The buccal branches of the facial nerve innervate the buccinator muscle and other muscles of facial expression that are near the upper lip, like levator anguli oris and levator labii superioris. The mental and infraorbital branches of the trigeminal nerve provide sensory innervation to the skin of the face--they do not innervate any muscles! The mental nerve, a branch of V3 (mandibular division), innervates the skin of the chin and the lower lip. The infraorbital nerve, a branch of V2 (maxillary division), innervates the skin of the lateral nose, lower eyelid, upper lip, and zygomatic region.
An elderly woman complained of a severe pain, felt above the right eye radiating to the upper eyelid, side of the nose and forehead. Branches of which of the following nerves convey pain sensations from areas of the skin described?
maxillary (V2)
greater auricular nerve
ophthalmic (V1)
mandibular (V3)
facial (VII)
The correct answer is: ophthalmic (V1)
The ophthalmic division of the trigeminal nerve provides sensory innervation to the skin of the nose, upper eyelid, and forehead. This is exactly where this woman feels pain, so the ophthalmic division of the trigeminal nerve must be the nerve transmitting the pain. The maxillary division of the trigeminal nerve (V2) provides sensory innervation to the skin of the side of the nose, the cheek, lower eyelid, and upper lip. The mandibular division of the trigeminal nerve (V3) provides sensory innervation to the skin of the chin, lower lip, and lower jaw. The great auricular nerve is a branch of the cervical plexus which supplies sensory innervation to the ear region. Finally, the facial nerve is mostly a motor nerve--it only supplies taste to the anterior 2/3 of the tongue and gives some sensory innervation to the skin of the exernal auditory meatus.
Due to multiple salivary calculi (stones) in the submandibular duct, the submandibular gland of a 45-year-old individual was surgically removed. What major artery directly related to the gland was of special concern to the surgeon?
lingual
superior thyroid
facial
ascending pharyngeal
maxillary
The correct answer is: facial
The facial artery arises from the external carotid and winds toward the inferior border of the mandible, crossing over the submandibular gland. So, if the submandibular gland was removed, the facial artery might be damaged. The lingual artery is a branch of the external carotid that runs in the floor of the mouth. It is associated with the submandibular duct, but not with the gland itself. The superior thyroid artery is a branch of the external carotid which travels anteroinferiorly to supply the upper pole of the thyroid. The ascending pharyngeal artery is a posterior branch of the external carotid which supplies blood to the pharynx. Finally, the maxillary artery is one of the two terminal branches of the external carotid artery--it supplies blood to the maxillary region, muscles of mastication, infratemporal fossa, and deep face. Take a look at Netter Plate 63 to get a better picture of this!
To study the compensatory response of selective suprahyoid muscles in elevating the hyoid bone, an experiment was designed in which the posterior belly of the digastric and stylohyoid muscles were paralyzed by drugs. The muscular branches of which of the following nerves must be chemically interrupted to produce paralysis in both muscles?
inferior alveolar
facial
hypoglossal
glossopharyngeal
lingual
The correct answer is: facial
The facial nerve (CN VII) provides motor innervation to the posterior belly of the digastric and the stylohyoid muscle. The inferior alveolar nerve is a branch of the mandibular division of the trigeminal nerve (V3) that innervates mylohyoid and the anterior belly of the digastric. The hypoglossal nerve (CN XII) supplies motor innervation to the intrinsic and extrinsic muscles of the tongue, with the exception of palatoglossus. The glossopharyngeal nerve (CN IX) provides motor innervation to stylopharyngeus. Finally, the lingual nerve is a sensory branch of the mandibular division of the trigeminal nerve (V3) which supplies general sense from the anterior 2/3 of the tongue and floor of the mouth.
Which nerve provides motor innervation to the buccinator muscle?
Auriculotemporal nerve
Buccal branches of VII
Buccal nerve
Mandibular division of V
Marginal mandibular nerve
The correct answer is: buccal branches of VII
The buccal branches of the facial nerve provide motor innervation to the buccinator muscle. Remember, these buccal branches of the facial nerve are motor nerves only--they do not do any sensory innervation. Don't mix this nerve up with the buccal nerve, which is a branch of the mandibular division of the trigeminal nerve (V3)! The buccal nerve is a sensory nerve only--it does not innervate any muscles; it only gives sensory innervation to the skin of the cheek and the mucosal lining of the cheek. The auriculotemporal nerve is also part of the mandibular division of the trigeminal nerve--it carries the postganglionic parasympathetic fibers to the parotid gland and provides sensory innervation to the skin of the anterosuperior ear, part of the external auditory meatus, and the temporomandibular joint. The marginal mandibular nerve is another branch of the facial nerve--it innervates the muscles of facial expression on the lower lip and chin.
Which nerve provides cutaneous innervation to the skin of the angle of the mandible?
Auriculotemporal nerve
Lesser petrosal nerve
Buccal branches of VII
Marginal mandibular nerve
Great auricular nerve
The correct answer is: great auricular nerve
The great auricular nerve is a branch of the cervical plexus that provides cutaneous innervation to the skin of the ear and skin below the ear, including the angle of the mandible. The auriculotemporal nerve is a branch of the mandibular division of the trigeminal nerve (V3) with two important functions. First, it carries postganglionic parasympathetic fibers to the parotid gland. Second, the auriculotemporal nerve provides sensory innervation to the skin of anterosuperior ear, part of the external auditory meatus, and the temporomandibular joint. The lesser petrosal nerve is not a sensory nerve--it is a branch of the glossopharyngeal nerve that carries preganglionic fibers to the otic ganglia. Finally, the buccal branches and marginal mandibular branches of the facial nerve are motor nerves only--not sensory nerves! The buccal branches of the facial nerve innervate the buccinator and the other muscles of facial expression above the lip. The marginal mandibular branch innervates the muscles of facial expression of the lower lip and chin.
Which nerve carries postganglionic parasympathetic fibers to the parotid gland?
Auriculotemporal nerve
Lesser petrosal nerve
Glossopharyngeal nerve
Great auricular nerve
Marginal mandibular nerve
The correct answer is: Auriculotemporal nerve
The auriculotemporal nerve is a branch of the mandibular division of the trigeminal nerve (V3). It has two important functions: First, it carries postganglionic parasympathetic fibers to the parotid gland. These fibers come from the otic ganglia, where they synapsed with the presynaptic fibers from the glossopharyngeal nerve (CN IX). These presynaptic fibers were carried to the otic ganglia by the lesser petrosal nerve. Second, the auriculotemporal nerve provides sensory innervation to the skin of anterosuperior ear, part of the external auditory meatus, and the temporomandibular joint.

The great auricular nerve is a sensory nerve from the cervical plexus--it innervates the skin of the ear and the skin below the ear. The marginal mandibular nerve is a branch of the facial nerve--it innervates the muscles of facial expression for the lower lip and chin.
A patient is unable to wink; what muscle is affected?
frontalis
levator palpebrae superioris
orbicularis oculi
superior tarsal
zygomaticus major
The correct answer is: orbicularis oculi
Orbicularis oculi is a muscle of facial expression that closes the eyelid for winking. It is innervated by the temporal and zygomatic branches of the facial nerve. Frontalis is the anterior belly of the epicranius muscle; it elevates the eyebrows and wrinkles the forehead. It is innervated by the temporal branches of the facial nerve (VII). Levator palpebrae superioris elevates the upper eyelid; it is innervated by the oculomotor nerve (III). The superior tarsal muscle is another muscle that elevates the eyelid, but it lifts the eyelid invoulntarily. It is innervated by the cervical sympathetic trunk; checking whether this muscle is functioning is a good test to see whether the cervical sympathetic trunk is intact. Finally, zygomaticus major is a muscle at corner of the mouth which elevates and draws the corner of the mouth laterally. It is innervated by the zygomatic and buccal branches of the facial nerve.
What structure lies deepest in the parotid gland?
External carotid artery
External jugular vein
Facial artery
Facial nerve
Retromandibular vein
The correct answer is: external carotid artery
The facial nerve, retromandibular vein, and external carotid artery all course through the parotid gland. From superficial to deep, they are arranged nerve, vein, artery. So, the facial nerve would be the structure most likely to be injured by a superficial injury to the parotid gland, while the external carotid artery is somewhat protected, deep in the gland. The facial artery branches from the external carotid artery before the external carotid artery enters the parotid gland. The external jugular vein is a superficial vein on the lateral surface of the neck.
A deep laceration of the face in the middle of the parotid gland could affect the:
External jugular vein
Facial nerve
Glossopharyngeal nerve
Hypoglossal nerve
Lingual artery
The correct answer is: facial nerve
The facial nerve travels through the parotid gland--it could become injured if there was a deep laceration through the parotid gland. Two other structures found within the parotid gland which might be damaged: the retromandibular vein and the external carotid artery. The nerve is the most superficial structure in the gland. Then, the vein is under the nerve, and the artery is the deepest structure in the gland.

The external jugular vein is a superficial structure on the lateral neck, so it's not really close to the parotid gland. The glossopharyngeal nerve is closely related to the stylopharyngeus muscle--it sweeps along the back of this muscle. It is not related to the parotid gland. The hypoglossal nerve travels laterally to the carotid vessels and then enters the floor of the mouth. This means that it travels inferior to the region of the parotid gland. Finally, the lingual artery is found in the floor of the mouth--far from the parotid gland!
Pain elicited from an infected facial wound is primarily conveyed by what nerve?
Facial
Great auricular
Hypoglossal
Transverse cervical
Trigeminal
The correct answer is: trigeminal
The trigeminal nerve is the nerve that supplies sensory innervation to the skin of the face, so pain sensations will be carried through this nerve. It has 3 divisions. The ophthalmic division (V1) is a sensory nerve that passes through the superior orbital fissure and supplies sensory innervation to the eyeball, conjunctiva, nasal mucosa, medial portion of the nose, upper eyelid, forehead, and scalp. The maxillary division (V2) is a sensory nerve that provides sensory innervation to the cheek, upper lip, lateral portion of nose and lower eyelid. The mandibular division (V3) is a sensory and motor nerve--it supplies the lower lip and chin and the lateral portion of the cheek. V3 also provides motor innervation to the muscles of mastication, tensor veli palatini, mylohyoid, the anterior belly of the digastric, and tensor tympani.

The facial nerve does not provide sensory innervation to the skin of the face--it provides motor innervation to the muscles of facial expression. The great auricular nerve and transverse cervical nerve are branches of the cervical plexus. The great auricular nerve supplies the skin of the ear and the skin below the ear, while the transverse cervical nerve supplies the skin of the anterior neck. The hypoglossal nerve (CN XII) provides motor innervation to the muscles of the tongue.
Inability to close the lips relates to the action of which muscle?
Anterior belly of the digastric
Mylohyoid
Orbicularis oris
Platysma
Zygomaticus major
The correct answer is: Orbicularis oris
Orbicularis oris is a muscle of facial expression. It surrounds the lips, and allows for pursing of the lips. It is innervated by the buccal branch of the facial nerve. The anterior belly of the digastric and mylohyoid both help elevate the hyoid bone and depress the mandible. They are both innervated by a branch of the mandibular division of the trigeminal nerve (V3). Platysma and zygomaticus major are both muscles of facial expression, innervated by the facial nerve. Platysma draws the corners of the mouth down and aids in depressing the mandible; zygomaticus major elevates and draws the corners of the mouth laterally.
Which muscle will not be affected when the mandibular division of the trigeminal nerve (V3) is anesthetized?
Anterior belly of digastric
Buccinator
Medial pterygoid
Mylohyoid
Temporalis
The correct answer is: Buccinator
Buccinator is innervated by the facial nerve (CN VII). It allows the corner of the mouth to be pulled laterally, and it allows the cheek to be pulled against the teeth. It is an important muscle for mastication, but it's not innervated by a branch of the mandibular division of the trigeminal nerve. (Note: the buccal branches of the facial nerve, which innervate buccinator, are not the same as the buccal nerve, a sensory branch of V3!)

The other muscles listed are all innervated by branches of V3. Mylohyoid and the anterior belly of the digastric are innervated by a small branch of V3. They both elevate the hyoid and depress the mandible. Medial pterygoid and temporalis are muscles of mastication, and the muscles of mastication are innervated by V3.
A 38-year-old female patient complained of parotid pain that increased while eating. Intraoral examination detected some pus oozing from the parotid duct opening. What was the most likely anatomical reference that the physician considered to locate the parotid duct opening?
Mucosa of the sublingual caruncle behind the central incisor teeth
Mucosa of the cheek across the 2nd upper (maxillary) molar tooth
Mucosa of the floor of the mouth along the sublingual fold
Mucosa of the cheek across the 2nd lower (mandibular) molar tooth
The correct answer is: Mucosa of the cheek across the 2nd upper (maxillary) molar tooth
The parotid duct opens into the vestibule of the mouth, draining into the mucosa of the cheek near the second upper molar tooth. The duct drains the parotid gland across the masseter and through the cheek. It passes through the buccinator muscle, and pours saliva into the vestibule of the mouth.

The sublingual caruncle is a small bump in the floor of the mouth, near the frenulum of the tongue. This is the site of the opening of the submandibular duct, which drains saliva from the submandibular gland. The mucosa along the sublingual fold contains many openings for the ducts coming from the sublingual gland. See Netter Plate 45 for a picture of all these ducts.
While recovering from a right facial paralysis, a 36-year-old female patient complained that food accumulated between the teeth and the cheek mucosa when chewing. The deficiency of which muscle was most likely the cause of the chewing problem?
Zygomaticus major
Orbicularis oris
Buccinator
Levator labii superioris
The correct answer is: Buccinator
Although buccinator is innervated by the buccal branches of the facial nerve and not a branch of V3, the buccinator is an important muscle for mastication. The buccinator keeps the cheek taut so it is not folding over and becoming injured by chewing. It aids mastication by pulling the cheek against the molar teeth so that food does not keep collecting in the vestibule of the mouth. So, if this muscle was injured, the cheek could not press against the molar teeth, and food would fall between the teeth and cheek mucosa while chewing.

The other 3 muscles are all innervated by the facial nerve, but they are important for facial expression not mastication. Zygomaticus major elevates and draws the corners of the mouth laterally. Orbicularis oris purses the lips. Levator labii superioris elevates the upper lip.
The parotid space contains all EXCEPT:
External carotid artery
Facial nerve
Intraparotid lymph nodes
Medial pterygoid muscle
Retromandibular vein
The correct answer is: Medial pterygoid muscle
The medial pterygoid muscle is not in the parotid space. It serves as the anterior boundary of the parotid fossa. The other structures mentioned are all found within the parotid gland. The facial nerve, retromandibular vein, and external carotid artery all course through the parotid gland. From superficial to deep, they are arranged nerve, vein, artery. The intraparotid lymph nodes are found in the parotid gland also.
As a result of meningitis, a patient develops Bell's palsy. One of the symptoms was hyperacusis. What nerve was involved?
Facial
Glossopharyngeal
Oculomotor
Trigeminal
Vagus
The correct answer is: Facial
Bell's palsy is the paralysis of the facial nerve. This means that, on the affected side, the muscles of facial expression will appear flaccid, leading to a loss of expression. The patient will be unable to smile, lift their eyebrow, or close their eyelid on the affected side of the face. Besides the muscles of facial expression, the facial nerve also innervates stapedius, a small muscle in the ear. This muscle serves to dampen the vibrations of the tympanic membrane and quiet sounds. If the facial nerve is paralyzed, stapedius is paralyzed, too. This means that the ear can't dampen the vibrations from loud sounds, and the patient experiences hyperacusis.

It's important to remember the association between Bell's palsy and the facial nerve!
Frey's Syndrome is marked by profuse sweating over one cheek, temple, and surrounding areas of the face, precipitated by eating. The condition may be idiopathic, but often follows parotid surgery. The condition is attributable to abberant reinnervation, the redirection of autonomic fibers normally going to salivary glands being redirected to sweat glands. What is the source of the nerve fibers involved?
Facial
Glossopharyngeal
Oculomotor
Trigeminal
Vagus
The correct answer is: Trigeminal
Frey's syndrome is a condition in which the postganglionic parasympathetic nerves that are contained in the auriculotemporal nerve (which normally supply the parotid gland) are redirected toward the sweat glands overlying the parotid gland. This means that a patient with Frey's syndrome sweats in the area over the parotid gland while eating. Since the auriculotemporal nerve is a branch of V3, the nerve fibers involved in Frey's syndrome are from the trigeminal nerve.

The nerve fibers on the external carotid and internal carotid arteries are sympathetic fibers. Remember--sympathetic fibers create periarterial plexuses that travel with the vasculature to reach different targets around the body. However, the parotid gland is innervated parasympathetically, not sympathetically. The glossopharyngeal nerve contributes to the innervation of the parotid gland by supplying preganglionic parasympathetic fibers to the otic ganglia, by way of the lesser petrosal nerve. However, these fibers are not involved with Frey's syndrome--this condition involves the misdirection of the postganglionic parasympathetic fibers. Finally, the vagus is not involved with innervating the parotid gland.
While recovering from multiple dental extractions, an elderly man experienced a radiating pain affecting the lower eyelid, lateral side of the nose, upper lip and over the zygomatic and temporal areas on the left side. Which nerve is involved in the patient's perception of pain?
Facial
Opthalmic division of trigeminal
Glossopharyngeal
Mandibular division of trigeminal
Maxillary division of trigeminal
The correct answer is: Maxillary division of trigeminal
The trigeminal nerve is the nerve that supplies sensory innervation to the skin of the face. It has 3 divisions. The maxillary division of trigeminal (V2) is the one that's important for this case--it is a sensory branch of the trigeminal that provides innervation to the skin of the cheek, upper lip, lower eyelid, and the lateral portion of the nose. This is exactly the area that the patient feels pain, so it is the correct answer. The ophthalmic division (V1) is a sensory nerve that passes through the superior orbital fissure and supplies sensory innervation to the eyeball, conjunctiva, nasal mucosa, medial portion of the nose, upper eyelid, forehead, and scalp. The mandibular division (V3) is a sensory and motor nerve--it supplies skin of the lower lip, chin and lower jaw. V3 also provides motor innervation to the muscles of mastication, tensor veli palatini, mylohyoid, the anterior belly of the digastric, and tensor tympani.

The facial nerve innervates the muscles of facial expression, but it does not provide sensory innervation to the skin of the face. The glossopharyngeal nerve provides sensory innervation to the pharynx and sensory and taste innervation to the posterior 1/3 of the tongue. But, it does not innervate any skin on the face.
The facial muscle most responsible for moving the lips both upward and laterally to produce a smile is:
Buccinator
Levator anguli oris
Levator labii superioris
Platysma
Zygomaticus major
The correct answer is: Zygomaticus major
Zygomaticus major is innervated by the zygomatic and buccal branches of the facial nerve. It elevates the corner of the mouth and draws it laterally. Remember zygomaticus major as the "smile" muscle! Levator anguli oris is close to the correct answer. This muscle, which is innervated by the facial nerve, elevates the corners of the mouth. However, zygomaticus major also draws the mouth laterally to produce a smile, so this answer is more correct.

The three other muscles are all innervated by branches of the facial nerve. Buccinator is innervated by the buccal branches of the facial nerve. It allows the cheek to be pulled taut against the molar teeth. Levator labii superioris is innervated by the buccal branch of the facial nerve--it pulls the lip upwards. Platysma is innervated by the cervical branches of the facial nerve. It draws the corners of the mouth downward and aids in depressing the mandible.
You are testing the extraocular muscles and their innervation in a patient who periodically experiences double vision. When you ask him to turn his right eye inward toward his nose and look downward he is able to look inward, but not down. Which nerve is most likely involved?
Abducens
Nasociliary
Oculomotor, inferior division
Oculomotor, superior division
Trochlear
The correct answer is: trochlear
To understand this question, you need to understand how the motions of the eye are tested. Since the actions of the extraocular muscles are complex, it is necessary to turn the eye to a position where a single action of each muscle predominates when evaluating the individual muscles. A key principle for muscle testing is: if a muscle has two actions and you perform one of those two, then it can't perform its other action. Superior and inferior recti turn the eye in and up or in and down. Superior and inferior oblique turn the eye out and down or out and up. So, if you turn your eye in (with the superior and inferior rectus as well as medial rectus), then only superior and inferior oblique can move the eye down or up (because the superior and inferior recti are already shortened by turning the eye in - they can't shorten any more). Similarly, if you turn the gaze out (with the obliques and lateral rectus) then only superior and inferior rectus can turn the eye up or down.

In this case, the patient has the eye turned inward, so the doctor must be testing the oblique muscles. The superior oblique muscle is the muscle that lowers the eye when it is turned inward. Since the patient can't do this, the superior oblique must not be functioning, and this muscle is innervated by the trochlear nerve.

Abducens (CN VI) innervates the lateral rectus muscle, which is not involved in the eye test. The nasociliary nerve comes from the ophthalmic division of the trigeminal nerve (V1). It is a sensory nerve to the eyeball that also carries some sympathetic fibers. The inferior division of the oculomotor nerve innervates inferior rectus, inferior oblique, and medial rectus. All of these muscles appear to be functioning. Finally, the superior division of the oculomotor nerve innervates levator palpebrae superioris and superior rectus. These are not the muscles that appear to be malfunctioning.
The outermost layer of the optic nerve sheath is a continuation of the:
Arachnoid membrane
Meningeal dura
Periosteal dura
Pia mater
Retina
The correct answer is: meningeal dura
The optic nerve comes off the base of the brain and passes through the optic canal. As it leaves the brain, it still retains all of the meningeal layer coverings. So, it is covered by meningeal dura, arachnoid membrane, and pia mater. This is significant, because an increase in intracranial pressure will increase the pressure in the subarachnoid space. This may squeeze the optic nerve and make the optic nerve bulge into the eye, a condition known as papilledema.

The periosteal dura is the layer of periosteum covering the internal surface of the calvaria. The retina is the inner layer of the eyeball which receives and absorbs visual light rays.
The inner lining of the eyelid is called the:
Orbital septum
Palpebral conjunctiva
Periorbita
Sclera
Tarsal plate
The correct answer is: palpebral conjunctiva
The palpebral conjunctiva is the thin membrane that lines the eyelid. It is continuous with the bulbar conjunctiva which lines the eyeball. The orbital septum is a weak membrane that spans from the tarsal plates to the margins of the orbit where it becomes continuous with the periosteum. It contains orbital fat and can limit the spread of infection in the orbit. The periorbita is the periosteum lining covering the bones forming the orbit. The sclera is the outer fibrous layer of the eyeball. Finally, the tarsal plate is a thin, cardboard-like layer of connective tissue in the eyelids which forms the "skeleton" of the eyelids.
What would the examining physician notice in the eye of a person who has taken a sympathetic blocking agent?
Exophthalmos and dilated iris
Enophthalmos and dry eye
Dry eye and inability to accommodate for reading
Wide open eyelids and loss of depth perception
Ptosis and miosis (pin-point pupil)
The correct answer is: Ptosis and miosis (pin-point iris)
Start this question out by thinking about what a sympathetic blocker would do to the pupil of the eye. Since sympathetic nerves allow the pupil to dilate, a sympathetic blocker would stop the eye from dilating and make the pupil constrict. Now think about the other issues. First, remember that sympathetic nerves innervate the superior tarsal muscle, which elevates the eyelids. If there is a problem with the regional sympathetics (as is the case with Horner's syndrome), the superior tarsal muscle will be paralyzed, and the eyelid will droop (ptosis). If the sympathetic nervous system is inhibited, sweating will cease, and you will observe the eye sinking back into the orbit.

Accomodation is not mediated by the sympathetic system; accomodation is a function of parasympathetic nerve so this should not be affected. Finally, the lacrimal gland is innervated by parasympathetics, so there should not be a major change in eye secretions after a sympathetic blocker. Putting all of these factors together, answer choice E is the only one that fits!
You are examining a patient who has a pituitary tumor involving the cavernous sinus. While doing a preliminary eye exam, you suspect the right abducens nerve of the patient has been damaged by the tumor. In which direction would you have the patient turn his right eye to confirm the defect?
Inward
Outward
Downward
Down and out
Down and in
Upward
Up and out
Up and in
The correct answer is: outward
To understand this question, you need to understand how the motions of the eye are tested. Since the actions of the extraocular muscles are complex, it is necessary to turn the eye to a position where a single action of each muscle predominates when evaluating the individual muscles. For the superior and inferior recti, turning the eye outward (abduction) by approximately 25 degrees places the superior rectus in position to raise the eye and the inferior rectus to lower the eye. Similarly, turning the eye inward (adduction) approximately 50 degrees places the inferior oblique in position to raise the eye and the superior oblique to lower the eye. The medial and lateral recti may be checked while the eye is staring straight ahead since they have simple planar actions.

In this case, you're interested in testing an "easy" muscle. Since the lesion appears to be in the abducens, which innervates the lateral rectus muscle, you could just ask the patient to turn the eye outward. If the patient could not do this, it would confirm that there was a lesion in the abducens nerve, since the muscle responsible for lateral movement of the eye would be paralyzed.

Also remember--a tumor in the cavernous sinus could affect many nerves. The oculomotor nerve (CN III), trochlear (CN IV), ophthalmic division of trigeminal (CN V1), and abducens (CN VI) all pass through the cavernous sinus.
You have a patient with a drooping right eyelid. You suspect Horner's syndrome. Which of the following signs on the right side would confirm this diagnosis?
Constricted pupil
Dry eye (lack of tears)
Exophthalmos
Pale, blanched face
Sweaty face
The correct answer is: constricted pupil
Horner's syndrome is a disorder involving damage to the sympathetic trunk in the neck. This means that the sympathetics of the head will be disrupted. This causes a variety of very characteristic symptoms, including a constricted pupil. Remember--sympathetic nerves innervate the dilator pupillae muscle. This muscle allows the eye to dilate. If these sympathetic nerves are lost, the pupil will contract.

Several of the other listed symptoms are the opposite of what you would expect with Horner's syndrome. Exophthalmos is the protrusion of the eye, but in Horner's syndome the eye sinks in, possibly due to the paralysis of a smooth muscle in the floor of the orbit. The face does not become blanched and sweaty with Horner's syndrome--instead, it becomes red and dry. Without the sympathetic nerve supply, the vasculature of the face cannot constrict. So, the arterioles in the patient's face are vasodilated, making the face red. Sympathetic nerves also innervate sweat glands; if these nerves are interrupted, the patient will not sweat and the face will appear very dry. Finally, the lacrimal gland is innervated by parasympathetics, not sympathetics. So, Horner's syndrome should produce no appreciable changes in tearing.

Make sure to know the different symptoms and signs of Horner's syndrome!
Following endarterectomy on the right common carotid, a patient is found to be blind in the right eye. It appears that a small thrombus embolized during surgery and lodged in the artery supplying the optic nerve. What artery would be blocked?
Central artery of the retina
Infraorbital
Lacrimal
Nasociliary
Supraorbital
The correct answer is: Central artery of the retina
The central artery of the retina is a branch of the ophthalmic artery. It is the sole blood supply to the retina; it has no significant collateral circulation and blockage of this vessel leads to blindness. The branches of this artery are what you view during a fundoscopic exam. The infraorbital artery is a branch of the maxillary artery. It comes through the infraorbital foramen, inferior to the eye. It supplies the maxillary sinus, the maxillary incisors, canine and premolar teeth, and the skin of the cheek below the orbit. The supraorbital artery is another branch of the ophthalmic artery. It comes through the supraorbital foramen or notch and supplies blood to the muscles, skin and fascia of the forehead. The lacrimal artery is a branch of the ophthalmic artery that supplies the lacrimal gland. The nasociliary artery doesn't exist, but there is a nasociliary nerve (the third and lowest branch of the ophthalmic division) that travels with the continuation of the ophthalmic artery.
You are asked to check the integrity of the trochlear nerve in the right eye of a patient. Starting with the eyes directed straight ahead, you would have the patient look:
Inward, toward the nose and downward
Inward, toward the nose and upward
Toward the nose in a horizontal plane
Laterally in a horizontal plane
Outward, away from the nose and downward
Outward, away from the nose and upward
The correct answer is: Inward, toward the nose and downward
To understand this question, you need to understand how the motions of the eye are tested. Since the actions of the extraocular muscles are complex, it is necessary to turn the eye to a position where a single action of each muscle predominates when evaluating the individual muscles. To test the superior and inferior recti, a patient needs to turn the eye outward approximately 25 degrees. At this postion, the superior rectus will simply act to raise the eye, and the inferior rectus will lower the eye. To test the superior and inferior obliques, a patient needs to turn the eye inward approximately 50 degrees. When the eye is in this position, the superior oblique muscle will act to lower the eye, and the inferior oblique will act to raise the eye.

So, now that you understand how to the test the eye, you have to decide which muscle is innervated by the trochlear nerve. And that's the superior oblique. So, to test this muscle, the eye needs to turn inward (toward the nose) and downward.

What nerves innervate the other muscles? The abducens nerve (CN VI) innervates the lateral rectus muscle. The oculomotor nerve (CN III) innervates the superior rectus, inferior rectus, medial rectus, and inferior oblique muscles.
The ducts of the lacrimal gland open into the:
Superior fornix of the conjunctiva
Inferior fornix of the conjunctiva
Lacrimal puncta
Lacrimal canaliculi
Lacrimal lake
The correct answer is: Superior fornix of the conjunctiva
Lacrimal fluid is produced by the lacrimal gland, which lies in a fossa in the superolateral part of each orbit. The fluid from this gland enters the conjunctival sac through up to 12 lacrimal ducts that open into the superior conjunctival fornix. The tears then flow to the medial angle of the eye and collect in the lacrimal lake. The lacrimal papilla are small elevations on the eyelids, found near the lacrimal lake. These papillae have small openings called the lacrimal puncta; tears flow from the lacrimal lake into these puncta. From there, the lacrimal fluid goes into small canniliculi which drain the fluid into the lacrimal sac. The lacrimal sac continues on as the nasolacrimal duct and drains tears into the inferior nasal meatus. Take a look at Netter Plate 77 and try to follow the path of tears from the lacrimal gland to the inferior meatus!
Starting from a position gazing straight ahead, to direct the gaze downward, the inferior rectus muscle must be active along with the:
Superior oblique
Inferior oblique
Medial rectus
Lateral rectus
Superior rectus
The correct answer is: Superior Oblique
The inferior rectus muscle depresses the eye and medially rotates it. So, to direct the gaze downward, you want to find a muscle that will depress the eye while counterbalancing the medial rotation with lateral rotation. And, the superior oblique, innervated by the trochlear nerve (CN IV), does just that--it depresses the eye while laterally rotating it. The inferior oblique muscle laterally rotates the eye and elevates the eye. The medial rectus adducts the eye--it does not raise or lower the eye. The lateral rectus abducts the eye--it also does not raise or lower the eye. Finally, the superior rectus elevates the eye and draws it medially.
During a physical examination it is noted that a patient has ptosis. What muscle must be paralyzed?
Orbicularis oculi, lacrimal part
Orbicularis oculi, palpebral part
Stapedius
Superior oblique
Superior tarsal (smooth muscle portion of levator palpebrae)
The correct answer is: Superior tarsal
The superior tarsal muscle is a smooth muscle which is sympathetically innervated. It is an involuntary muscle that elevates the eyelid. It is innervated by the cervical sympathetic trunk, and this muscle's functioning provides a good indication of the integrity of the cervical sympathetic trunk. If the cervical sympathetic trunk has been damaged, a patient will have ptosis, a droopy eyelid. Orbicularis oculi is innervated by the facial nerve. If this muscle is paralyzed, the problem won't be a droopy eyelid--instead, the patient won't be able to close the eyelid. This is why patients with Bell's palsy are prescribed lubricating eye drops--if they can't close the eyelid, they may be at risk for corneal irritation. Stapedius is another muscle innervated by the facial nerve -- it serves to dampen the vibrations of the stapes and the tympanic membrane. Finally, the superior oblique muscle depresses the eyeball and turns it laterally. It does not affect the eyelid.
The extraocular muscle that does not originate at or near the apex of the orbit is the :
Inferior oblique
Inferior rectus
Levator palpebrae superioris
Superior oblique
Superior rectus
The correct answer is: Inferior oblique
The inferior oblique muscle does not originate at the apex of the orbit. It takes origin from the floor of the orbit, lateral to the lacrimal groove. The inferior rectus and superior rectus muscles take origin from the common tendinous ring at the apex of the orbit. The levator palpebrae superioris takes origin from the apex of the orbit above the optic canal. The superior oblique muscle takes origin from the apex of the orbit, above the optic canal. For a picture of this, see Netter Plate 79.
An adolescent boy suffers from severe acne. As is often the case he frequently squeezed the pimples on his face. He subsequently develops a fever and deteriorates into a confused mental state and drowsiness. He is taken to his physician and after several tests a diagnosis of cavernous sinus infection and thrombosis is made. The route of entry to the cavernous sinus from the face was most likely the:
Carotid artery
Mastoid emissary vein
Middle meningeal artery
Ophthalmic vein
Parietal emissary vein
The correct answer is: Ophthalmic vein
The ophthalmic veins are continuous with the facial vein and the pterygoid plexus of veins. These veins drain the face toward the cavernous sinus. They are valveless, so infections from the face can drain into the cavernous sinus. Besides causing fever and confusion, thrombotic congestion and edema in the cavernous sinus can compress the nerves that traverse that space to exit through the superior orbital fissure(CN III, CN IV, CN V1, and CN VI). This can affect the function of the ocular muscles, so one symptom of a cavernous sinus infection might be an inability to perform different eye movements.

The carotid artery and middle meningeal artery would not be the source of the infections. Infections do not tend to enter through arterial circulation. Remember--the common carotid is the major source of blood to the head and neck, and the middle meningeal artery is the branch of the maxillary artery that supplies blood to the dura. The emissary veins are valveless veins of the scalp. These veins can carry blood from the scalp to the dural venous sinuses or in the reverse direction depending on blood pressure. These veins may carry infectious materials from the scalp into the dural venous sinuses, but they are not important for carrying infections to the cavernous sinus.
If a person looking inward towards their nose is unable to look down, which nerve may be injured?
Abducens (CN VI)
Inferior division of oculomotor (III)
Optic (II)
Superior division of oculomotor (III)
Trochlear (IV)
The correct answer is: Trochlear (IV)
To understand this question, you need to understand how to evaluate the muscles of the eye. Since the actions of the extraocular muscles are complex, it is necessary to turn the eye to a position where a single action of each muscle predominates. To isolate the superior and inferior recti, the patient needs to turn the eye outward by approximately 25 degrees. This places the superior rectus in position to raise the eye and the inferior rectus in position to lower the eye. Turning the eye inward approximately 50 degrees places the inferior oblique in position to raise the eye and the superior oblique in position to lower the eye. The medial and lateral recti are the easy muscles -- they may be checked while the eye is staring straight ahead since they have simple planar actions

So, this patient is looking inward, which means that the obliques are being tested. The patient can't look downward, which shows that the superior oblique is not functional. This is the only muscle innervated by the trochlear nerve (CN IV).

Abducens (CN VI) innervates the lateral rectus muscle, which is tested by asking the patient to move the eye outward. The inferior division of the oculomotor nerve innervates inferior rectus, inferior oblique, and medial rectus. The superior branch of the oculomotor nerve innervates levator palpebrae superioris and superior rectus muscles. Finally, the optic nerve (CN II) provides the special sense of vision, and it is not tested in the eye-movement tests.

Are you getting the idea that you really need to know about testing the eye muscles? Take the time and really understand this concept--you'll be glad that you did!
If a person is taking a sympathetic blocking agent, what would you notice in her or his eyes?
Dry eyes and inability to accommodate for reading
Enophthalmos and teary eyes (III)
Exophthalmos and dilated pupil
Ptosis and constricted pupil
Wide open eyes and loss of depth perception (IV)
The correct answer is: Ptosis and constricted pupil
To understand this question, it's important to look at all the different choices and determine which ones fit with a sympathetic block. First, the lacrimal gland is innervated parasympathetically, so a sympathetic blocker should have no effect on eye secretions. Accomodation is also a function of the parasympathetic nervous system; it should not be altered by a sympathetic blocker. Enophthalmos is the name for the eye sinking into its orbit. A sympathetic block does cause enophthalmos, due to the paralysis of a smooth muscle in the floor of the orbit. Exophthalmos is the opposite of enophthalmos--it is the protrusion of the eye from the orbit. You would not see exophthalmos with a sympathetic blockade. Sympathetic nerves allow the eye to dilate--if you blocked these nerves, the eye would constrict. A sympathetic blocker would also cause ptosis--it would paralyze the superior tarsal muscle, which holds the lids up involuntarily and receives sympathetic innervation. Finally, the sympathetic blocker should not affect depth perception. If you put all of these things together, answer choice D is the correct one.

If it helps to remember, taking a sympathetic blocking agent will lead to similar symptoms in the head and neck as Horner's syndrome, a disease characterized by a loss of sympathetic innervation to the head and neck.
The entry of bacteria through which space could lead to an infection in the mastoid air cells:
Auditory (nasopharyngeal) tube
Cochlea
External acoustic meatus
Internal acoustic meatus
Sacculus
The correct answer is: Auditory (nasopharyngeal) tube
The auditory (nasopharyngeal) tube is a connection between the nasal portion of the pharynx and the tympanic cavity that allows pressure to equalize on either side of the tympanic membrane. It is located in the anterior wall of the middle ear and is comprised of bone at the tympanic end and cartilage on the pharyngeal end. The pharyngeal mucosa is continuous with the lining of the tympanic cavity and mastoid air cells. This allows infectious material to pass to the middle ear and mastoid area.

The cochlea is the organ of hearing that receives, interprets, and transmits sound via the vestibulocochlear nerve (CN VIII). The external acoustic meatus is the opening in the temporal bone that allows sound waves to reach the tympanic membrane. The internal acoustic meatus is the foramen in the temporal bone that allows the vestibulocochlear nerve and the facial nerve to pass into the skull at the base of the brain. Finally, the sacculus is a fluid filled sac that is part of the balancing apparatus of the ear - it is located in the vestibule of the ear.
Which structure is attached to the center of the tympanic membrane?
Foot plate of the stapes
Handle (manubrium) of the malleus
Long process of the incus
Tragus
Utricle
The correct answer is: handle (manubrium) of the malleus
The handle of the malleus is attached to the center of the tympanic membrane. The head of the malleus then articulates with the body of incus, and the long process of incus articulates with the head of stapes. The foot plate of stapes sits in the oval window. Take a look at Netter Plate 88 for a better idea of how these ossicles connect!

The tragus is a cartilagenous structure of the external ear, located anterior to the external auditory meatus. It is hard to describe the tragus; it is labeled in the upper left picture of Netter Plate 88. The utricle is a part of the balancing apparatus of the ear--like the saccule, it is a fluid filled sac in the vestibule.
A 45-year old woman with recurrent left middle ear infection (otitis media) complained of partial dryness of her mouth to her ENT surgeon. Taste sensation and hearing were normal. After a thorough clinical examination at the hospital, the doctor concluded that the infection must have spread to a component of the glossopharyngeal nerve (CN IX) that supplies the parotid gland. On which of the following walls of the middle ear is this nerve component located?
Anterior
Lateral
Medial
Posterior
Roof
The correct answer is: Medial
To answer this question, you first need to identify the component of the glossopharyngeal nerve that innervates parotid gland. Remember--preganglionic parasympathetic fibers from CN IX are arriving at the otic ganglion via the lesser petrosal nerve and synapsing in the ganglion; the postganglionic fibers then travel on the auriculotemporal nerve (V3) and innervate the parotid gland. So, the lesser petrosal nerve, or fibers that create the lesser petrosal nerve, must have been injured. Now, you just need to think about where these fibers are in the ear.

These fibers from the glossopharyngeal nerve are covering the promontory on the medial wall of the ear. The tympanic nerve is a branch of the glossopharyngeal nerve that carries the preganglionic parasympathetic fibers that will eventually travel to the otic ganglion. The tympanic nerve lies on the promontory and creates the tympanic plexus, which gives rise to the lesser petrosal nerve. Given the clinical presentation, the patient must have an infection in the tympanic nerve, tympanic plexus or lesser petrosal nerve. And, all of these nerves are associated with the medial wall of the middle ear.

What are the important associations with the other walls of the ear? The lateral wall is formed by the tympanic membrane, and the chorda tympani courses across this membrane. The anterior wall contains the opening of the auditory tube and a semicanal for tensor tympani. The posterior wall of the ear has the aditus ad antrum, which is the entrance to the mastoid antrum and air cells. The facial nerve (VII) and a small branch of the facial nerve which innervates stapedius are found on the posterior wall.
A 3-year-old girl ruptured her eardrum when she inserted a pencil into her ear. Her mother took her to the emergency department after noticing that the child was crying and complaining of pain in her ear with a few drops of blood in the external auditory meatus. The attending doctor examined the child for possible injury to a nerve that runs across the eardrum. The most likely nerve to be injured is the:
Auricular branch of the vagus
Chorda tympani
Glossopharyngeal (CN IX)
Lesser petrosal
Trigeminal (CN V)
The correct answer is: chorda tympani
When this girl ruptured her eardrum, she damaged the tympanic membrane. So, you need to think about the nerve and structures that are associated with the tympanic membrane (ie, on the lateral wall of the middle ear). Chorda tympani lies across the tympanic membrane, so it's possible that this nerve was injured by the pencil.

The auricular branch of the vagus nerve is a small branch of the vagus that supplies afferent sensory innervation to the external acoustic meatus. This nerve is not close to the tympanic membrane The glossopharyngeal nerve and lesser petrosal nerve are associated with the promontory of the ear, which is on the medial wall of the middle ear. The trigeminal nerve is not close to the ear and would not be damaged by the injury.

What might happen if chorda tympani was injured? No taste sensation to the anterior 2/3 of the tongue and no secretomotor innervation to the sublingual and submandibular glands!
An elderly patient with chronic otitis media (middle ear infection) might have all the following complications EXCEPT:
Inabilty to chew food due to injury to the mandibular division of the trigeminal nerve (CN V)
Loss of taste in the anterior part of the tongue due to injury to the chorda tympani nerve
Mastoiditis
Paralysis of facial muscles due an injury to the facial nerve (CN VII)
Some degree of deafness due to damage to the ossicles
The correct answer is: Inability to chew food due to injury to the mandibular division of the trigeminal nerve (CN V3)
The mandibular division of the trigeminal nerve is not associated with the middle ear. So, chronic ear infections should have no effect on this structure. All of the other answers refer to structures that are closely associated with the middle ear--a chronic infection in this area could cause any of the other listed complications. The chorda tympani nerve travels along the lateral wall of the middle ear, running across the tympanic membrane. It could be damaged by chronic infection and inflammation. Remember--chorda tympani provides secretomotor innervation to the submandibular and sublingual glands and taste sensation to the anterior 2/3 of the tongue. Mastoiditis is an infection of the mastoid air cells. Since these air cells connect to the middle ear through the aditus ad antrum on the posterior wall, an infection in the middle ear could easily spread to the mastoid air cells. The facial nerve is also located on the posterior wall of the middle ear, so it could also be damaged by the chronic infection. Finally, a chronic infection can damage the three ossicles and lead to deafness.
A patient with a facial nerve paralysis suffers from inability to dampen loud noises (hyperacusis) due to denervation of which muscle?
Posterior belly of digastric
Stapedius
Tensor tympani
Stylohyoid muscle
The correct answer is: Stapedius
Stapedius is a small muscle in the ear innervated by the facial nerve. It dampens large vibrations of the stapes and the tympanic membrane; this allows the ear to diminish loud noises. If the facial nerve is paralyzed (as seen with Bell's palsy), the nerve to stapedius is lost, and the ear cannot lessen the vibrations of stapedius. This causes hyperacusis.

The digastric muscle elevates the hyoid bone and depresses the mandible. Its posterior belly is innervated by the facial nerve, so this muscle would be paralyzed if the facial nerve was damaged. However, the posterior belly of the digastric is not involved with the ear. Tensor tympani is a muscle in the ear with a similar function to stapedius--it dampens vibrations of the tympanic membrane. This muscle is innervated by the mandibular branch of the trigeminal nerve (V3)--not the facial nerve. Stylohyoid elevates and retracts the hyoid bone. It's innervated by the facial nerve, but it does not have any effect on the ear.
All of the following are true about the middle ear EXCEPT:
The joints between ossicles are synovial
The chorda tympani nerve is related to the lateral wall
The facial nerve passes in a canal situated in the medial and anterior walls
The auditory tube connects the nasopharynx with the anterior wall
Its mucous membrane is supplied by the glossopharyngeal nerve (CN IX)
The correct answer is: The facial nerve passes in a canal situated in the medial and anterior wall.
The facial nerve passes in a canal situated in the posterior and medial walls of the middle ear. It is not associated with the anterior wall. The other 4 statements are true. The ossicles articulate with each other at synovial joints. The chorda tympani is a branch of the facial nerve that is related to the lateral wall of the middle ear, near the tympanic membrane. It passes between the malleus and incus. The auditory tube is found in the anterior wall--it is a path of communication between the nasal portion of the pharynx and the tympanic cavity that allows pressure to equalize on either side of the tympanic membrane. Finally, the mucous membrane of the middle ear is supplied by branches of the tympanic plexus, which is formed by the tympanic branch of the glossopharyngeal nerve.
A patient has sustained a fracture to the base of the skull. Thorough examination concluded that the right greater petrosal nerve, among other structures, has been injured. This conclusion was based on which of the patient's signs:
Partial dryness of the mouth due to lack of salivary secretions from the submandibular and sublingual glands
Partial dryness of the mouth due to lack of salivary secretions from the parotid gland
Dryness of the right cornea due to lack of lacrimal secretion
Loss of taste sensation from the right anterior 2/3rd of the tongue
Loss of general sensation from the right anterior 2/3rd of the tongue
The correct answer is: Dryness of the right cornea due to lack of lacrimal secretion.
The greater petrosal nerve is a branch of the facial nerve that arises adjacent to the geniculate ganglion. It carries preganglionic parasympathetic fibers to the pterygopalatine ganglion. The fibers synapse in this ganglion, and the postsynaptic parasympathetic fibers go on to innervate the lacrimal gland, mucous glands of the palate, and the mucous glands of the nasal cavity. So, if the greater petrosal nerve was damaged, the presynaptic parasympathetic fibers would be lost, and the lacrimal gland would not function properly.

The sublingual and submandibular glands are innervated by the chorda tympani, which brings preganglionic parasympathetic fibers to the submandibular ganglion. The chorda tympani also provides taste sensation to the anterior 2/3 of the tongue. Although the chorda tympani, like the greater petrosal nerve, is a branch of the facial nerve, the chorda tympani never runs with the greater petrosal nerve. So, injuring the greater petrosal nerve would not harm the chorda tympani. The parotid gland is innervated by the lesser petrosal nerve, a branch of the glossopharyngeal nerve (CN IX). The fibers from this nerve go to the otic ganglion, synapse there, and then continue on to the parotid gland via the auriculotemporal nerve. Finally, general sensation to the anterior 2/3 of the tongue is carried by the lingual nerve, a branch of V3.
The location of the otic ganglion is in the:
Pterygopalatine fossa
Internal ear
Infratemporal fossa
Middle cranial fossa
None of the above
The correct answer is: Infratemporal fossa
The otic ganglion is in the infratemporal fossa, just inferior to the foramen ovale, medial to the mandibular nerve and posterior to the medial pterygoid muscle. It is the ganglion where fibers from the lesser petrosal nerve synape. The postsynaptic parasympathetic fibers from the otic ganglion are secretory to the parotid gland--they reach the parotid gland by the auriculotemporal nerve. The pterygopalatine fossa is a small pyramidal space inferior to the apex of the orbit. It lies between the pterygoid process of the sphenoid bone posteriorly and the posterior aspect of the maxilla anteriorly. It contains the terminal part of the maxillary artery, the maxillary nerve, and the pterygopalatine ganglion. The internal ear is buried in the petrous part of the temporal bone--it contains the vestibulocochlear organ which allows for the reception of sound and maintaining balance.

Finally, the middle cranial fossa is the large depression in the cranial base, formed by the greater wings of the sphenoid and squamous parts of the temporal bones laterally and the petrous parts of the temporal bones posteriorly. It contains four important foramina: the superior orbital fissure, which transmits the ophthalmic veins and the nerves entering the orbit (CN III, CN IV, CN V1 and CN VI); the foramen rotundum which transmits CN V2; the foramen ovale, which transmits CN V3, and the foramen spinosum, which transmits the middle meningeal vessels.
A patient complains of loss of hearing in the right ear. Examination reveals ankylosis (otosclerosis) of the footplate of the stapes to the surrounding bone. Which part of the bony labyrinth is involved?
Aditus ad antrum
Cochlear (round) window
Cochlear duct
Internal acoustic meatus
Vestibular (oval) window
The correct answer is: Vestibular (Oval) Window
The oval window is a fossa in the medial wall of the middle ear--it is found just above the promontory. It leads into the vestibule of the bony labyrinth. In life, this fossa is completely covered by the footplate of the stapes. So, if there was otosclerosis of the stapes to the surrounding bone, there would be damage to the oval window. The round window is also a fossa in the medial wall of the middle ear, but it is located below the promontory. The round window is covered by a thin membrane.

The aditus ad antrum is an opening on the posterior wall of the middle ear. It is the opening to the mastoid antrum and mastoid air cells. The cochlear duct is the spiral tube suspended in the cochlear canal--it is part of the membranous labyrinth. Finally, the internal acoustic meatus is a foramen in the base of the skull. The facial nerve (CN VII) and the vestibulocochlear nerve (CN VIII) enter the petrous temporal bone through this foramen.
The geniculate ganglion is the sensory ganglion of which nerve:
Facial
Glossopharyngeal
Trigeminal
Vagus
Vestibulocochlear
The correct answer is: Facial nerve
The geniculate ganglion is the sensory ganglion of the facial nerve. These fibers travel as part of the chorda tympani to provide taste sensation to the anterior 2/3 of the tongue. The glossopharyngeal nerve has superior and inferior ganglia, located near the jugular foramen. These ganglia contain the cell bodies for the afferent components of this nerve. The trigeminal nerve has a trigeminal ganglion which contains the cell bodies of neurons composing the sensory root of this nerve. The vagus nerve also has a superior and inferior ganglion, which are located near the ganglia of the glossopharyngeal nerve. The superior ganglion is concerned with the general sensory component of the nerve, and the inferior ganglion is concerned with the visceral sensory component of the nerve. Finally, the vestibulocochlear nerve has a vestibular ganglion, which houses the neurons for balance, and the cochlear ganglion, which houses the neurons concerned with hearing.
Repeated middle ear infections have destroyed the tympanic plexus in the middle ear cavity. The loss of preganglionic parasympathetic fibers that pass through the plexus diminish production of:
Mucus in the nasal cavity
Mucus on the soft palate
Saliva by the parotid gland
Saliva by the submandibular and sublingual glands
Tears by the lacrimal gland
The correct answer is: Saliva by the parotid gland
The tympanic plexus is formed by the tympanic nerve, a branch of the glossopharyngal nerve (CN IX). The tympanic plexus provides sensory innervation to the mucosal lining of the middle ear. The lesser petrosal nerve also comes out of the tympanic plexus. This nerve carries preganglionic parasympathetic fibers from the tympanic plexus--these fibers eventually synapse in the otic ganglion. The postsynaptic fibers that leave the otic ganglion provide parasympathetic secretomotor innervation to the parotid gland. So, if the tympanic plexus was destroyed, the lesser petrosal nerve would be destroyed and the parotid gland would not secrete saliva.

Mucosal secretions in the nasal cavity and the soft palate and tear secretions by the lacrimal gland are all mediated by the postganglionic fibers of the pterygopalatine ganglion. This ganglion receives preganglionic fibers from the greater petrosal nerve, a branch of the facial nerve (CN VII). The submandibular and sublingual glands receive their preganglionic fibers from the chorda tympani--another branch of the facial nerve.
The soft palate is active in all of the following except:
Breathing
Chewing
Coughing
Swallowing
Yawning
The correct answer is: Breathing
The soft palate is the movable posterior 1/3 of the palate, which is suspended from the posterior border of the hard palate. When a person swallows, the soft palate is initially tensed to allow the tongue to press against it, squeezing the bolus of food to the back of the mouth. The soft palate is elevated posteriorly and superiorly against the wall of the pharynx, thereby preventing the passage of food into the nasal cavity. The soft palate functions similarly to prevent the bolus of food from passing into the nasal cavity while chewing and to prevent expectorated material from entering the nasal cavity while coughing. The soft palate also elevates when yawning. The soft palate does not elevate during breathing--if it did rise during breathing, the air inspired through the nose might be blocked from entering the trachea.
Most paranasal sinuses and/or air cells drain, directly or indirectly, into the:
Inferior meatus
Middle meatus
Superior meatus
Nasal vestibule
Sphenoethmoidal recess
The correct answer is: middle meatus
The middle meatus contains the semilunar hiatus, which receives drainage from the frontonasal duct (draining the frontal sinus), the anterior ethmoidal air cells, and the maxillary sinus. So, it's draining most of the paranasal sinuses. The inferior meatus receives the nasolacrimal duct which conducts tears from the orbit. The superior meatus receives drainage from the posterior ethmoidal air cells. The nasal vestibule is the opening of the nose--none of the sinuses drain directly into this area. The sphenoethmoidal recess is the opening for the sphenoethmoidal sinus.
Irrigation of the maxillary sinus through its opening is a supportive measure to accelerate the resolution of a maxillary sinus infection. Which of the following nasal spaces is the most likely approach to the sinus opening?
choana
inferior meatus
middle meatus
sphenoethmoidal recess
superior meatus
The correct answer is: middle meatus
The middle meatus contains the semilunar hiatus, which receives drainage from the maxillary sinus, the frontonasal duct (draining the frontal sinus), and the anterior ethmoidal air cells. The maxillary sinus is draining into the middle meatus, so it would be easiest to approach the maxillary sinus through this space. The choana is the space where the nasal cavity opens into the nasopharynx--it is found at the very posterior border of the nasal cavity. The inferior meatus receives the nasolacrimal duct which conducts tears from the orbit. The sphenoethmoidal recess is the opening for the sphenoethmoidal sinus. Finally, the superior meatus receives drainage from the posterior ethmoidal air cells.
In assessing a deep laceration of the right side of the nose, the attending physician determines that the cartilage on the lateral side of the nostril has been cut. What cartilage was injured?
Accessory
Alar
Lateral
Septal
None of the above
The correct answer is: alar cartilage
The alar cartilage is found on the inferolateral side of the nose. This is probably the cartilage that was injured in this laceration. The septal cartilage creates the septum of the nose. The lateral cartilage is a specialization of the septal cartilage that is superior to the alar cartilage, also on the lateral side of the nose. The accessory cartilage is a small piece of cartilage between the alar cartilage and the lateral cartilage
The nasolacrimal duct empties into which part of the nasal cavity?
Hiatus semilunaris
Inferior meatus
Middle meatus
Sphenoethmoidal recess
Vestibule
The correct answer is: inferior meatus
The inferior meatus receives the nasolacrimal duct which conducts tears from the orbit. The semilunar hiatus is found in the middle meatus--it receives drainage from the frontonasal duct (draining the frontal sinus), the anterior ethmoidal air cells, and the maxillary sinus. The sphenoethmoid recess is the opening for the sphenoethmoidal sinus. The vestibule of the nose is the opening of the nose that is covered with skin and stiff hairs.
The pharyngeal tonsils, or adenoids, may become inflamed and in serious cases need to be removed. Where would the physician search for them?
In the tonsillar fossa
In the pharyngeal recess
In the piriform recess
In the roof of the nasopharynx
Upon the dorsal surface of the tongue
The correct answer is: In the roof of the nasopharynx
The pharyngeal tonsil is located on the roof of the nasopharynx. The pharyngeal recess is a space located posterior to the torus tubarius in nasopharynx. The tonsillar fossa is the space where the palatine tonsil is located--it is found between the palatoglossal fold and the palatopharyngeal fold. The piriform recess is a shallow depression located lateral to the aryepiglottic fold in the laryngopharynx. This is a common place where food can get stuck. The piriform recess is also significant because the internal branch of the superior laryngeal nerve is located immediately deep to the mucosa of this region--so, an injury here may damage this nerve! Finally, the lingual tonsil is found on the dorsal surface of the tongue.
The middle nasal concha is part of what bone?
Ethmoid bone
Maxilla
Palatine bone
Sphenoid bone
Vomer
The correct answer is: ethmoid
The superior and middle nasal conchae are part of the ethmoid bone. The maxillary bone is the bone that forms the midface. It forms the inferior orbital margin and contains the teeth and maxillary sinus. The palatine bone forms the posterior part of the hard palate. The sphenoid bone is an irregularly shaped bone forming the central portion of the skull. It has many parts, including a body, greater wing, lesser wing and pterygoid plates. The vomer is a thin plate of bone forming the posteroinferior part of the nasal septum.
A 23-year-old man comes to you complaining that he can't stop crying, i.e. tears regularly run down the right side of his face. You suspect that one of the lacrimal ducts on the right side of the face is blocked. You look into an endoscope to see if the nasolacrimal duct is blocked. Into which part of the nasal cavity would you look to see the opening of the duct?
Hiatus semilunaris
Inferior meatus
Middle meatus
Sphenoethmoidal recess
Superior meatus
The correct answer is: Inferior meatus
The inferior meatus receives the nasolacrimal duct which conducts tears from the orbit. So, this is the place where you should be looking to see the opening of the nasolacrimal duct. The semilunar hiatus is found in the middle meatus--it receives drainage from the frontonasal duct (draining the frontal sinus), the anterior ethmoidal air cells, and the maxillary sinus. The sphenoethmoid recess is the opening for the sphenoethmoidal sinus. The superior meatus is the opening for the posterior ethmoidal air cells.
Which structure forms a border of the tonsillar fossa?
Torus tubarius
Palatopharyngeal fold
Salpingopharyngeal fold
Lateral glossoepiglottic fold
Medial pterygoid plate
The correct answer is: Palatopharyngeal fold
The tonsillar fossa is the place where you find the palatine tonsil - it is a space found between the palatoglossal and palatopharyngeal folds. Torus tubaris is a mucosal fold covering the anteromedial end of the auditory tube cartilage - it projects toward the midline from the lateral wall of the nasopharynx. The salpingopharyngeal fold is a vertical fold of mucous membrane that extends from the medial end of the auditory tube - it covers the salpingopharyngeus muscle. The lateral glossoepiglottic fold is a small fold extending from the tongue to the epiglottis - it is found posterior to the tonsillar fossa. Finally, the medial pterygoid plate is a part of the sphenoid bone in the skull - it is the attachment of the superior pharyngeal constrictor muscle and the pharyngobasilar fascia.
The communication between the pharynx and the nasal cavity is known as the:
Aditus
Auditory tube
Choanae
Fauces
Piriform recess
The correct answer is: choanae
The choanae are the opening at the posterior border of the nasal cavity that allow the nasal cavity to communicate with the nasopharynx. The aditus is the laryngeal inlet - this is the space that is covered by the epiglottis when swallowing. The auditory tube is a tube that connects the nasopharynx with the middle ear, allowing for pressure to equalize on both sides of the tympanic membrane. The fauces is the passage from the mouth to the oropharynx, including the lumen and its boundaries. Finally, the piriform recess is a shallow depression located lateral to the aryepiglottic fold in the laryngopharynx. This is a place where food is commonly lodged.
To drain an abscess (a closed collection of pus) affecting the cheek area, an emergency room physician used local anesthesia for the surgery. Which of the following nerves must be anesthetized because it carries pain sensation from the cheek area?
buccal (V3)
buccal (VII)
inferior alveolar
lingual
mental
The correct answer is: Buccal (V3)
The buccal nerve is a branch of the mandibular division of the trigeminal nerve that transmits sensory information from the skin of the cheek area and the oral mucosa of the cheek. The buccal nerve is NOT the same as the buccal branches of the facial nerve, which are motor nerves that innervate the buccinator and the facial muscles of the upper lip. The buccal branches of the facial nerve do not have a sensory component!

The inferior alveolar nerve is another branch of V3 that penetrates the mandibular foramen and is the sensory nerve for the mandible and all mandibular teeth. This is the nerve that dentists must anesthesize when they are working on the mandibular teeth. The lingual nerve is also a branch of V3--it transmits general sensation from the tongue. The lingual nerve also receives the chorda tympani, a branch of the facial nerve, which provides the lingual nerve with preganglionic parasympathetic fibers for the submandibular and and sublingual glands, as well as the taste fibers for the anterior 2/3 of the tongue. Finally, the mental nerve is a branch of V3 from the inferior alveolar nerve--it provides sensory innervation to the skin of the chin and lower lip.
A patient who experienced bilateral anterior dislocation of the jaw (temporomandibular joints) could not swallow or talk since the mouth was held open. What is the position of the condyles of the mandible as a result of the dislocation?
against the external acoustic meatus
against the anterior slope of the articular eminence
against the posterior slope of the articular eminence
in the mandibular fossa
in the pterygoid fossa
The correct answer is: against the anterior slope of the articular eminence
The TMJ may dislocate anteriorly due to excessive contraction of the lateral pterygoids. This could happen if someone yawned too much or took a large bite of food. So, you want to look for the answer here that represents anterior movement of the jaw. Answer B, against the anterior slope of the articular eminence, is anterior to the normal position of the mandible, so that's the correct answer.

The external acoustic meatus is posterior to the normal position of the mandible, so you know that the mandible would not touch this structure if it dislocated anteriorly. In its normal position, the head of the mandible is near the posterior slope of the articular eminence and in the mandibular fossa, so these answers are not correct. Finally, the pterygoid fossa (also known as pterygoid fovea) is a fossa found on the mandible itself; it's not a place where the mandible could move to dislocate. For a better understanding of this, see Netter Plate 10 and 11.
In reducing an ankylosis of the TMJ, a surgeon provoked an intense hemorrhage by lacerating the artery coursing transversely just medial to the neck of the condyle. Which artery was involved in the accident?
Buccal
External carotid
Maxillary
Middle meningeal
Superficial temporal
The correct answer is: Maxillary
The maxillary artery is one of the terminal branches of the external carotid artery which is closely related with the TMJ. It travels medial to the neck of the condyle, so it would be the artery damaged in this scenario. The buccal artery is a branch of the maxillary artery that travels in the cheek. It supplies blood to the cheek mucosa and skin. The external carotid artery is the source of the maxillary artery, but it is not the artery lying medial to the neck of the mandibular condyle. The middle meningeal artery is a branch of the maxillary artery which is deep to the lateral pterygoid muscle. It supplies the calvaria and the dura surrounding the brain. Finally, the superficial temporal artery is the other terminal branch of the external carotid artery. It courses posterior and lateral to the head of the mandible to supply the scalp of the lateral side of the head and lateral face. See Netter Plate 35 for a picture of all of these arteries and their interconnections.
The muscle which separates the submandibular triangle from the paralingual space is the:
Digastric, posterior belly
Hyoglossus
Mylohyoid
Stylohyoid
Styloglossus
The correct answer is: mylohyoid
The submandibular triangle is a space bounded by the anterior and posterior bellies of the digastric and the body of the mandible. It is found superficial to the mylohyoid muscle. The submandibular triangle contains the superficial submandibular gland, stylohyoid muscle, facial artery and facial vein.

The paralingual space is a space found deep to the mylohyoid muscle, which makes the mylohyoid the dividing line between these two distinct spaces. It is bounded by the lateral tongue (including the hyoglossus, styloglossus, and genioglossus muscles), hyoid bone, and oral mucosa. It contains the deep portion of the submandibular gland, the submandibular duct and the sublingual caruncle, lingual nerve and submandibular ganglion, hypoglossal nerve, and sublingual gland and fold.
The predominant muscle most associated with retraction of the mandible is the:
lateral pterygoid
masseter
medial pterygoid
temporalis
mylohyoid
The correct answer is: temporalis
Temporalis is the one muscle responsible for retracting the mandible--it pulls the mandible backwards. The lateral pterygoid protracts the mandible, or pulls it forward. It is the one jaw muscle that allows for opening the mouth. The masseter is a powerful chewing muscle that elevates the mandible. The medial pterygoid also elevates the mandible; it has a similar position and action to the masseter, but the ramus of the mandible separates the two muscles. Mylohyoid elevates the hyoid bone and the tongue and depresses the mandible.
At the temporomandibular joint (TMJ), hinge movements occur between the:
condyle and articular eminence
articular disc and articular eminence
condyle and articular disc
articular disc and articular cavity
condyle and articular cavity
The correct answer is: condyle and articular disc
The TMJ joint is a synovial joint with two articular cavities. Each cavity is responsible for a different movement at the joint. An articular disc sits between the condylar process of the mandible on its inferior side and the mandibular fossa and articular eminence of the temporal bone on the superior side. This disc divides the joint into the two articular cavities, with one cavity acting as a hinge component and the other cavity serving as a gliding component. The lower part of the joint, between the condyle and the articular disc, is the hinge component of the joint. When the joint moves, this hinge component of the joint moves first, to initiate mandibular opening. The upper part of the joint, between the articular disc and the mandibular fossa and articular eminence of the temporal bone, creates the gliding component. During joint movement, this gliding cavity moves after the hinge component to terminate mandibular opening.
The correct answer is: condyle and articular disc
The TMJ joint is a synovial joint with two articular cavities. Each cavity is responsible for a different movement at the joint. An articular disc sits between the condylar process of the mandible on its inferior side and the mandibular fossa and articular eminence of the temporal bone on the superior side. This disc divides the joint into the two articular cavities, with one cavity acting as a hinge component and the other cavity serving as a gliding component. The lower part of the joint, between the condyle and the articular disc, is the hinge component of the joint. When the joint moves, this hinge component of the joint moves first, to initiate mandibular opening. The upper part of the joint, between the articular disc and the mandibular fossa and articular eminence of the temporal bone, creates the gliding component. During joint movement, this gliding cavity moves after the hinge component to terminate mandibular opening.
The correct answer is: Inferior alveolar
The inferior alveolar nerve is a branch of the mandibular division of the trigeminal nerve (V3). It penetrates the mandibular foramen and is the sensory nerve for the mandible and all mandibular teeth. This is the nerve anesthetized by dentists working on the mandibular teeth. It is anesthetized near the mandibular foramen, so sensory nerves from branches distal to that point would be blocked. The lingual nerve is another branch of V3--it transmits general sensation from the tongue. The lingual nerve also receives the chorda tympani, a branch of the facial nerve. The chorda tympani gives the lingual nerve preganglionic parasympathetic fibers for the submandibular and sublingual glands, as well as the taste fibers for the anterior 2/3 of the tongue. The buccal nerve is a branch of V3 that transmits sensory information from the skin of the cheek area and the oral mucosa of the cheek. Remember: The buccal nerve is NOT the same as the buccal branch of the facial nerve, which is a motor nerve innervating the buccinator and muscles of the upper lip!!!

The mental nerve is a branch of V3 from the inferior alveolar nerve--it provides sensory innervation to the skin of the chin and lower lip. The nerve to mylohyoid is also a branch of V3--it innervates the mylohyoid muscle and the anterior belly of the digastric. All of these branches of the inferior alveolar nerve could end up being anesthetized as a result of the dental work, but these nerves are not the dentist's main target!
Incapacity to protrude the mandible indicates a dysfunction of which muscle?
Anterior belly of digastric
Buccinator
Lateral pterygoid
Mylohyoid
Temporalis
The correct answer is: Lateral pterygoid
The lateral pterygoid muscle protrudes the mandible--it pulls the mandible forward to allow for depression of the chin (which is mostly produced by gravity). None of the other muscles help with this function. The anterior belly of the digastric and mylohyoid have similar functions: they both help elevate the hyoid bone and depress the mandible. The buccinator is a muscle in the cheek; it pulls the corner of mouth laterally and presses the cheek against the teeth. Temporalis is important for retracting and elevating the mandible.
Damage to the facial nerve near the stylomastoid foramen would likely cause each of the following motor deficits EXCEPT:
Paralysis of the buccinator muscle
Inability to whistle
Paralysis of the muscles that elevate the mandible
Inability to close the lips
The correct answer is: paralysis of the muscles which elevate the mandible
The masseter, medial pterygoid, and temporalis are all muscles that elevate the mandible. These muscles are innervated by branches of V3, the mandibular division of the trigeminal nerve. So, none of these muscles would be denervated if the facial nerve was injured. The other actions discussed all involve the muscles of facial expression, which are innervated by the facial nerve (CN VII). The buccinator muscle is innervated by the buccal branches of the facial nerve--this muscle pulls the corner of mouth laterally and presses the cheek against the teeth. The orbicularis oris is the muscle that allows for whistling or closing the lips; this muscle is innervated by the buccal branches of the facial nerve.
The lesser petrosal nerve carries preganglionic parasympathetic fibers to the:
Geniculate ganglion
Otic ganglion
Submandibular ganglion
Ciliary ganglion
The correct answer is: otic ganglion
The lesser petrosal nerve is a branch of the glossopharyngeal nerve (CN IX). The lesser petrosal nerve carries presynaptic parasympathetic fibers to the otic ganglion. These fibers synapse in the otic ganglion, and the postsynaptic fibers travel on the auriculotemporal nerve to innervate the parotid gland. The geniculate ganglion is the sensory ganglion of the facial nerve--it holds the cell bodies of the neurons that carry taste sensations from the anterior 2/3 of the tongue. The submandibular ganglion receives preganglionic parasympathetic fibers from the chorda tympani; the postsynaptic fibers from the submandibular ganglion go to the sublingual and submandibular glands. Finally, the ciliary ganglion receives preganglionic parasympathetic fibers from the inferior division of the oculomotor nerve. The postganglionic fibers from this ganglion leave via short ciliary nerves and innervate sphincter pupillae and the ciliary muscle of the eye. Also remember that sensory and sympathetic fibers are passing through the ciliary ganglion and distributing on the short ciliary nerves, but these fibers never synapse in that ganglion.
What bony feature of the mandible can be used to find and palpate the facial artery?
Oblique line
Mental trigone
Angle
Premasseteric notch
The correct answer is: Premasseteric notch
The facial artery arises from the external carotid artery and winds around the inferior border of the mandible deep to the platysma, immediately anterior to the insertion of masseter muscle. This is the area of the premasseteric notch. The notch lies anterior to the angle of the mandible, so angle would not be a landmark to find the facial artery.
Which of the following suprahyoid muscles would be paralyzed if the inferior alveolar nerve were severed at its origin?
Geniohyoid m.
Hyoglossus m.
Mylohyoid m.
Stylohyoid m.
The correct answer is: Mylohyoid
Mylohyoid is innervated by the nerve to mylohyoid, which is a branch of the inferior alveolar nerve. The inferior alveolar nerve is a branch of the mandibular division of the trigeminal nerve. Geniohyoid is a muscle that spans from the mental spines of the mandible to the body of the hyoid bone--it elevates the hyoid and depresses the mandible. It is innervated by the ventral primary ramus of spinal nerve C1 via fibers carried by the hypoglossal nerve. Hyoglossus is a tongue muscle; it is innervated by the hypoglossal nerve (CN XII). Finally, stylohyoid is a muscle that spans from the styloid process to the hyoid bone. It elevates and retracts the hyoid, and it is innervated by the facial nerve.
Which nerve is endangered during surgical removal of an impacted third mandibular molar tooth?
Hypoglossal n.
Glossopharyngeal n.
Inferior alveolar n.
Lingual n.
The correct answer is: inferior alveolar nerve
The inferior alveolar nerve provides sensory innervation to the mandibular teeth. This nerve runs in the mandibular foramen, near the roots of the teeth. So, it might be endangered when removing an impacted tooth. The hypoglossal nerve travels lateral to the carotid vessels and through the floor of the mouth. The glossopharyngeal nerve consists of pharyngeal branches that provide sensory innervation to the upper pharynx and lingual branches that sweep around the stylopharyngeus muscle and continue to the base of the tongue. Finally, the lingual nerve, which is from the mandibular division of the trigeminal nerve, is found in the floor of the mouth. These three nerves are far from the area of the impacted tooth, so they would not be endangered by the dental procedure.
After the mandibular condyle is moved forward onto the articular eminence (e.g., by opening the mouth widely), what muscle can then retract the mandible?
Superficial head of masseter m.
Deep head of masseter m.
Posterior part of temporalis m.
Anterior part of temporalis m.
The correct answer is: posterior part of temporalis
The fibers of the posterior part of temporalis retract the mandible; the fibers from the anterior part of temporalis elevate the mandible. The anterior and deep heads of the masseter muscle are both important for elevating the mandible--remember, the masseter is the very powerful chewing muscle!
Two nerves usually emerge from between the two heads of the lateral pterygoid muscle: the anterior deep temporal nerve and the:
Masseteric n.
Buccal n.
Lingual n.
Inferior alveolar n.
The correct answer is: buccal
The lateral pterygoid muscle has 2 heads: The superior head of the muscle inserts into the disc and capsule of the temporomandibular joint while the inferior head inserts into the neck of the mandible (the pterygoid fovea). There are two nerves coming between the heads of the lateral pterygoid muscle: the anterior deep temporal nerve and the buccal nerve. See Netter Plate 42A for a picture of these nerves. The masseteric nerve courses superior to the 2 heads of the lateral pterygoid, not between the 2 heads. The lingual nerve travels inferior to the two heads of the muscle. The inferior alveolar nerve also travels inferior to the two heads of the lateral pterygoid before it enters the mandibular foramen.
The chorda tympani enters the infratemporal fossa after it exits the:
Stylomastoid foramen
Foramen spinosum
Foramen lacerum
Petrotympanic fissure
The correct answer is: petrotympanic fissure
The chorda tympani exits the skull through the petrotympanic fissure. The stylomastoid foramen is the space that the facial nerve travels through to leave the skull--remember, the chorda tympani has already separated away from the facial nerve by this point. Foramen spinosum is a hole in the base of the skull that transmits the middle meningeal artery and vein. Finally, foramen lacerum is a ragged foramen that is an artifact of a dried skull. In life, it is closed by cartilage. Nothing passes directly through foramen lacerum, although both greater and deep petrosal nerve pass through some of the cartilage that fills it, in order to enter the pterygoid canal at the anterior margin of foramen lacerum.
Paralysis of which of the following muscles would impede retraction of the mandible?
Buccinator
Lateral pterygoid, lower portion
Lateral pterygoid, upper (sphenomeniscus) portion
Medial pterygoid
Temporalis
The correct answer is: Temporalis
Temporalis is the important muscle for retracting the mandible! Buccinator is a muscle on the side of the face which pulls the corner of mouth laterally and presses the cheek against the teeth. Both portions of the lateral pterygoid protract the mandible and open the mandible. (Remember: lateral pterygoid is the only muscle that opens the mandible!) Finally, the medial pterygoid protracts and elevates the mandible.
A cranial fracture through the foramen ovale that compresses the enclosed nerve, will have an effect on all muscles EXCEPT :
Tensor tympani
Masseter
Buccinator
Mylohyoid
Temporalis
The correct answer is: buccinator
The mandibular division of the trigeminal nerve (V3) is transmitted through foramen ovale. Branches from this nerve innervate the muscles of mastication, like temporalis and masseter. A branch from V3 also innervates tensor tympani, which is a muscle that dampens the vibrations of the tympanic membrane. Mylohyoid is innervated by a branch of the inferior alveolar nerve which is also from V3--this muscle elevates the hyoid bone and the tongue and depresses the mandible.

Buccinator is a muscle on the side of the cheek which is innervated by the facial nerve (VII). This muscle, which pulls the corner of mouth laterally and presses the cheek against the teeth, would still be functional even after disrupting the mandibular division of the trigeminal nerve.
Which muscle is also known as the sphenomeniscus?
Inferior head of the lateral pterygoid
Masseter
Medial pterygoid
Superior head of the lateral pterygoid
Temporalis
The correct answer is: Superior head of the lateral pterygoid
The superior head of lateral pterygoid is sometimes called sphenomeniscus due to its insertion into the disc of the temporomandibular joint. Remember, the superior head of the lateral pterygoid inserts into the disc of the TMJ, while the inferior head of the lateral pterygoid inserts into the neck of the mandible (pterygoid fovea). None of the other listed muscles insert into a joint like the superior head of the lateral pterygoid.
Forward movement of the condyle of the mandible during wide opening of the jaws is accomplished mainly by the:
anterior part of temporalis muscle
lateral pterygoid muscle
masseter muscle
medial pterygoid muscle
posterior part of the temporalis muscle
The correct answer is: lateral pterygoid muscle
Remember--the lateral pterygoid muscle is the one muscle of mastication that is responsible for opening the jaw! It also draws the mandible forward. The anterior part of the temporalis muscle elevates the mandible, while the posterior part of temporalis retracts the mandible. Masseter is a powerful chewing muscle that elevates the mandible. The medial pterygoid muscle elevates the mandible and can help to move the mandible forward. But, when the mouth is open, the lateral pterygoid will be the most important muscle for drawing the mandible forward.
To drill a mandibular tooth without causing undue pain, a dentist has injected an anesthetic into the space located between the medial pterygoid muscle and the mandible near the lingula. Given the nerves passing through the immediate vicinity of the injection site, where would one expect anesthesia in addition to the mandibular teeth?
back of tongue
external ear
maxillary incisor teeth
skin of chin
upper lip
The correct answer is: Skin of chin
The dentist would need to numb the inferior alveolar nerve, which is the nerve that runs in the mandibular foramen and provides sensory innervation to the teeth. The inferior alveolar nerve also gives off the mental nerve, which is a sensory nerve innervating the skin of the chin. So, if the inferior alveolar nerve was anesthetized, the mental nerve would be anesthetized, too. Then, the patient would lose sensory innervation to the skin of the chin.

The back of the tongue (posterior 1/3) receives sensory innervation from the glossopharyngeal nerve. This nerve would not be exposed to the anesthetic. The external ear is innervated by the auriculotemporal nerve, which is not near the area of anesthesia. The maxillary incisor teeth are innervated by the superior alveolar nerves, which are branches of the maxillary division of the trigeminal nerve (V2). The skin of the upper lip is also innervated by a branch of V2--the infraorbital nerve. These branches of V2 would not be exposed to the anesthesia.

If too much anesthesic was injected or if the parotid fascia was pierced by the needle, it would be possible for the anesthesic to diffuse through the soft tissue and paralyze the facial nerve. A high dose of anesthesic might also diffuse to paralyze the muscles of mastication.
The middle meningeal artery:
enters the skull through the foramen ovale
passes through a split in the trunk of the mandibular nerve (V3)
is typically a branch of the second part of the maxillary artery
supplies blood to the temporal lobe of the brain
usually arises deep to the neck of the mandible
The correct answer is: usually arises deep to the neck of the mandible
The middle meningeal artery is always found deep to the lateral pterygoid muscle, so it is arising deep to the neck of the mandible. It enters the skull by passing through the foramen spinosum, not through foramen ovale. Remember--foramen ovale transmits the mandibular branch of trigeminal. Although the middle meningeal artery appears to pass through a fork in the auriculotemporal nerve, it does not pass through a split in the trunk of V3. The middle meningeal artery is not a branch of the second part of the maxillary artery--it is a very early branch that separates from the maxillary artery almost immediately. It supplies blood to the dura mater and the bones of the cranial vault, but it does not supply the temporal lobe of the brain. The temporal lobe is supplied by the middle cerebral artery.
The surgical removal of a metastatic tumor in the infratemporal fossa caused an intense hemorrhage. The surgeon clamped the main source of arterial supply to the area, which is the:
Internal carotid
Lingual
Maxillary
Posterior auricular
Superficial temporal
The correct answer is: Maxillary artery
The maxillary artery is one of the terminal branches of the external carotid artery. It enters the infratemporal fossa and is the major source of blood for that region. The internal carotid artery is the primary blood supply to the brain. The lingual artery is another branch of the external carotid artery--it supplies blood to the tongue and the floor of the mouth. The posterior auricular is a branch of the external carotid artery that supplies the external ear, scalp and the deeper structures posterior to the ear. The superficial temporal artery is the other terminal branch of the external carotid artery -- it supplies blood to the scalp of the lateral side of the head and the lateral face.
In acute inflammation (arthritis) of the TMJ, the muscle most likely to be affected by the inflammatory process is the:
Temporal
Medial pterygoid
Masseter
Lateral pterygoid
The correct answer is: lateral pterygoid
The inferior head of the lateral pterygoid inserts into the neck of the mandible, while the superior head of the lateral pterygoid inserts directly into the capsule and articular disk of the temporomandibular joint. So, this muscle would be affected by arthritis of the joint. This muscle is the only one that opens the mandible, so this movement might be weakened if there was inflammation at the TMJ.

Temporalis elevates and retracts the mandible; it inserts on the coronoid process of the mandible and the anterior surface of the ramus of the mandible. The medial pterygoid muscle protracts and elevates the mandible; it inserts on the medial surface of the ramus and angle of the mandible. Finally, the masseter is the muscle that powerfully elevates the mandible--it inserts on the lower half of the ramus of the mandible. None of these other muscles are inserting into the TMJ, so they would not be impaired quite as much as the lateral pterygoid.
In explaining the pain caused by the drilling of a mandibular molar tooth crown to a freshman medical student, a dentist identified the nerve conducting the pain sensations as the:
Lingual
Mylohyoid
Inferior alveolar
Buccal
The correct answer is: Inferior alveolar
The inferior alveolar nerve is a branch of the mandibular division of the trigeminal nerve (V3). It travels through the mandibular foramen and provides sensory innervation to the mandibular teeth. The lingual nerve is another branch of V3--it travels in the floor of the mouth and provides sensory innervation to the anterior 2/3 of the tongue and the floor of the mouth. The nerve to mylohyoid is a motor branch of V3--it provides motor innervation to mylohyoid and the anterior belly of the digastric. Finally, the buccal nerve is a sensory branch of V3 that supplies the cheek and oral mucosa.
The temporomandibular joint is characterized by all EXCEPT:
A capsule strengthened by ligaments on its lateral side only
A completely flat surface for its gliding action
An articular disc
Extracapsular ligaments
Two joint cavities of different shapes
The correct answer is: A completely flat surface for its gliding action
The TMJ joint is a synovial joint with two articular cavities. Each cavity is responsible for a different movement at the joint. The lower part of the joint is the hinge component of the joint. When the joint moves, this hinge component of the joint initiates mandibular opening. The upper part of the joint is the gliding component. During joint movement, this gliding cavity moves to terminate mandibular opening. The gliding cavity is the space between the articular disc and the mandibular fossa and articular eminence of the temporal bone--it's not a completely flat surface.

There are extracapsular ligaments around the TMJ joint capsule, but these ligaments are on the lateral side only. The lateral ligament reinforces the lateral part of the capsule, while other ligaments (the stylomandibular and sphenomandibular) only have a minor role in stabilizing the joint capsule. There is also an articular disc dividing the two components of the joint, and the two cavities are different shapes. See Netter Plate 11 for a picture of all of these structures.
In dislocation of the jaw, displacement of the articular disc beyond the articular tubercle of the temporomandibular joint results from excessive contraction of which muscle?
Buccinator
Lateral pterygoid
Medial pterygoid
Masseter
Temporalis
The correct answer is: Lateral pterygoids
The TMJ may dislocate anteriorly due to excessive contraction of the lateral pterygoids (for example, during excessive opening of the mouth). This could happen if someone yawned too much or took a large bite of food. Posterior dislocations of the TMJ are rare due to resistance from the postglenoid tubercle and the strong lateral ligaments.

The buccinator is an important muscle for mastication because it keeps the cheek taut, so the cheek can press against the molars. However, it is a facial muscle innervated by the facial nerve, and it is not active at the TMJ. The medial pterygoid, masseter and temporalis produce motion at the TMJ, but contracting these muscles does not cause the joint to dislocate.
Sympathetic fibers reach the tongue by way of the:
lingual nerve
maxillary artery
hypoglossal nerve
lingual artery
glossopharyngeal nerve
The correct answer is: Lingual artery
Remember--all over the body, the vasculature is the number one target of sympathetic nerves. Sympathetic nerves help to constrict the vasculature, and they are found covering arteries in periarterial plexuses. In the head, sympathetics travel to targets on vessels. So, you know that the sympathetic nerves are coming to the tongue on an artery--now, you just need to determine what artery goes to the tongue. And that artery is the lingual artery.

The lingual artery is the second branch off the anterior side of the external carotid artery. It travels in the floor of the mouth and supplies blood to the tongue, suprahyoid muscles, and the palatine tonsil. This is the artery going to the targeted area, so that's the answer you're looking for. The maxillary artery is one of the 2 terminal branches of the external carotid artery. It supplies blood to the deep face and infratemporal fossa. It is not going to the tongue.

As far as the nerves go... The lingual nerve is a branch of V3, the mandibular division of the trigeminal nerve. It supplies general sensation (touch and temperature) to the anterior 2/3 of the tongue. (Taste sensation for the anterior 2/3 of the tongue arrives at the tongue by way of the lingual nerve, but the original fibers for taste came from the chorda tympani, a branch of CN VII.) The hypoglossal nerve (CN XII) provides motor innervation to the muscles of the tongue. The glossopharyngeal nerve (CN IX) provides taste sensation and general sensation to the posterior 1/3 of the tongue.
After a radiograph revealed a sialolith (stone) in a patient's right submandibular duct, the surgeon exposed the duct via an intraoral approach. In this approach, what tissues or structures must be cut through?
Mucous membrane only
Mucous membrane and genioglossus muscle
Mucous membrane and mylohyoid muscle
Mucous membrane and hyoglossus muscle
The correct answer is: Mucous membrane only
The submandibular duct is found deep to the mucous membrane of the mouth, but superficial to the muscles of the tongue and the muscles on the floor of the mouth. So, genioglossus, mylohyoid, and hyoglossus are deep to the submandibular duct. See Netter Plate 56B for a picture of these relationships.
Damage of the lingual nerve before it is joined by the chorda tympani in the infratemporal fossa would cause loss of:
general sensation to the anterior two thirds of the tongue
general sensation to the posterior one third of the tongue
secretion of the submandibular gland
taste sensation from the anterior two thirds of the tongue
taste sensation from the posterior one third of the tongue
The correct answer is: General sensation to the anterior 2/3 of the tongue
The lingual nerve is a branch of V3, the mandibular division of the trigeminal nerve. It transmits general sensation from the anterior 2/3 of the tongue. Damaging this nerve anywhere along its course would cause someone to lose general sensation to the anterior 2/3 of the tongue. The chorda tympani is a branch of VII--it carries taste fibers to the anterior 2/3 of the tongue and presynaptic parasympathetic fibers to the submandibular ganglion. The fibers from the chorda tympani join the lingual nerve as they travel to the submandibular ganglion and the anterior tongue. However, if the lingual nerve was damaged before the chorda tympani had joined it, the chorda tympani fibers would still be intact. So, there would still be taste innervation to the anterior 2/3 of the tongue and the submandibular gland would still secrete. If the lingual nerve was disrupted after the chorda tympani had already joined it, there would be no secretomotor innervation to the submandibular and sublingual glands, and there would be no taste or general sensation to the anterior 2/3 of the tongue.

As for the posterior 1/3 of the tongue, it receives taste and general sensation from the glossopharyngeal nerve (CN IX).
The teeth and gums separate the oral cavity proper from the:
Nasal cavity
Oral vestibule
Oropharynx
Paralingual space
Submandibular space
The correct answer is: oral vestibule
The oral vestibule is the space in the mouth lying between the lips and the teeth. See Netter Plate 45 for an illustration of the mouth--although the vestibule isn't labeled, you can get some idea of where it lies. The nasal cavities are spaces that connect the nasopharynx with the external environment. The roof of the nasal cavity is made of the cribriform plate of the ethmoid bone. The floor of the nasal cavity is made of the hard palate. The medial wall is made by the nasal septum, and the lateral wall is the place where the inferior, superior, and middle conchae project into the nasal cavity.

The oropharynx is the middle part of the pharynx, found behind the mouth. It communicates anteriorly with oral cavity through the palatoglossal arch, superiorly with the nasopharynx through the posterior margin of soft palate, and inferiorly with the laryngopharynx at the superior margin of epiglottis. The oropharynx contains the palatine tonsil, which is located between the palatoglossal arch and the palatopharyngeal arch.

The paralingual space is a space inside the floor of the mouth. It is bounded by the mylohyoid muscle, the lateral tongue, the hyoid bone, and the oral mucosa. The paralingual space contains the deep portion of the submandibular gland, the lingual nerve and submandibular ganglia, the sublingual gland and fold, and the hypoglossal nerve.

Finally, the submandibular space is a space defined by the body of the mandible and the anterior and posterior digastric muscles. It contains the superficial submandibular gland, mylohyoid and stylohyoid muscles, the facial artery, and the facial vein.
Description for the following questions: Examination of a patient with an ulcerative carcinoma of the posterior third of the tongue revealed bleeding from the lesion and difficulty swallowing (dysphagia).
The bleeding was seen to be arterial; which of the following arteries was involved?
Deep lingual
Dorsal lingual
Facial
Sublingual
Tonsillar
The correct answer is: Dorsal lingual artery
The dorsal lingual artery runs on the superficial surface of the tongue--it is a branch of the lingual artery that delivers blood to the posterior superficial tongue. So, this artery must be the source of the hemorrhage. The deep lingual artery and sublingual artery are two terminal branches of the lingual artery. These branches run in the floor of the mouth (sublingual) and the deep surface of the tongue (deep lingual). See Netter Plate 53 for a picture. The facial artery is a branch of the external carotid artery that courses across the face. The tonsillar artery is a branch of the facial artery that supplies blood to the palatine tonsil.
Description for the following questions: Examination of a patient with an ulcerative carcinoma of the posterior third of the tongue revealed bleeding from the lesion and difficulty swallowing (dysphagia). The difficulty in swallowing was due to involvement of which muscle that elevates the tongue?
Genioglossus
Hyoglossus
Styloglossus
Stylohyoid
Stylopharyngeus
The correct answer is: Styloglossus
Styloglossus retracts and elevates the tongue. Genioglossus is a large, fan shaped muscle with many actions: its inferior fibers protrude the tongue, its middle fibers depress the tongue, and its superior fibers draw the tip back and down. Hyoglossus retracts and depresses the tongue. All of these muscles are innervated by the hypoglossal nerve (CN XII). Stylohyoid elevates and retracts the hyoid bone. It is innervated by the facial nerve (CN VII). Stylopharyngeus elevates the larynx--it is innervated by the glossopharyngeal nerve (IX).
Description for the following questions: Examination of a patient with an ulcerative carcinoma of the posterior third of the tongue revealed bleeding from the lesion and difficulty swallowing (dysphagia). Cutting of the hypoglossal nerve in the hypoglossal canal would not interrupt the nerve supply to the:
Hyoglossus muscle
Genioglossus muscle
Palatoglossus muscle
Styloglossus muscle
The correct answer is: Palatoglossus muscle
All of the tongue muscles are innervated by the hypoglossal nerve. So, hyoglossus, genioglossus, and styloglossus are all innervated by the hypoglossal nerve. Although palatoglossus sounds like a tongue muscle, it is a palatal muscle, innervated by the vagus nerve. (Palatoglossus is the only "glossus" muscle that is not innervated by the hypoglossal nerve.)
Description for the following questions: Examination of a patient with an ulcerative carcinoma of the posterior third of the tongue revealed bleeding from the lesion and difficulty swallowing (dysphagia). The contents of the paralingual space do NOT include the:
Hypoglossal nerve
Lingual artery
Lingual nerve
Submandibular gland
Sublingual gland
The correct answer is: Lingual artery
The paralingual space is a space inside the floor of the mouth. It is bounded by the mylohyoid muscle, the lateral tongue, the hyoid bone, and the oral mucosa. The paralingual space contains the deep portion of the submandibular gland, the lingual nerve and submandibular ganglion, the sublingual gland and fold, and the hypoglossal nerve. The lingual artery is not in the paralingual space.
Description for the following questions: Examination of a patient with an ulcerative carcinoma of the posterior third of the tongue revealed bleeding from the lesion and difficulty swallowing (dysphagia). A patient is unable to taste a piece of sugar placed on the anterior part of the tongue. Which cranial nerve is most likely to have a lesion?
Facial nerve
Glossopharyngeal nerve
Hypoglossal nerve
Trigeminal nerve
Vagus nerve
The correct answer is: facial nerve
Taste fibers to the anterior 2/3 of the tongue come from the chorda tympani--a branch of the facial nerve. So, if the patient has lost taste sensation to the anterior 2/3 of the tongue, the patient might have a lesion of the facial nerve. The glossopharyngeal nerve (CN IX) carries taste and general sensory fibers to the posterior 1/3 of the tongue. The hypoglossal nerve (CN XII) gives motor innervation to all the muscles of the tongue (ie, all the muscles that end in glossus except for palatoglossus, which is a palate muscle innervated by the vagus). The trigeminal nerve supplies general sensory fibers to the anterior 2/3 of the tongue via the lingual nerve, a branch of V3. Remember, the chorda tympani fibers jump on the lingual nerve to reach the tongue, but the chorda tympani fibers originate from the facial nerve! Finally, the vagus nerve supplies general and taste sensation to a very small part of the posterior tongue, right next to the epiglottis.
The chorda tympani contains which component before it joins the lingual nerve?
Preganglionic sympathetics
Postganglionic sympathetics
Preganglionic parasympathetics
Postganglionic parasympathetics
Taste fibers to the posterior third of the tongue
The correct answer is: preganglionic parasympathetics
The chorda tympani contains preganglionic parasympathetic nerves that go to the submandibular ganglion. These preganglionic parasympathetics synapse in the submandibular ganglion and then provide secretomotor innervation to the submandibular and sublingual glands. The chorda tympani also contains taste fibers to the anterior 2/3 of the tongue, but not to the posterior 1/3.
The cell bodies of the taste fibers from the anterior two-thirds of the tongue are located in the:
Geniculate ganglion
Otic ganglion
Pterygopalatine ganglion
Submandibular ganglion
Trigeminal ganglion
The correct answer is: geniculate ganglion
The geniculate ganglion is the sensory ganglion of the facial nerve--it holds the cell bodies of the neurons that carry taste sensations from the anterior 2/3 of the tongue. These fibers then travel on the chorda tympani, which carries the fibers to the lingual nerve. The chorda tympani fibers then jump on the lingual nerve so they can reach the tongue and provide taste sensation to the anterior 2/3. The otic ganglion is the ganglion where parasympathetic fibers synapse before innervating the parotid gland. The lesser petrosal nerve is a branch of the glossopharyngeal nerve (CN IX) that carries presynaptic parasympathetic fibers to the otic ganglion. These fibers synapse in the otic ganglion, and the postsynaptic fibers travel on the auriculotemporal nerve to the parotid gland.

The pterygopalatine ganglion is the ganglion that enables secretomotor innervation to the mucous glands of the palate, nasal cavity, and lacrimal gland. Preganglionic fibers arrive at this ganglion from the greater petrosal nerve of the facial nerve. The submandibular ganglion receives preganglionic parasympathetic fibers from the chorda tympani; the postsynaptic fibers from the submandibular ganglion go to provide secretomotor innervation to the sublingual and submandibular glands. Finally, the trigeminal ganglia (also called the semilunar ganglia) is a sensory ganglion equivalent in histological structure and function to a dorsal root ganglion. The cell bodies of the afferent fibers of the trigeminal nerve reside in this ganglion.
Which of the following structures is located in the vestibule of the oral cavity?
Tongue
Opening of the parotid duct
Opening of the submandibular duct
Sublingual fold
Uvula
The correct answer is: the opening of the parotid duct
The oral vestibule is the space in the mouth lying between the lips and the teeth. See Netter Plate 45 for an illustration of the mouth and vestibule--although the vestibule isn't labeled, you can get some idea of where it lies. You can also see that the parotid duct is opening into this space . The parotid duct drains the parotid gland; it crosses the masseter to enter the cheek and drain into the oral cavity. The parotid duct drains into the cheek near the upper 2nd molar tooth.

The tongue is not in the vestibule--it's behind the teeth. The submandibular duct opens into the area under the tongue, near the tongue's frenulum. The sublingual fold is also underneath the tongue; this is the fold that contains the openings for the ducts of the sublingual gland to drain into the mouth. The uvula is the "punching bag" hanging from the soft palate at the back of the oral cavity.
When one presses the tongue tip against the anterior (incisor) teeth, which of the following muscles must contract?
Styloglossus
Hyoglossus
Genioglossus
Superior longitudinal
Verticalis
The correct answer is: genioglossus
Genioglossus is a large, fan shaped muscle with many actions: its inferior fibers protrude the tongue, its middle fibers depress the tongue, and its superior fibers draw the tip back and down. So, since the tongue is being protruded, the inferior fibers of genioglossus must be contracting. Styloglossus retracts and elevates the tongue. Hyoglossus depresses the sides of tongue and retracts the tongue. The superior longitudinal and verticalis muscles are intrinsic muscles of the tongue. They help shape the tongue for speech and chewing. All of these muscles are innervated by the hypoglossal nerve!
The muscle responsible for raising the floor of the mouth in the early stages of swallowing is the:
genioglossus
geniohyoid
hyoglossus
mylohyoid
palatoglossus
The correct answer is: mylohyoid
The mylohyoid muscle elevates the hyoid bone and the tongue and depresses the mandible. It is a muscle in the floor of the mouth that helps elevate the floor of the mouth, so this is the correct answer. The genioglossus is a large, fan shaped tongue muscle that protrudes tongue with its inferior fibers and depresses the tongue with its middle fibers. Geniohyoid is a small muscle in the floor of the mouth that elevates the hyoid and depresses the mandible. Hyoglossus is a tongue muscle that depresses the sides of the tongue and retracts the tongue. Finally, palatoglossus is a palate muscle, innervated by the vagus nerve. It elevates and retracts the tongue.
The cell bodies of the postganglionic parasympathetic neurons innervating the sublingual gland are found in which of the following ganglia?
ciliary
otic
submandibular
superior cervical
trigeminal
The correct answer is: submandibular ganglion
The submandibular ganglion contains the cell bodies of the postganglionic parasympathetic neurons that innervate the sublingual and submandibular gland. Preganglionic parasympathetic fibers arrived at the submandibular ganglion via the chorda tympani. These neurons then synapsed with the cell bodies in the submandibular ganglion, and the postganglionic fibers traveled out to provide secretomotor innervation to the sublingual and submandibular glands.

The ciliary ganglion receives preganglionic parasympathetic fibers from the inferior division of the oculomotor nerve. The postganglionic fibers from this ganglion leave via short ciliary nerves and innervate sphincter pupillae and the ciliary muscle of the eye. Also remember that sensory and sympathetic fibers are passing through the ciliary ganglion and distributing on the short ciliary nerves, but these fibers never synapse in that ganglion. The otic ganglion is the ganglion where parasympathetic fibers synapse before innervating the parotid gland. The lesser petrosal nerve is a branch of the glossopharyngeal nerve (CN IX) that carries presynaptic parasympathetic fibers to the otic ganglion. These fibers synapse in the otic ganglion, and the postsynaptic fibers travel on the auriculotemporal nerve to the parotid gland.

The superior cervical ganglia is at the level of the C1 and C2 vertebrae--it is the highest ganglia in the sympathetic chain. Sympathetic nerves from this ganglia go to form the internal carotid sympathetic plexus along the internal carotid artery. Finally, the trigeminal ganglia (also called the semilunar ganglia) a sensory ganglion equivalent in histological structure and function to a dorsal root ganglion. The cell bodies of the afferent fibers of the trigeminal nerve reside in this ganglion.
A 46-year-old female patient comes to the emergency department complaining of pain in the area just below her mandible on the right side of her face. She says that the pain is particularly severe when she eats. The area of the submandibular gland is tender and swollen, as is the area in the floor of her mouth lateral to the tongue. You suspect a stone in the submandibular duct, and a plain film radiograph shows a density in that region consistent with a stone. In order to remove the stone, the duct must be incised in the floor of the mouth. What nerve, that loops around the duct, is in danger in such an incision?
Chorda Tympani
Glossopharyngeal
Hypoglossal
Internal branch of the superior laryngeal
Lingual
The correct answer is: Lingual
The lingual nerve is found in the floor of the mouth--it wraps around the submandibular duct. So that nerve might be injured as you try to remove the stone from the submandibular duct. The lingual nerve contains general sensory fibers for the anterior 2/3 of the tongue, and fibers from the chorda tympani that provide taste to the anterior 2/3 of the tongue. All of these sensory components might be lost if the lingual nerve was damaged.

At the point where the lingual nerve is wrapping around the submandibular gland, the fibers from chorda tympani have already joined the lingual nerve.This means that fibers from chorda tympani might be damaged, but the chorda tympani itself is not near the submandibular duct. The glossopharyngeal nerve provides sensory and taste innervation to the posterior 1/3 of the tongue and sensory innervation to the pharynx--it is not associated with the submandbular duct. The hypoglossal nerve is in the sublingual space, just like the submandibular duct. However, it is deeper in the sublingual space and does not wrap around the duct. Remember--the lingual nerve is a much more superficial structure in the sublingual space. Finally, the internal branch of the superior laryngeal nerve crosses the thyrohyoid membrane to provide sensory innervation to the mucosa of the pharynx, superior to the vocal folds.
In accessing the submandibular gland in the submandibular triangle, what vessel coursing through the gland and triangle would need to be protected?
External jugular vein
Facial artery
Maxillary artery
Retromandibular vein
Superior thyroid artery
The correct answer is: facial artery
The submandibular triangle is formed by the lower border of the mandible and the anterior and posterior bellies of the digastric muscle. The facial artery and facial vein course through this triangle. The facial artery lies deep to the superficial part of the submandibular gland and wraps around the mandible. The facial vein is superficial to the gland--see Netter Plate 27 for a picture. The external jugular vein is a superficial vein on the lateral side of the neck. The maxillary artery is a branch of the external carotid artery that is the main source of blood to the infratemporal fossa. The retromandibular vein is a vein that passes through the parotid gland, along with the facial artery and facial nerve. Finally, the superior thyroid artery is a branch of the external carotid that supplies blood to the superior pole of the thyroid.
All of the following may be found in the paralingual space EXCEPT:
Hypoglossal nerve
Lingual nerve
Sublingual gland
Submandibular gland duct
Superficial lobe of the submandibular gland
The correct answer is: Superficial lobe of the submandibular gland
The paralingual space is found in the floor of the mouth, deep to the mylohyoid muscle. The space is bounded by the lateral tongue, the hyoid bone, and the oral mucosa. Remember--the submandibular gland has a deep and superficial portion. The deep portion of the submandibular gland is found deep to the mylohyoid muscle, while the superficial portion of the submandibular gland is found superficial to the mylohyoid muscle. So, the deep portion of the submandibular gland is in the paralingual space, while the superficial portion of the submandibular gland is not. See Netter Plate 55 for a picture of this.

The paralingual space also contains the submandibular duct and sublingual caruncle, the lingual nerve and submandibular ganglion, the hypoglossal nerve, and the sublingual gland and fold.
An athlete has a knee injury, and the doctor performs a "drawer test" by pulling and pushing on the leg with the knee flexed. If the leg translates anteriorly, i. e. "gives" or moves anteriorly when the leg is pulled anteriorly, what joint structure is most likely injured?
anterior cruciate ligament
lateral collateral ligament
medial collateral ligament
medial meniscus
posterior cruciate ligament
The correct answer is: Anterior cruciate ligament
The drawer test is designed to test the integrity of the anterior and posterior cruciate ligaments. It involves firmly grasping the leg with both hands just below the knee, with the thumbs on the tibial tuberosity. With the knee flexed, the examiner pushes and pulls the leg in a line parallel to the long axis of the femur. If the leg moves too far anteriorly, this indicates a ruptured anterior cruciate ligament. If the leg moves too far posteriorly, the posterior cruciate ligament is probably ruptured. In this case, the leg is moving anteriorly, so the anterior cruciate ligament must be injured.

ACL tears often result from a blow to the lateral side of the knee. In these types of incidents, there are three structures that are commonly torn: the anterior cruciate ligament, the medial collateral ligament, and the medial meniscus. Remember, the MCL and the medial meniscus are attached, so an injury to the MCL will usually disrupt the meniscus. This constellation of injuries is sometimes referred to as "the terrible triad."
In an auto accident, the patient's knee strikes the dashboard which in turn pushes the head of the femur posteriorly out of its socket. Which ligament is most likely ruptured by this posterior dislocation?
iliofemoral
ischiofemoral
pubofemoral
transverse acetabular
The correct answer is: ischiofemoral
Three ligaments comprise the hip joint capsule: pubofemoral, iliofemoral, and ichiofemoral. The iliofemoral ligament forms the anterior wall of the hip capsule, the pubofemoral lies beneath the joint, and the ischiofemoral ligament forms the posterior free margin of the hip capsule. Since the ischiofemoral ligament is the posterior edge of the hip capsule, it is the ligament that is most likely to be injured if the femur is dislocated posteriorly. The transverse acetabular ligament is a structure that bridges the acetabular notch and forms a bridge over the artery in the ligament of the femoral head.

The fibrous capsule around the hip joint does not attach to the posterior aspect of the femur. This means that the femur can dislocate backwards. The hip joint is also less stable and more prone to dislocating when it is flexed (like when a person is sitting). So, posterior femoral dislocations are classically seen in car accidents, when a seated passenger bangs his or her knees into the dashboard and the femur is pushed posteriorly.
To test the integrity of the knee joint, a physician pulls anteriorly on the flexed leg of his patient. This "drawer" test is positive if the leg moves excessively anteriorward. This would indicate a weakness in or rupture of the:
medial meniscus
posterior cruciate ligament
fibular collateral ligament
medial collateral ligament
anterior cruciate ligament
The correct answer is: anterior cruciate ligament
The drawer test is designed to assess the integrity of the anterior and posterior cruciate ligaments. It involves firmly grasping the leg with both hands just below the knee, with the thumbs on the tibial tuberosity. With the knee flexed, the examiner pushes and pulls the leg in a line parallel to the long axis of the femur. If the leg moves too far anteriorly, this indicates a ruptured anterior cruciate ligament. If the leg moves too far posteriorly, the posterior cruciate ligament is probably ruptured. In this case, the leg is moving anteriorly, so the anterior cruciate ligament must be injured.

You should understand the drawer test and what different findings mean!
One of the menisci of the knee is often injured in a sprain of the knee because the:
Anterior cruciate ligament is attached to the lateral meniscus
Anterior cruciate ligament is attached to the medial meniscus
Lateral collateral ligament is attached to the lateral meniscus
Medial collateral ligament is attached to the medial meniscus
Posterior cruciate ligament is attached to the lateral meniscus
The correct answer is: medial collateral ligament is attached to the medial meniscus
The medial collateral ligament (tibial collateral ligament) is attached to the medial meniscus of the knee. This is clinically significant because an injury or tear to the MCL can result in tearing to the medial meniscus. This type of injury can be caused by a blow to the lateral side of the knee. The lateral collateral ligament (fibular collateral ligament) is not attached to the lateral meniscus--remember, this ligament is separated from the joint capsule by the tendon of popliteus.The anterior and posterior cruciate ligaments are intracapsular ligaments found within the knee capsule. These ligaments connect the tibia to the femur and are not attached to either of the menisci.
A soldier developed "fallen arches" from marching with a heavy pack in boots that lacked arch support. The ligament that normally supports the head of the talus and is primarily responsible for holding up the medial longitudinal arch of the foot is the:
calcaneometatarsal
deltoid
long plantar
plantar calcaneonavicular (spring)
short plantar
The correct answer is: plantar calcaneonavicular (spring)
The plantar calcaneonavicular ligament (spring ligament) connects the sustentaculum tali with the plantar surface of the navicular bone. It provides major support for the medial longitudinal arch of the foot, so this must be the ligament that the soldier injured. The calcaneometatarsal ligament is a superficial ligament in the foot--it is the lateral band of the plantar aponeurosis. The long plantar ligament connects calcaneus with cuboid and the bases of the lateral 3 metatarsals. The long plantar ligament is generally important in maintaining all the arches of the foot, but the spring ligament is specifically associated with the medial longitudinal arch and the head of the talus. The deltoid ligament is on the medial side of the ankle--this ligament stabilizes the ankle joint during eversion and prevents the ankle from dislocating. The short plantar ligament is deep to the long plantar ligament--it extends from the anterior aspect of the inferior surface of the calcaneus to the inferior surface of the cuboid.
While water skiing in Florida following final exams, a medical student falls and twists her ankle. Her foot is forcibly everted, which could cause a sprain of which ligament?
Anterior talofibular
Anterior tibiofibular
Calcaneofibular
Deltoid
Plantar calcaneonavicular (spring ligament)
The correct answer is: Deltoid
The deltoid ligament connects medial malleolus with talus, navicular & calcaneus. It is on the medial side of the ankle joint, and its role is to prevent the ankle from dislocating when forcibly everted. So, if a foot was strongly everted, the deltoid ligament might tear.

The anterior talofibular ligament connects the lateral malleolus with the talus anterolaterally, and the calcaneofibular ligament connects the lateral malleolus with calcaneus. Both of these ligaments contribute to the lateral ligaments of the ankle, and they can be damaged during forced inversion of the foot.

The anterior tibiofibular ligament connects the tibia and the fibula, providing support to the distal ends of these bones. The plantar calcaneonavicular ligament (spring ligament) connects the sustentaculum tali with the plantar surface of the navicular bone. It provides major support for the medial longitudinal arch of the foot.
During the Orange Bowl, the national championship football game between Florida State and a team from a land-locked state, a player is blocked from behind during a kick-off return, injuring his medial collateral ligament. The team doctor tests his knee by pulling anteriorly on the leg with the knee flexed. If the leg translates (moves) forward significantly, this indicates damage to which structure?
Anterior cruciate ligament
Lateral collateral ligament
Medial meniscus
Medial collateral ligament
Posterior cruciate ligament
The correct answer is: Anterior cruciate ligament
The doctor is performing the drawer test on this football player. The drawer test is a test designed to evaluate the integrity of the anterior and posterior cruciate ligaments. It involves firmly grasping the leg with both hands just below the knee, with the thumbs on the tibial tuberosity. With the knee flexed, the examiner pushes and pulls the leg in a line parallel to the long axis of the femur. If the leg moves too far anteriorly, this indicates a ruptured anterior cruciate ligament. If the leg moves too far posteriorly, the posterior cruciate ligament is probably ruptured. In this case, the leg is moving anteriorly, so the anterior cruciate ligament must be injured--which is what happened in this case.

ACL tears often result from a blow to the lateral side of the knee; a blow to the medial side of the knee may signal an injury to the lateral collateral ligament. In ACL injuries, there are three structures that are commonly torn: the anterior cruciate ligament, the medial collateral ligament, and the medial meniscus. Remember, the MCL and the medial meniscus are attached, so an injury to the MCL will usually disupt the medial meniscus, too. This constellation of injuries is sometimes referred to as "the terrible triad." (Although these injuries tend to happen together, the anterior drawer test is specifically evaluating the integrity of the ACL, and you can't just assume that all of the structures are damaged after a positive anterior drawer test.)
A young man involved in a head-on automobile collision had his flexed knee hit the dashboard of the car. He was later found to have a major instability of the knee, in that his tibia could be moved posteriorly relative to the femur. What ligament was likely damaged?
Lateral collateral ligament
Deltoid
Medial collateral ligament
Anterior cruciate ligament
Posterior cruciate ligament
The correct answer is: Posterior cruciate ligament
When the tibia can be pushed posteriorly in relation to the femur, a patient has a positive posterior drawer test. The drawer test evaluates the integrity of the anterior and posterior cruciate ligaments. It involves firmly grasping the leg with both hands just below the knee, with the thumbs on the tibial tuberosity. With the knee flexed, the examiner pushes and pulls the leg in a line parallel to the long axis of the femur. If the leg moves too far anteriorly, this indicates a ruptured anterior cruciate ligament. If the leg moves too far posteriorly, the posterior cruciate ligament is probably ruptured. In this case, the leg is moving posteriorly, so the posterior cruciate ligament must be injured.

PCL injuries commonly occur when a person lands on the tibial tuberosity with a flexed knee. They may also occur in head-on collisions when the proximal end of the tibia strikes the dashboard.
During a basketball game, the center of the team went up for a rebound and when coming down, her foot landed on the foot of another player, sharply everting it. She limped off the floor, having severely sprained the medial side of her ankle. Which ligament was injured?
Calcaneofibular
Deltoid
Short plantar ligament
Plantar calcaneonavicular
Tibial collateral ligament
The correct answer is: Deltoid
The deltoid ligament connects medial malleolus with talus, navicular & calcaneus. It is on the medial side of the ankle joint, and its role is to prevent the ankle from dislocating when forcibly everted. So, if a foot was strongly everted (which is what happened to this basketball player), the deltoid ligament might tear. The calcaneofibular ligament connects the lateral malleolus with calcaneus. It is one of the lateral ligaments of the ankle, and it can be damaged by forcibly inverting the foot. The short plantar ligament is deep to the long plantar ligament--it extends from the anterior aspect of the inferior surface of the calcaneus to the inferior surface of the cuboid. It is a ligament supporting the foot, not the ankle. The plantar calcaneonavicular ligament (spring ligament) connects the sustentaculum tali with the plantar surface of the navicular bone. It provides major support for the medial longitudinal arch of the foot, but it's not important for ankle stability. The tibial collateral ligament (medial collateral ligament) provides support on the medial side of the knee. It is connected to the medial meniscus, and both structures can be damaged with a blow to the lateral side of the knee.
Which ligament limits extension at the hip joint?
Iliofemoral
Ligamentum capitis femorus
Pubofemoral
Zona orbicularis
The correct answer is: Iliofemoral
The iliofemoral ligament is a Y-shaped ligament extending from the anterior inferior iliac spine to the anterior surface of the intertrochanteric line of the femur. It prevents hyperextension of the hip joint during standing by screwing the femoral head into the acetabulum. (The ischiofemoral ligament also helps to prevent hyperextension of the hip joint by screwing the femoral head into the acetabulum.) Ligamentum capitis femoris attaches the head of the femur to the acetabular fossa. The pubofemoral ligament connects the pubic portion of the rim of the bony acetabulum to the medial surface of the femoral neck. It prevents overabduction of the hip joint. Zona orbicularis is a band of circularly oriented ligamentous fibers that reinforce the capsule of the hip joint--it helps keep the head of the femur in its socket.
In injuries of the knee, the medial meniscus is frequently torn because it is firmly attached to which structure?
Anterior cruciate ligament
Fibular collateral ligament
Tibial collateral ligament
Patellar ligament
Patellar retinaculum
The correct answer is: Tibial collateral ligament
The tibial collateral ligament (medial collateral ligament) is attached to the medial meniscus. So, a tear to the MCL will often result in a tear to the medial meniscus. These structures are often damaged following a lateral blow to the knee. The anterior cruciate ligament also may be damaged after a lateral blow to the knee. In fact, the MCL, ACL, and medial meniscus are often all injured together, resulting in a set of injuries known as the "terrible triad." However, you should remember that the ACL is not attached to the medial meniscus! The fibular (lateral) collateral ligament is found on the lateral side of the knee; it can be damaged by a blow to the medial side of the knee.

The patellar ligament is the tendon of insertion of the quadriceps femoris muscle--it connects the patella to the tibial tuberosity. Remember--the patella is a sesamoid bone in the tendon of quadriceps femoris. The patellar retinacula stabilize the knee joint.
During an intramural baseball game a player is hit in the side of the head, between the eye and the ear. He immediately loses consciousness, wakes up momentarily and then becomes comatose. He is rushed to the ER and is immediately given a CT scan. The scan shows a skull fracture and an accumulation of blood between the dura and the cranial bone on the side of his head, compressing his cerebrum. He is rushed to surgery where a hole is bored into his skull to relieve the pressure. After a few tense hours, he regains consciousness and has an uneventful recovery. The hemorrhage from the fracture would be described as:
Epidural
Intracerebral
Subaponeurotic
Subarachnoid
Subdural
The correct answer is: Epidural
An epidural hemorrhage is a hemorrhage of blood into the space between the dura and the skull. These hemorrhages are usually caused by rupturing the middle meningeal artery, which supplies blood to the dura and the bones of the cranial vault. This hemorrhage results in compression of the dura mater and the brain; if it is not drained, it may result in the brain herniating through the tentorium and death. An intracerebral hemorrhage is a hemorrhage within the cerebral hemispheres. A subaponeurotic hemorrhage could be a collection of blood under the aponeurosis of the scalp, but this is not really a brain hemorrhage and is not as clinically significant as the other answer choices. A subarachnoid hemorrhage is an acute condition where blood collects in the area between the pia mater and arachnoid mater. This is often secondary to a head injury or a ruptured aneurysm. A subdural hemorrhage is characterised by a collection of blood beneath the dura, often caused by a head injury.
Infections may spread from the nasal cavity to the meninges along the olfactory nerves, as its fibers pass from the mucosa of the nasal cavity to the olfactory bulb via the:
Cribriform plate of the ethmoid
Crista galli
Foramen caecum
Superior orbital fissure
The correct answer is: cribriform plate of the ethmoid.
The olfactory nerve exits the skull through the cribriform plate of the ethmoid bone--an infection in the nasal cavity may be carried to the olfactory bulb by the nerves that are passing through the cribriform plate. The crista galli is a ridge on the ethmoid bone between the two sides of the cribriform plate; it provides an anchor for the falx cerebri. Foramen cecum is a small hole in the frontal bone near the anterior end of the crista galli--it transmits an emissary vein. Finally, the superior orbital fissure is a hole in the sphenoid bone that transmits many cranial nerves: the oculomotor nerve (CN III), the trochlear nerve (CN IV), the ophthalmic division of the trigeminal nerve (CN V1) and the abducens nerve (CN VI) all pass through the superior orbital fissure.
A patient who has sustained a fracture to the middle cranial fossa following a fall from a height, might have any of these nerves injured EXCEPT:
Trigeminal
Oculomotor
Abducens
Trochlear
Hypoglossal
The correct answer is: hypoglossal
The middle cranial fossa is the part of the skull that supports the temporal lobes of the brain. It is made of the greater wings of the sphenoid and squamous part of the temporal bones laterally and the petrous part of the temporal bones posteriorly. See Netter Plate 6 and 7 for a better picture of this.

Several cranial nerves enter foramina in the middle cranial fossa; all of these nerves might have been damaged in the fall. The trigeminal nerve (CN V) has three divisions that all leave through spaces in the middle cranial fossa. V1, the ophthalmic division, exits through the superior orbital fissure; V2, the maxillary division, leaves through foramen rotundum; V3, the mandibular division, leaves through foramen ovale. The oculomotor nerve (CN III) crosses through the superior orbital fissure, along with abducens (CN VI), the trochlear nerve (CN IV) and the ophthalmic division of the trigeminal nerve (CN V1). So, all of these nerves might have been damaged in the fall.

The hypoglossal nerve, however, leaves the base of the skull by passing through the hypoglossal canal, which is in the occipital bone and the posterior cranial fossa. It is not likely that this nerve was injured in the fall.
The most likely source of blood in a patient with an epidural hemorrhage is:
Vertebral artery
Middle meningeal artery
Superior cerebral veins
Anterior cerebral artery
Circle of Willis
The correct answer is: middle meningeal artery
The middle meningeal artery supplies most of the dura mater and the bones of the cranial vault. It is this artery or vein that is usually ruptured in an epidural hemorrhage. The vertebral artery carries blood to the deep neck, cervical spinal cord, and hindbrain. It does not supply blood to the dura. Superior cerebral veins drain blood into the superior sagittal sinus. When injured, they bleed into the subdural space and cause a subdural hematoma. The anterior cerebral artery supplies blood to the frontal pole of the brain. It is not found near the epidural space. Finally, the circle of Willis is an important anastomosis at the base of the brain between the following arteries: posterior cerebral arteries, posterior communicating arteries, internal carotid arteries, anterior cerebral arteries, and anterior communicating arteries.
A 35-year-old man was admitted to the hospital complaining of double vision (diplopia), inability to see close objects, and blurred vision in the right eye. A vertebrobasilar angiogram revealed an aneurysm of the superior cerebellar artery close to its origin on the right side. The doctor attributed the symptoms to the compression of an adjacent cranial nerve by the aneurysm. The compressed nerve is the:
Abducens (CN VI)
Oculomotor (CN III)
Optic (CN II)
Trigeminal (CN V)
Trochlear (CN IV)
The correct answer is: Oculomotor (CN III)
Given the patient's symptoms, it seems that some nerve involving vision and the ability to control the eye has been injured. Now, you need to think about which nerve might be damaged by an aneurysm of the superior cerebellar artery. The oculomotor nerve, which innervates the superior rectus, medial rectus, inferior rectus, and inferior oblique muscles, passes between the posterior cerebral artery and the superior cerebellar artery. It could be injured if there was enlargement of or damage to either of these vessels. None of the other cranial nerves are in the right position to be injured from an aneurysm of the superior cerebellar artery.
You have been asked to assess the neurological deficit that might exist in a patient diagnosed with cavernous sinus thrombosis. You will focus your examination on cranial nerves related to the sinus that includes all the following EXCEPT:
Abducens (CN VI)
Facial (CN VII)
Oculomotor (CN III)
Ophthalmic division of the trigeminal nerve (CN V1)
Trochlear (CN IV)
The correct answer is: Facial (CN VII)
The cavernous sinus is a venous sinus of the brain, lateral to the body of the sphenoid bone. All of the cranial nerves and vessels that pass out of the skull at the superior orbital fissure pass through the cavernous sinus. This includes the oculomotor nerve, the trochlear nerve, the ophthalmic division of the trigeminal nerve, and the abducens nerve. The internal carotid artery also passes through the cavernous sinus. Since three of the nerves in the cavernous sinus control the motions of the extraocular muscles, testing eye movements would be a good way to see if nerves in the cavernous sinus were disrupted. The ophthamic division of the trigeminal nerve, which supplies cutaneous sensation to the skin of the upper face, could be tested by evaluating the sensations on the forehead.

The facial nerve is not associated with the cavernous sinus. It passes through the internal acoustic meatus and exits the skull through the stylomastoid foramen. So, you would not need to test to see if the facial nerve was intact.
The glossopharyngeal nerve exits the skull via what opening?
Foramen ovale
Carotid canal
Jugular foramen
Hypoglossal canal
Stylomastoid foramen
The correct answer is: Jugular foramen
The glossopharyngeal nerve (CN IX), vagus (CN X) and accessory nerve (CN XI) exit the skull at the jugular foramen. The posterior meningeal artery enters the skull through this space. The mandibular division of the trigeminal nerve (V3) exits the skull through foramen ovale. The carotid canal is the place where the internal carotid artery and the internal carotid nerve plexus enter the skull. The hypoglossal canal is where the hypoglossal nerve (CN XII) leaves the skull. The stylomastoid foramen is the hole that the facial nerve (CN VII) uses to exit the skull.
A person develops a cavernous sinus thrombosis. Because of its relationship to the sinus, which cranial nerve might be affected?
Abducens
Facial
Mandibular V3
Olfactory
Optic
The correct answer is: Abducens (CN VI)
The cavernous sinus is a venous sinus of the brain, lateral to the body of the sphenoid bone. All of the cranial nerves and vessels that pass out of the skull at the superior orbital fissure pass through the cavernous sinus. This includes the abducens nerve, the oculomotor nerve, the trochlear nerve, and the ophthalmic division of the trigeminal nerve. Any of these nerves might be affected by a cavernous venous sinus thrombosis.

The facial nerve is not associated with the cavernous sinus. It passes through the internal acoustic meatus and exits the skull through the stylomastoid foramen. The mandibular division of the trigeminal nerve (V3) exits the skull through foramen ovale. The olfactory nerves enter the skull through the cribriform plate of the ethmoid bone. The optic nerve exits the skull through the optic canal. None of these nerves are associated with the cavernous sinus.
All of the following nerves exit the cranial cavity by way of bony openings located in the middle cranial fossa EXCEPT:
Abducens
Trochlear
Oculomotor
Trigeminal
Facial
The correct answer is: facial
The middle cranial fossa is the part of the skull that supports the temporal lobes of the brain. It is made of the greater wings of the sphenoid and squamous parts of the temporal bones laterally and the petrous parts of the temporal bones posteriorly. See Netter Plate 6 and 7 for a better picture of this.

Several cranial nerves enter foramina in the middle cranial fossa. The abducens (CN VI) crosses through the superior orbital fissure, along with the oculomotor nerve (CN III), trochlear nerve (CN IV) and ophthalmic division of the trigeminal nerve (CN V1). The trigeminal nerve (CN V) has three divisions that all leave through spaces in the middle cranial fossa. V1, the ophthalmic division, exits through the superior orbital fissure; V2, the maxillary division, leaves through foramen rotundum; V3, the mandibular division, leaves through foramen ovale.

The facial nerve, however, leaves the base of the skull by passing through the internal acoustic meatus, which is in the part of the temporal bone that is in the posterior cranial fossa.
What are the 3 paired cartilages of the laryngeal skeleton?
corniculate, cuneiform, and arytenoid